Annotation of db/baza/izmi1718.txt, revision 1.2

1.1       rubashki    1: Чемпионат:
                      2: Синхронный турнир "Из Минска с любовью" - 2017/18
                      3: 
                      4: Дата:
                      5: 13-Oct-2017 - 13-Apr-2018
                      6: 
                      7: Тур:
                      8: 1 этап
                      9: 
                     10: Дата:
                     11: 13-Oct-2017
                     12: 
                     13: Редактор:
                     14: 1-18 - Юрий Разумов; 19-36 - Дмитрий Буценец (Могилёвцы)
                     15: 
                     16: Инфо:
                     17: Юрий Разумов благодарит Артема Гулецкого, Андрея Забавина, Марию
                     18: Кленницкую, Евгения Миротина, команду "Утро в Запретном лесу". Дмитрий
                     19: Буценец благодарит за тестирование и помощь в подготовке пакета: Михаила
                     20: Савченкова, Серафима Шибанова, Галину Пактовскую (все - Москва), Марию
                     21: Минакову (Санкт-Петербург), Евгения Миротина, Михаила Карпука, Юрия
                     22: Разумова, Артема Гулецкого, Рената Рустамова, Дарью Данилевич, Евгения
                     23: Лешковича, Даниила Шункевича, Сергея Дубелевича, а также команды
                     24: "Одушевленные аэросани" и "Мамонтлошечка" (все - Минск).
                     25: 
                     26: Вопрос 1:
                     27: Застенчивая героиня компьютерной игры не так давно перешла в новую
                     28: школу. Комментируя афишу на стене школы, она надеется, что ИКС никогда
                     29: не закончится. Какое русское слово мы заменили ИКСОМ?
                     30: 
                     31: Ответ:
                     32: Октябрь.
                     33: 
                     34: Комментарий:
                     35: Постер приглашал на празднование Хэллоуина.
                     36:    Начинаем октябрьский этап синхрона "Из Минска с любовью".
                     37: 
                     38: Источник:
                     39: Life is Strange, Episode 1.
                     40: 
                     41: Автор:
                     42: Юрий Разумов
                     43: 
                     44: Вопрос 2:
                     45: Алистер Кроули успешно создавал репутацию могущественного мага. Какими
                     46: двумя эпитетами называет Кроули российский литературовед Иван Мартов?
                     47: 
                     48: Ответ:
                     49: Великий и ужасный.
                     50: 
                     51: Комментарий:
                     52: То же делал Гудвин.
                     53: 
                     54: Источник:
                     55: https://gorky.media/context/parizh-eto-tsirtseya-prevrashhayushhaya-russkih-v-svinej/
                     56: 
                     57: Автор:
                     58: Антон Воронько
                     59: 
                     60: Вопрос 3:
                     61: Книга Пола Оффита об опасных научных открытиях называется "ЕЕ
                     62: лаборатория". Напишите ЕЕ имя.
                     63: 
                     64: Ответ:
                     65: Пандора.
                     66: 
                     67: Комментарий:
                     68: Слово "открытия" - подсказка.
                     69: 
                     70: Источник:
                     71: Paul A. Offit, Pandora's Lab: Seven Stories of Science Gone Wrong.
                     72: 
                     73: Автор:
                     74: Юрий Разумов
                     75: 
                     76: Вопрос 4:
                     77: Рост цен в одной отрасли нередко объясняют не только дорогими
                     78: исследованиями, но и большИм числом законов и правил. Блогер Скотт
                     79: Александр отмечает, что это вряд ли единственная причина, ведь ИХ
                     80: деятельность регулируется гораздо слабее, а цены растут так же быстро.
                     81: Назовите ИХ одним словом.
                     82: 
                     83: Ответ:
                     84: Ветеринары.
                     85: 
                     86: Комментарий:
                     87: Современная медицина (особенно в развитых странах) регулируется огромным
                     88: количеством законов, а разработка новых лекарств требует дорогостоящих
                     89: тестирований. Лечение животных не так зарегулировано, однако и там цены
                     90: растут.
                     91: 
                     92: Источник:
                     93: http://slatestarcodex.com/2017/02/09/considerations-on-cost-disease/
                     94: 
                     95: Автор:
                     96: Юрий Разумов
                     97: 
                     98: Вопрос 5:
                     99: В 1997 году на поверхность Марса был спущен марсоход "Sojourner"
                    100: [соджОрнер], оснащенный несколькими камерами. День высадки марсохода
                    101: веб-сайт НАСА называет днем, когда ОН остановился. Назовите ЕГО одним
                    102: словом.
                    103: 
                    104: Ответ:
                    105: Интернет.
                    106: 
                    107: Комментарий:
                    108: В 1997 году широкополосное соединение было редкостью, а фотографии,
                    109: сделанные марсоходом, хотели увидеть многие. Французскому правительству
                    110: пришлось прямо попросить граждан прекратить открывать сайт НАСА, чтобы
                    111: телефонная связь в стране заработала стабильнее.
                    112: 
                    113: Источник:
                    114: https://www.nasa.gov/specials/pathfinder20/
                    115: 
                    116: Автор:
                    117: Юрий Разумов
                    118: 
                    119: Вопрос 6:
                    120: Когда больному королю из пьесы Эжена Ионеско кажется, что ему стало
                    121: лучше, его жена возражает и говорит, что тот умрет через полтора часа, а
                    122: именно - ПРОПУСК. Заполните пропуск тремя словами.
                    123: 
                    124: Ответ:
                    125: В конце спектакля.
                    126: 
                    127: Зачет:
                    128: В конце пьесы; в конце книги; на последней странице.
                    129: 
                    130: Источник:
                    131: Э. Ионеско. Король умирает. http://flibusta.is/b/103768/read
                    132: 
                    133: Автор:
                    134: Кирилл Шкурин
                    135: 
                    136: Вопрос 7:
                    137: В недавней статье Пэт МОрроу отмечает, что летом в большинстве лиг
                    138: перерыв, поэтому ЕГО выход очень помог букмекерской конторе Морроу.
                    139: Назовите ЕГО точно.
                    140: 
                    141: Ответ:
                    142: Седьмой сезон "Игры престолов".
                    143: 
                    144: Зачет:
                    145: Новый сезон "Игры престолов"; очередной сезон "Игры престолов".
                    146: 
                    147: Комментарий:
                    148: Букмекеры стали принимать ставки на смерти персонажей, а также на то,
                    149: кто в итоге займет Железный трон. Этот вопрос неслучайно стоит после
                    150: вопроса о смерти короля.
                    151: 
                    152: Источник:
                    153: https://www.theringer.com/2017/7/20/16078298/tv-game-of-thrones-gambling-bovada-99918a2bf471
                    154: 
                    155: Автор:
                    156: Юрий Разумов
                    157: 
                    158: Вопрос 8:
                    159: Словами "ДЕЛАТЬ ЭТО" мы заменили другие слова.
                    160:    Историк ДИэрмэд МаккАлок предполагает, что взгляды Парацельса со
                    161: временем становились радикальнее потому, что он не мог иметь много книг,
                    162: ведь часто ДЕЛАЛ ЭТО. Кто ДЕЛАЕТ ЭТО в известной задаче?
                    163: 
                    164: Ответ:
                    165: Коммивояжер.
                    166: 
                    167: Комментарий:
                    168: Парацельс не очень интересовался мнением отцов церкви - частично потому,
                    169: что постоянно переезжал из города в город (нередко - из-за ухудшившихся
                    170: отношений с местными властями) и возил с собой очень мало книг.
                    171: 
                    172: Источник:
                    173: Diarmaid MacCulloch, Reformation: Europe's House Divided 1490-1700.
                    174: 
                    175: Автор:
                    176: Юрий Разумов
                    177: 
                    178: Вопрос 9:
                    179: Описывая след, оставляемый винтами ледокола, Дмитрий ЛОбусов упоминает
                    180: ЕГО. На Олимпиаде 2010 года половина медалей, разыгранных в НЕМ,
                    181: досталась спортсменам из США. Назовите ЕГО.
                    182: 
                    183: Ответ:
                    184: Хаф-пайп.
                    185: 
                    186: Зачет:
                    187: Хафпайп; half-pipe.
                    188: 
                    189: Комментарий:
                    190: Если вращать достаточно крепкий винт над поверхностью льда, получится
                    191: что-то очень похожее на хаф-пайп. Слово "половина" - подсказка.
                    192: 
                    193: Источник:
                    194:    1. https://topwar.ru/95774-sokrushayuschiy-ldy.html
                    195:    2. https://en.wikipedia.org/wiki/Snowboarding_at_the_2010_Winter_Olympics
                    196: 
                    197: Автор:
                    198: Сергей Алексетович
                    199: 
                    200: Вопрос 10:
                    201: В романе "Флатландия" описан двумерный мир, жители которого представляют
                    202: собой геометрические фигуры. В одной из глав романа рассказывается об
                    203: обычаях аристократов Флатландии. Какие два слова в предыдущем
                    204: предложении мы заменили словом "аристократов"?
                    205: 
                    206: Ответ:
                    207: Высших кругов.
                    208: 
                    209: Комментарий:
                    210: В романе Эдвина Эбботта общественное положение жителя Флатландии во
                    211: многом определяется количеством его углов. Высшие позиции в обществе
                    212: занимают круги.
                    213: 
                    214: Источник:
                    215: Э. Эбботт. Флатландия. http://flibusta.is/b/179946/read
                    216: 
                    217: Автор:
                    218: Кирилл Шкурин
                    219: 
                    220: Вопрос 11:
                    221: Полное имя писателя Эдвина Эбботта - Эдвин Эбботт Эбботт. Дело в том,
                    222: что его родители ПРОПУСК. Родители какого художника тоже ПРОПУСК?
                    223: 
                    224: Ответ:
                    225: [Анри де] Тулуз-Лотрека.
                    226: 
                    227: Комментарий:
                    228: Второе имя Эбботта - это девичья фамилия его матери. Она такая же, как
                    229: фамилия отца, потому что мать и отец были кузенами. То же справедливо и
                    230: для Тулуз-Лотрека, который, как считается, страдал от различных
                    231: заболеваний из-за кровосмесительной связи родителей.
                    232: 
                    233: Источник:
                    234:    1. https://ru.wikipedia.org/wiki/Эбботт,_Эдвин_Эбботт
                    235:    2. https://ru.wikipedia.org/wiki/Тулуз-Лотрек,_Анри_де
                    236: 
                    237: Автор:
                    238: Евгений Миротин
                    239: 
                    240: Вопрос 12:
                    241: Осмотрев заболевшего главного героя мультфильма "Джек и механика
                    242: сердца", специалист говорит, что сердце в полном порядке. Какие три
                    243: слова мы заменили словами "в полном порядке"?
                    244: 
                    245: Ответ:
                    246: Работает как часы.
                    247: 
                    248: Комментарий:
                    249: У героя мультфильма действительно часы вместо сломавшегося в детстве
                    250: сердца.
                    251: 
                    252: Источник:
                    253: Мультфильм "Джек и механика сердца" (2013), реж. Стефан Берла.
                    254: 
                    255: Автор:
                    256: Тимофей Прокопенко
                    257: 
                    258: Вопрос 13:
                    259: Дуплет.
                    260:    1. Героиня КрИстофа Хайна говорит, что ее монотонная жизнь движется
                    261: от весеннего события к осеннему и обратно. Назовите любое из этих
                    262: событий четырьмя словами.
                    263:    2. Подобную жизнь героиня КрИстофа Хайна сравнивает с ЕГО движением.
                    264: Назовите ЕГО одним словом.
                    265: 
                    266: Ответ:
                    267:    1. Переход на летнее время.
                    268:    2. Маятник.
                    269: 
                    270: Зачет:
                    271:    1. Переход на зимнее время.
                    272: 
                    273: Комментарий:
                    274: Жизнь героини настолько монотонна, что только перевод часов вносит в нее
                    275: какое-то разнообразие. Неудивительно, что эту метафору она дополняет
                    276: сравнением с маятником.
                    277: 
                    278: Источник:
                    279: Christoph Hein, Der Fremde Freund.
                    280: 
                    281: Автор:
                    282: Антон Воронько
                    283: 
                    284: Вопрос 14:
                    285: По ошибке купив поддельную картину Гойи, два испанца попытались продать
                    286: ее кому-то еще, однако вскоре после успешной сделки были арестованы.
                    287: Дело в том, что покупатель был ИМ. Назовите ЕГО словом с двумя корнями.
                    288: 
                    289: Ответ:
                    290: Фальшивомонетчик.
                    291: 
                    292: Комментарий:
                    293: Когда два брата, получив 1,7 миллиона швейцарских франков за картину,
                    294: попытались положить их на счет в банке, оказалось, что все эти деньги
                    295: такие же фальшивые, как и картина. Кроме того, братья еще до сделки
                    296: заплатили мошеннику 300 тысяч евро - он утверждал, что является только
                    297: агентом арабского шейха, и хотел получить процент от сделки.
                    298: 
                    299: Источник:
                    300: https://www.independent.co.uk/news/world/europe/con-mens-attempt-to-sell-forged-goya-painting-backfires-when-they-are-paid-with-fake-money-10063027.html
                    301: 
                    302: Автор:
                    303: Юрий Разумов
                    304: 
                    305: Вопрос 15:
                    306: Словами "ДЕЛАТЬ ЭТО" мы заменили другие слова.
                    307:    Героиня сериала готовится к трудному разговору с отцом и рассказывает
                    308: подруге, кАк в детстве отец наказал ее, застав курящей. Подруга в шутку
                    309: надеется, что отец не заставит героиню СДЕЛАТЬ ЭТО. Какой бог ДЕЛАЛ ЭТО?
                    310: 
                    311: Ответ:
                    312: Кронос.
                    313: 
                    314: Зачет:
                    315: Сатурн.
                    316: 
                    317: Комментарий:
                    318: Отец заставил героиню съесть сигарету. Своих детей, по легенде, съел
                    319: Кронос (он же Сатурн), что можно увидеть, например, на картине Гойи.
                    320: 
                    321: Источник:
                    322:    1. Телесериал "Друзья", s08e08, "The One With The Stripper".
                    323:    2. https://ru.wikipedia.org/wiki/Кронос
                    324: 
                    325: Автор:
                    326: Кирилл Шкурин
                    327: 
                    328: Вопрос 16:
                    329: Словом "ИКС" мы заменили другое слово.
                    330:    Отмечая юный возраст участников, Андрей Мовчан называет российские
                    331: протесты марта 2017 года "революцией школьных ИКСОВ". Персонажем какого
                    332: сериала является ИКС?
                    333: 
                    334: Ответ:
                    335: "Даша-путешественница".
                    336: 
                    337: Зачет:
                    338: "Dora the Explorer".
                    339: 
                    340: Комментарий:
                    341: Среди протестующих было много школьников и студентов, поэтому Мовчан
                    342: назвал акции "революцией рюкзаков". Рюкзак - один из главных героев
                    343: сериала о Даше-путешественнице.
                    344: 
                    345: Источник:
                    346:    1. https://republic.ru/posts/81172
                    347:    2. https://en.wikipedia.org/wiki/Dora_the_Explorer
                    348: 
                    349: Автор:
                    350: Антон Воронько
                    351: 
                    352: Вопрос 17:
                    353: Американский закон рекомендует финансовым компаниям обязать сотрудников
                    354: ДЕЛАТЬ ЭТО. В пример нередко приводят сотрудника банка ТошихИде ИгУчи,
                    355: который 11 лет не ДЕЛАЛ ЭТОГО, чтобы никто не заметил нелегальные сделки
                    356: на миллиарды долларов. Что такое "ДЕЛАТЬ ЭТО"?
                    357: 
                    358: Ответ:
                    359: Брать отпуск.
                    360: 
                    361: Зачет:
                    362: Уходить в отпуск; другие синонимичные глаголы.
                    363: 
                    364: Комментарий:
                    365: Сложные схемы мошенничества требуют постоянного контроля и присутствия,
                    366: поэтому финансовые регулирующие органы рекомендуют делать отпуска
                    367: обязательными или хотя бы присматривать за сотрудниками, которые не
                    368: пропускают ни одного рабочего дня.
                    369: 
                    370: Источник:
                    371: https://www.nytimes.com/1995/12/03/business/earning-it-workaholics-aren-t-the-only-ones-who-hate-vacations.html
                    372: 
                    373: Автор:
                    374: Юрий Разумов
                    375: 
                    376: Вопрос 18:
                    377: Последний вопрос первого тура.
                    378:    Герой романа Томаса Харди долго разглядывает трилобита, а также
                    379: отмечает неожиданно сильный ветер. Какой англоязычный термин появился
                    380: благодаря этой сцене?
                    381: 
                    382: Ответ:
                    383: Клиффхэнгер.
                    384: 
                    385: Комментарий:
                    386: Герой в прямом смысле висит на краю скалы, а потому может внимательно
                    387: рассмотреть окаменелость.
                    388: 
                    389: Источник:
                    390:    1. https://en.wikipedia.org/wiki/Cliffhanger
                    391:    2. Thomas Hardy, A Pair of Blue Eyes.
                    392: 
                    393: Автор:
                    394: Дарьяна Добрянская
                    395: 
                    396: Вопрос 19:
                    397: В одном сериале на двери в полицейском участке висит табличка, на
                    398: которой можно увидеть слово "disturb" [дистёрб] - "беспокоить". Что
                    399: изображено на табличке рядом с этим словом?
                    400: 
                    401: Ответ:
                    402: Пончик.
                    403: 
                    404: Зачет:
                    405: Donut; doughnut; дОнат. Незачет: Don't; do not.
                    406: 
                    407: Комментарий:
                    408: Слово "пончик" по-английски звучит как "donut" [дОнат] и созвучно с
                    409: отрицанием "не" в выражении "не беспокоить". Так Дэвид Линч в сериале
                    410: "Твин Пикс" обыгрывает известный стереотип о полицейских.
                    411: 
                    412: Источник:
                    413:    1. Телесериал "Твин Пикс", s03e03.
                    414:    2. http://welcometotwinpeaks.com/inspiration/donut-disturb-sign/
                    415: 
                    416: Автор:
                    417: Дарья Данилевич, Ренат Рустамов (Минск)
                    418: 
                    419: Вопрос 20:
                    420: В одной поэме гром без дождя назван ТАКИМ. Люди из касты БхАнту считают,
                    421: что ТАКАЯ женщина опасна для садов. Какое прилагательное мы заменили
                    422: словом "ТАКАЯ"?
                    423: 
                    424: Ответ:
                    425: Бесплодная.
                    426: 
                    427: Комментарий:
                    428: Примитивные племена часто сравнивают плодовитость природы с
                    429: плодовитостью женщины. В первой части вопроса речь идет о поэме Томаса
                    430: Элиота "Бесплодная земля".
                    431: 
                    432: Источник:
                    433:    1. Т. Элиот. Бесплодная земля. http://flibusta.is/b/282891/read
                    434:    2. С. Бовуар. Второй пол. http://flibusta.is/b/491900/read
                    435: 
                    436: Автор:
                    437: Дмитрий Буценец (Могилёвцы)
                    438: 
                    439: Вопрос 21:
                    440: Опасаясь исказить память о прошлом, герой КОрмака Маккарти старается
                    441: реже предаваться воспоминаниям. Аргументируя свою позицию, персонаж
                    442: приводит в пример игру. Какую?
                    443: 
                    444: Ответ:
                    445: "Испорченный телефон".
                    446: 
                    447: Зачет:
                    448: "Сломанный телефон".
                    449: 
                    450: Комментарий:
                    451: Персонаж считает, что каждое воспоминание наносит вред оригиналу, т.к.,
                    452: осознанно или нет, ты изменяешь то, что вспоминаешь.
                    453: 
                    454: Источник:
                    455: К. Маккарти. Дорога. http://flibusta.is/b/184846/read
                    456: 
                    457: Автор:
                    458: Дмитрий Буценец (Могилёвцы)
                    459: 
                    460: Вопрос 22:
                    461: Художник, разработавший карету для одного фильма, сравнил себя с НЕЙ
                    462: главной героини. В русском прокате фильм "Bella Mafia" [бЭлла мАфиа]
                    463: вышел под названием "ОНА". Назовите ЕЕ.
                    464: 
                    465: Ответ:
                    466: Крестная мать.
                    467: 
                    468: Зачет:
                    469: Крестная.
                    470: 
                    471: Комментарий:
                    472: Речь идет о фильме "Золушка", в котором, как и в сказке, фея-крестная
                    473: превращает тыкву в карету. Фильм "Крестная мать" снят на ту же тему, что
                    474: и "Крестный отец", но на 25 лет позже, и повествует о женщине, ставшей
                    475: главой мафии после убийства ее мужа.
                    476: 
                    477: Источник:
                    478:    1. https://ru.disney.wikia.com/wiki/Золушка_(2015)
                    479:    2. https://ru.wikipedia.org/wiki/Крёстная_мать_(фильм)
                    480: 
                    481: Автор:
                    482: Дмитрий Буценец (Могилёвцы), Никита Шевела (Минск)
                    483: 
                    484: Вопрос 23:
                    485: На верхнем уровне воронки продаж находятся потенциальные покупатели, а
                    486: на нижнем - успешные сделки. Если мАркетинг максимально эффективен, то
                    487: воронка СДЕЛАЕТ ЭТО. В одном фэнтези-романе описывается кролик, который
                    488: умеет ДЕЛАТЬ ЭТО. Что мы заменили словами "ДЕЛАТЬ ЭТО"?
                    489: 
                    490: Ответ:
                    491: Превращаться в цилиндр.
                    492: 
                    493: Зачет:
                    494: Трансформироваться в цилиндр; преобразовываться в цилиндр; становиться
                    495: цилиндром.
                    496: 
                    497: Комментарий:
                    498: В своем романе Джоан Роулинг подшучивает над классическим фокусом.
                    499: Воронка продаж - маркетинговый инструмент, который позволяет увеличить
                    500: количество клиентов, найти ошибки на каждой из стадий и устранить их, а
                    501: также привлечь целевую аудиторию. Чем эффективнее маркетинг - тем ближе
                    502: воронка к цилиндру.
                    503: 
                    504: Источник:
                    505:    1. https://marketer.ua/voronka-prodazh-ne-rabotaet/
                    506:    2. Дж.К. Роулинг. Гарри Поттер и узник Азкабана.
                    507: http://flibusta.is/b/265597/read
                    508: 
                    509: Автор:
                    510: Дмитрий Буценец (Могилёвцы)
                    511: 
                    512: Вопрос 24:
                    513: Персонаж одного рассказа сравнивает психику после нервного срыва с НЕЙ и
                    514: говорит, что даже после восстановления прежней надежности не будет.
                    515: Комментируя свой перфОрманс, Ай ВейвЕй заявил, что ОНА времен династии
                    516: Хань должна заставить общество пересмотреть отношение к искусству и к
                    517: прошлому. Назовите ЕЕ двумя словами.
                    518: 
                    519: Ответ:
                    520: Разбитая ваза.
                    521: 
                    522: Комментарий:
                    523: Фарфоровые вазы являются важным направлением искусства Древнего Китая.
                    524: Современный китайский художник Ай Вейвей известен своими перформансами
                    525: на грани вандализма. Так, художник приобрел, а затем разбил вазу
                    526: династии Хань.
                    527: 
                    528: Источник:
                    529:    1. С. Кинг. Иногда они возвращаются.
                    530: https://books.google.ru/books?id=qeG7AAAAQBAJ&pg=PT22#v=onepage&q&f=false
                    531:    2. http://be-inart.com/post/view/1733
                    532: 
                    533: Автор:
                    534: Дмитрий Буценец (Могилёвцы)
                    535: 
                    536: Вопрос 25:
                    537: (pic: 20170877.jpg)
                    538:    Многие гены выполняют небольшую работу в функционировании
                    539: биохимических путей, хотя в случае мутаций могут быть виновниками
                    540: различных нарушений. Олег Рева называет эти гены представителями
                    541: устаревающей профессии. Какой?
                    542: 
                    543: Ответ:
                    544: Стрелочник.
                    545: 
                    546: Комментарий:
                    547: Рева говорит, что многие гены - всего лишь стрелочники на перекрестках
                    548: биохимических путей, и роль у них сравнительно скромная. Несмотря на то
                    549: что мутации, как правило, вызваны внешними факторами, виноват, как
                    550: известно, стрелочник.
                    551: 
                    552: Источник:
                    553: М. Ридли. Геном (в переводе Олега Ревы).
                    554: http://flibusta.is/b/349905/read
                    555: 
                    556: Автор:
                    557: Дмитрий Буценец (Могилёвцы)
                    558: 
                    559: Вопрос 26:
                    560: Некоторые из противников теории англичанина поддерживали аристотелевское
                    561: учение о "приливе и отливе". Когда в 1661 году Марчелло увидел связь
                    562: между ПЕРВЫМИ и ВТОРЫМИ, остававшиеся споры вокруг теории прекратились.
                    563: Назовите ПЕРВЫЕ и ВТОРЫЕ.
                    564: 
                    565: Ответ:
                    566: Артерии, вены.
                    567: 
                    568: Зачет:
                    569: В любом порядке.
                    570: 
                    571: Комментарий:
                    572: Идеи ГАрвея о кровообращении прошли через шквал критики. Ранее ученые
                    573: придерживались теории ГалЕна, согласно которой кровь образуется в
                    574: печени, а некоторые - и еще более древних теорий. Несмотря на то что
                    575: многие приняли идеи Гарвея еще при его жизни, убедительные
                    576: доказательства появились через четыре года после смерти ученого, когда
                    577: МальпИги, воспользовавшись микроскопом, обнаружил капиллярную сеть,
                    578: соединяющую артерии и вены.
                    579: 
                    580: Источник:
                    581:    1. М. Энгельгардт. Уильям Гарвей. Его жизнь и научная деятельность.
                    582: https://profilib.com/chtenie/58198/mikhail-engelgardt-uilyam-garvey-ego-zhizn-i-nauchnaya-deyatelnost-7.php
                    583:    2. С.Н. Зигуненко. 100 великих тайн медицины.
                    584: http://flibusta.is/b/482131/read
                    585: 
                    586: Автор:
                    587: Дмитрий Буценец (Могилёвцы)
                    588: 
                    589: Вопрос 27:
                    590: Считается, что ЕГО традиционно поддерживают представители среднего и
                    591: высшего классов. Так, одно из ЕГО прозвищ связано с пирожными, которые в
                    592: начале XX века были доступны лишь зажиточным горожанам. Назовите ЕГО.
                    593: 
                    594: Ответ:
                    595: [Футбольный клуб] "Реал Мадрид".
                    596: 
                    597: Комментарий:
                    598: "Реал" в отношении "фанатского слоя" действительно можно назвать
                    599: "королевским" клубом. Прозвище "Сливочные", закрепившееся в русском
                    600: языке, могло вам намекнуть на крем. Впрочем, на самом деле это очень
                    601: вольное переложение испанского "Los Merengues" [лос мерЕнгес]. Меренги,
                    602: они же безе, имеют "сливочный" цвет, чем, видимо, и обусловлена русская
                    603: локализация прозвища.
                    604: 
                    605: Источник:
                    606: https://ru.wikipedia.org/wiki/Мадридское_дерби
                    607: 
                    608: Автор:
                    609: Ренат Рустамов (Минск)
                    610: 
                    611: Вопрос 28:
                    612: (pic: 20170878.jpg)
                    613:    Перед вами фрагмент иллюстрации к одной статье. Ее автор пишет, что
                    614: именно так в XIX веке должен был выглядеть "ОН", чтобы избежать
                    615: скандала. Назовите "ЕГО".
                    616: 
                    617: Ответ:
                    618: "Завтрак на траве".
                    619: 
                    620: Комментарий:
                    621: "Завтрак на траве" - скандально известная картина Эдуарда Мане. Выбор
                    622: сюжета картины - двое полностью одетых мужчин с нагой женщиной на
                    623: природе - вызвал полное непонимание и обвинения в плохом вкусе.
                    624: 
                    625: Источник:
                    626: https://ru.wikipedia.org/wiki/Завтрак_на_траве_(картина_Мане)
                    627: 
                    628: Автор:
                    629: Дмитрий Буценец (Могилёвцы)
                    630: 
                    631: Вопрос 29:
                    632: В одном эссе Сергей Довлатов выделяет следующие последовательные уровни:
                    633: сАбвей, супермаркет, TV [ти-ви], дейли ньюз, чайна таун. Какое слово мы
                    634: пропустили в названии каждого уровня?
                    635: 
                    636: Ответ:
                    637: Инглиш.
                    638: 
                    639: Зачет:
                    640: English; английский.
                    641: 
                    642: Комментарий:
                    643: Сергей Довлатов, уехавший в США по политическим причинам, рассказывает
                    644: об уровнях владения английским языком эмигрантами. Причем самым высшим
                    645: уровнем он называет понимание английского, на котором говорят другие
                    646: эмигранты, в частности китайцы.
                    647: 
                    648: Источник:
                    649: С.Д. Довлатов. Блеск и нищета русской литературы.
                    650: http://flibusta.is/b/129387/read
                    651: 
                    652: Автор:
                    653: Дарья Данилевич (Минск)
                    654: 
                    655: Вопрос 30:
                    656: В свое время массовые репрессии в Латинской Америке были настолько
                    657: обычным делом, что глагол "исчезнуть" стал ТАКИМ. По современным данным
                    658: одной из причин проявлений ТАКОГО ЕГО является активное размножение
                    659: нейронов. Что мы заменили словами "ТАКОЙ ОН"?
                    660: 
                    661: Ответ:
                    662: Переходный возраст.
                    663: 
                    664: Комментарий:
                    665: В испанском языке глагол "исчезнуть" стал переходным, т.е. начали
                    666: говорить "исчезнуть кого-либо" в значении "устранить, уничтожить". Ранее
                    667: ученые считали, что активное размножение нейронов происходит только в
                    668: раннем детском возрасте, однако современные исследования показывают, что
                    669: повторная волна размножения нейронов происходит в подростковом возрасте,
                    670: чем отчасти обусловлено не всегда адекватное поведение в этот период.
                    671: 
                    672: Источник:
                    673:    1. М. Хем. Быть диктатором. Практическое руководство.
                    674: http://flibusta.is/b/457718/read
                    675:    2. https://www.infoniac.ru/news/10-faktov-kotoryh-kazhdyi-roditel-dolzhen-znat-o-mozge-svoego-podrostka.html
                    676: 
                    677: Автор:
                    678: Никита Шевела (Минск), Дмитрий Буценец (Могилёвцы)
                    679: 
                    680: Вопрос 31:
                    681: (pic: 20170879.jpg)
                    682:    Перед вами картина, изображающая игру, популярную на рубеже XVIII-XIX
                    683: веков. Согласно одной версии, поднятые руки девушек символизируют ЕЕ.
                    684: Назовите ЕЕ.
                    685: 
                    686: Ответ:
                    687: Гильотина.
                    688: 
                    689: Комментарий:
                    690: Изображенная игра напоминает игру "ручеек": на последней строчке руки
                    691: опускаются, и тот участник, который оказывается пойман, - выбывает.
                    692: Согласно мрачной теории, игра была полна символизма и отсылала к модному
                    693: в то время способу казни.
                    694: 
                    695: Источник:
                    696:    1. http://academics.hamilton.edu/mediascholarship/internproject/steph.html
                    697:    2. https://en.wikipedia.org/wiki/File:Agnes_Rose_Bouvier00.jpg
                    698: 
                    699: Автор:
                    700: Дмитрий Буценец (Могилёвцы)
                    701: 
                    702: Вопрос 32:
                    703: Описывая выступления Шарля де Голля, один еженедельник писал, что лица
                    704: людей, которые воодушевленно поддерживали генерала, становились такого
                    705: же цвета, как ОН. Чтобы избежать кровотечения в бою, ЕГО, как правило,
                    706: отрезают. Назовите ЕГО двумя словами.
                    707: 
                    708: Ответ:
                    709: Петушиный гребень.
                    710: 
                    711: Зачет:
                    712: Петушиный гребешок; гребень петуха; гребешок петушка.
                    713: 
                    714: Комментарий:
                    715: Бойцовских петухов готовят заранее: так, в возрасте полугода им обычно
                    716: отрезают гребень. Рассказывая о патриотизме и делая отсылку к
                    717: национальному символу Франции, репортер сравнил раскрасневшиеся лица
                    718: людей с петушиным гребнем.
                    719:    z-checkdb: Это ошибка переводчика: в оригинале упоминается цвет мака
                    720: (coquelicot), см.
                    721: https://archive.org/details/LeDeuxiemeSexeTome2SimoneDeBeauvoir/page/n667
                    722: (Евгений Рубашкин).
                    723: 
                    724: Источник:
                    725:    1. https://ru.wikipedia.org/wiki/Петушиные_бои
                    726:    2. http://allrefrs.ru/2-20416.html
                    727: 
                    728: Автор:
                    729: Дмитрий Буценец (Могилёвцы)
                    730: 
                    731: Вопрос 33:
                    732: Для изготовления особенно прочных механизмов нацистская Германия в
                    733: большом количестве покупала редкое сырье. Сэм Кин образно замечает, что
                    734: поставки сырья удовлетворяли ЕГО немецкой промышленности. Назовите ЕГО
                    735: устойчивым выражением.
                    736: 
                    737: Ответ:
                    738: Волчий аппетит.
                    739: 
                    740: Зачет:
                    741: Волчий голод.
                    742: 
                    743: Комментарий:
                    744: Речь идет о химическом элементе - вольфраме. Как известно, сплавы
                    745: вольфрама обладают высокой тугоплавкостью и прочностью. Слово "вольфрам"
                    746: можно перевести с немецкого языка как "волчья пена". Кстати, Салазар,
                    747: который продавал Германии вольфрам, нажил на нем немалое богатство.
                    748: 
                    749: Источник:
                    750: С. Кин. Исчезающая ложка, или Удивительные истории из жизни
                    751: периодической таблицы Менделеева. http://flibusta.is/b/429981/read
                    752: 
                    753: Автор:
                    754: Дмитрий Буценец (Могилёвцы)
                    755: 
                    756: Вопрос 34:
                    757: [Ведущему: во втором предложении сделать паузу между словами "ТАКОГО" и
                    758: "ОНО".]
                    759:    В фильме о тяжелой жизни рыбаков говорится, что ОНО ТАКОЕ. У ТАКОГО
                    760: ОНО ловит стрелы молний. Что мы заменили словами "ОНО ТАКОЕ"?
                    761: 
                    762: Ответ:
                    763: Море горькое.
                    764: 
                    765: Комментарий:
                    766: Говоря об изматывающем труде рыбаков и их частой гибели, героиня
                    767: печально размышляет, что море горькое. В "Песне о Буревестнике" Горького
                    768: море "ловит и гасит" молнии.
                    769: 
                    770: Источник:
                    771:    1. Х/ф "Земля дрожит" (1948), реж. Лукино Висконти.
                    772: http://cinematext.ru/movie/zemlja-drozhit-la-terra-trema-episodio-del-mare-1948/
                    773:    2. https://murzim.ru/nauka/himiya/21084-soli-v-prirodnoy-vode.html
                    774:    3. http://gorkiy-lit.ru/gorkiy/proza/rasskaz/pesnya-o-burevestnike.htm
                    775: 
                    776: Автор:
                    777: Дмитрий Буценец (Могилёвцы)
                    778: 
                    779: Вопрос 35:
                    780: [Ведущему: четко произнести букву "д" в слове "Лад".]
                    781:    В переводе одного романа описывается высокотехнологичный в прошлом
                    782: город Лад, разрушенный его жителями. После разъяснения автора в более
                    783: поздних переводах название города немного изменили. На какое?
                    784: 
                    785: Ответ:
                    786: Луд.
                    787: 
                    788: Зачет:
                    789: Лудд.
                    790: 
                    791: Комментарий:
                    792: Стивен Кинг рассказал, что название города произошло от названия
                    793: противников промышленной революции - луддитов. Луддиты считали своим
                    794: предводителем некоего Неда Лудда, которому приписывалось уничтожение
                    795: двух чулочных станков. Подобно луддитам жители из произведения Кинга
                    796: разрушали свой город.
                    797: 
                    798: Источник:
                    799: https://ru.wikipedia.org/wiki/Бесплодные_земли
                    800: 
                    801: Автор:
                    802: Дмитрий Буценец (Могилёвцы)
                    803: 
                    804: Вопрос 36:
                    805:    <раздатка>
                    806:    Новое кино
                    807:    </раздатка>
                    808:    На розданном вам материале часть цитаты главной фигуры движения
                    809: "Новое кино" ГлАубера РОши. Какими двумя короткими словами она
                    810: заканчивается?
                    811: 
                    812: Ответ:
                    813: Это я.
                    814: 
                    815: Комментарий:
                    816: Легендарная фраза "Государство - это я" приписывается Людовику XIV.
                    817: 
                    818: Источник:
                    819: https://en.wikipedia.org/wiki/Glauber_Rocha
                    820: 
                    821: Автор:
                    822: Ренат Рустамов (Минск)
                    823: 
                    824: Тур:
                    825: 2 этап
                    826: 
                    827: Дата:
                    828: 10-Nov-2017
                    829: 
                    830: Редактор:
                    831: 1-18 - Евгений Лешкович (Минск); 19-36 - Валерий Семёнов (Минск -
                    832: Могилев)
                    833: 
                    834: Инфо:
                    835: Евгений Лешкович благодарит за тестирование и помощь в подготовке пакета
                    836: Андрея Танану, Анну Якушевич, Вадима Кузмича, Даниила Шункевича,
                    837: Александра Шустера, Валерия Семёнова, Дмитрия Буценца, Анастасию
                    838: Балмакову, Василия Бобкова, Михаила Карпука, Александру Ермалович, Елену
                    839: Ваксман-Атрохову, Дарью Соловей, Руслана Огородника, Екатерину Лагуту,
                    840: Андрея Кравченко, Игоря Тюнькина, Никиту Лопуха, Сергея Очтова, а также
                    841: команды "И тебя тоже" и "Жидкость для розжига". Валерий Семёнов
                    842: благодарит за помощь в работе над пакетом всех авторов вопросов, а также
                    843: команды "И тебя тоже", "Жидкость для розжига" и "Одушевленные аэросани"
                    844: за тестирование и ценные замечания.
                    845: 
                    846: Вопрос 1:
                    847: Говоря об ИКСЕ своего друга, герой романа Ирвина Уэлша использует
                    848: выражения "паук брюхом" и "скорпион хвостом". Какие два слова,
                    849: начинающиеся на одну и ту же букву, мы заменили ИКСОМ?
                    850: 
                    851: Ответ:
                    852: Плохой почерк.
                    853: 
                    854: Комментарий:
                    855: Обычно такой почерк характеризуют как "курица лапой", но в данном случае
                    856: всё так плохо, что герой решил поискать другие сравнения.
                    857:    Игровое жюри надеется, что ему их искать не придется.
                    858: 
                    859: Источник:
                    860: И. Уэлш. Порно. http://flibusta.is/b/77708/read
                    861: 
                    862: Автор:
                    863: Евгений Лешкович (Минск)
                    864: 
                    865: Вопрос 2:
                    866: По мнению МИкала Хема, у африканских диктаторов есть одно отличие от
                    867: остальных - они не любят покупать роскошные автомобили. В качестве
                    868: причины Хем называет ИХ. В известном выражении ОНИ упоминаются в паре...
                    869: С кем?
                    870: 
                    871: Ответ:
                    872: С дураками.
                    873: 
                    874: Комментарий:
                    875: Африканские диктаторы не покупают дорогих автомобилей, потому что в их
                    876: странах плохие дороги и кататься всё равно негде. Дураки и дороги - две
                    877: вечные проблемы России.
                    878: 
                    879: Источник:
                    880: М. Хем. Быть диктатором. Практическое руководство.
                    881: http://flibusta.is/b/457718/read
                    882: 
                    883: Автор:
                    884: Евгений Лешкович (Минск)
                    885: 
                    886: Вопрос 3:
                    887: (pic: 20170880.jpg)
                    888:    Перед вами дом, в названии которого есть ОНА. Где ОНА находится в
                    889: заглавии произведения второй половины XX века?
                    890: 
                    891: Ответ:
                    892: На склоне.
                    893: 
                    894: Комментарий:
                    895: ОНА - улитка. Речь в вопросе идет о произведении Аркадия и Бориса
                    896: Стругацких.
                    897: 
                    898: Источник:
                    899:    1. https://novate.ru/blogs/071109/13373/
                    900:    2. https://ru.wikipedia.org/wiki/Улитка_на_склоне
                    901: 
                    902: Автор:
                    903: Вадим Кузмич (Логойск)
                    904: 
                    905: Вопрос 4:
                    906: Историк Сергей Иванов не любит вопросов о будущем. По мнению Иванова,
                    907: задавать историку такие вопросы - то же самое, что спрашивать ИКСА о
                    908: бессмертии души. ИКСОМ можно назвать героя произведения 1632 года.
                    909: Назовите автора этого произведения.
                    910: 
                    911: Ответ:
                    912: Рембрандт [Харменс ван Рейн].
                    913: 
                    914: Комментарий:
                    915: По мнению Сергея Иванова, спросить историка о будущем - то же самое, что
                    916: спросить патологоанатома о бессмертии души. Заглавный герой картины
                    917: Рембрандта "Урок анатомии доктора Тульпа" был в том числе и
                    918: патологоанатомом.
                    919: 
                    920: Источник:
                    921:    1. https://arzamas.academy/materials/191
                    922:    2. https://ru.wikipedia.org/wiki/Урок_анатомии_доктора_Тульпа
                    923: 
                    924: Автор:
                    925: Евгений Лешкович (Минск)
                    926: 
                    927: Вопрос 5:
                    928: Герой американского мультсериала говорит, что смотрит в будущее, и носит
                    929: значок с надписью "Иван 2028". Какие две буквы мы пропустили?
                    930: 
                    931: Ответ:
                    932: ка.
                    933: 
                    934: Комментарий:
                    935: Полностью надпись на значке выглядит как "Иванка 2028". Видимо, персонаж
                    936: не сомневается, что одним из кандидатов в президенты США в 2028 году
                    937: будет Иванка Трамп. Это хорошо впишется в американский политический
                    938: тренд, заданный семействами Бушей и Клинтонов.
                    939: 
                    940: Источник:
                    941: https://pikabu.ru/story/simpsonyi_prodolzhayut_prorochit_4616031
                    942: 
                    943: Автор:
                    944: Андрей Танана, Анна Якушевич (Минск)
                    945: 
                    946: Вопрос 6:
                    947: После второго срока Джордж Вашингтон поселился в поместье МАунт-ВЕрнон.
                    948: Вскоре он распорядился, чтобы вокруг поместья были установлены
                    949: неправильные указатели. Современный журналист, рассказывающий об этой
                    950: истории, упоминает слово с удвоенной согласной. Какое?
                    951: 
                    952: Ответ:
                    953: Папарацци.
                    954: 
                    955: Комментарий:
                    956: Вашингтона очень быстро утомили назойливые журналисты. Пришлось сделать
                    957: так, чтобы случайный человек никак не мог добраться до Маунт-Вернона.
                    958: 
                    959: Источник:
                    960: "Дилетант", 2017, N 1. - С. 21.
                    961: 
                    962: Автор:
                    963: Евгений Лешкович (Минск)
                    964: 
                    965: Вопрос 7:
                    966: В стихотворении "Танки идут по Праге" скрепки советских чиновников
                    967: превращаются в НИХ. Стадия ЕЕ может длиться от нескольких недель до
                    968: нескольких лет. Назовите ЕЕ.
                    969: 
                    970: Ответ:
                    971: Гусеница.
                    972: 
                    973: Комментарий:
                    974:    "Танки идут по склепам,
                    975:    По тем, что еще не родились.
                    976:    Четки чиновничьих скрепок
                    977:    В гусеницы превратились".
                    978: 
                    979: Источник:
                    980:    1. http://www.ruthenia.ru/60s/evtushenko/tanki.htm
                    981:    2. https://ru.wikipedia.org/wiki/Гусеница
                    982: 
                    983: Автор:
                    984: Максим Осмоловский (Минск)
                    985: 
                    986: Вопрос 8:
                    987: У Аристофана рассказывается о встрече Эсхила и Еврипида. Упоминая об
                    988: этом, Михаил ШвыдкОй шутит, что ИХ история насчитывает уже двадцать пять
                    989: веков. Назовите ИХ словом, пишущимся через дефис.
                    990: 
                    991: Ответ:
                    992: Рэп-баттлы.
                    993: 
                    994: Комментарий:
                    995: Эсхил и Еврипид в пьесе "Лягушки" устроили жесткий поэтический поединок,
                    996: напомнивший Швыдкому современные рэп-баттлы. Кстати, победил Эсхил.
                    997: 
                    998: Источник:
                    999: https://meduza.io/shapito/2017/09/17/rep-battly-imeyut-davnyuyu-istoriyu-esli-otkryt-aristofana-mihail-shvydkoy-pozval-gnoynogo-na-telekanal-kultura
                   1000: 
                   1001: Автор:
                   1002: Евгений Лешкович (Минск)
                   1003: 
                   1004: Вопрос 9:
                   1005: Джеймс УиттАкер осуждает тестировщиков, склонных преувеличивать важность
                   1006: каждого дефекта. Дальше Уиттакер проводит параллель с известной
                   1007: историей, заменяя словом "баг" другое слово. Назовите это слово.
                   1008: 
                   1009: Ответ:
                   1010: Волк.
                   1011: 
                   1012: Комментарий:
                   1013: К тестировщику-паникеру со временем перестают прислушиваться. Уиттакер
                   1014: сравнивает его с мальчиком, который кричал "Волки!".
                   1015: 
                   1016: Источник:
                   1017: Дж. Уиттакер, Дж. Арбон, Дж. Кароло. Как тестируют в Google.
                   1018: http://flibusta.is/b/382371/read
                   1019: 
                   1020: Автор:
                   1021: Евгений Лешкович (Минск)
                   1022: 
                   1023: Вопрос 10:
                   1024: Историк Маргарет Макмиллан пишет, что в начале XX века отношения
                   1025: Великобритании с Соединенными Штатами тоже стали радушными. Какое слово
                   1026: мы заменили в этом вопросе?
                   1027: 
                   1028: Ответ:
                   1029: Сердечными.
                   1030: 
                   1031: Комментарий:
                   1032: В это же время Великобритания вошла в состав Антанты. "Entente cordiale"
                   1033: [антАнт кордиАль] означает "сердечное согласие". Слова "радушный" и
                   1034: "сердечный" являются синонимами.
                   1035: 
                   1036: Источник:
                   1037: М. Макмиллан. Война, которая покончила с миром. Кто и почему развязал
                   1038: Первую мировую. http://flibusta.is/b/452488/read
                   1039: 
                   1040: Автор:
                   1041: Евгений Лешкович (Минск)
                   1042: 
                   1043: Вопрос 11:
                   1044: [Ведущему: как можно четче прочитать фамилию Квонг!]
                   1045:    Выступавший в середине тридцатых хоккеист Ларри Квонг отличался
                   1046: развитой мускулатурой и агрессивным стилем игры. Партнеры дали ему
                   1047: прозвище, лишь одной буквой отличающееся от названия произведения.
                   1048: Назовите это произведение.
                   1049: 
                   1050: Ответ:
                   1051: "Кинг-Конг".
                   1052: 
                   1053: Комментарий:
                   1054: Мощный и агрессивный хоккеист удостоился прозвища "Кинг-Квонг". Фильм
                   1055: "Кинг-Конг" как раз вышел в середине тридцатых.
                   1056: 
                   1057: Источник:
                   1058: https://www.sports.ru/tribuna/blogs/antarktida/1409410.html
                   1059: 
                   1060: Автор:
                   1061: Евгений Лешкович (Минск)
                   1062: 
                   1063: Вопрос 12:
                   1064: Главного героя анимационного фильма зовут Джек Фрост. Однажды его сестра
                   1065: провалилась под лед, но Джек спас ее ценой собственной жизни. Кто должен
                   1066: был озвучивать Джека по первоначальному плану режиссера?
                   1067: 
                   1068: Ответ:
                   1069: Леонардо ди Каприо.
                   1070: 
                   1071: Комментарий:
                   1072: История со спасением сестры сильно перекликается с сюжетом "Титаника",
                   1073: главную роль в котором сыграл Ди Каприо. Кстати, там его персонажа тоже
                   1074: зовут Джек.
                   1075: 
                   1076: Источник:
                   1077:    1. https://ru.wikipedia.org/wiki/Хранители_снов
                   1078:    2. http://www.kinobusiness.com/movies/Rise-of-the-Guardians/
                   1079: 
                   1080: Автор:
                   1081: Анна Якушевич (Минск)
                   1082: 
                   1083: Вопрос 13:
                   1084: Джованни Баттиста ГрАсси был одним из крупнейших исследователей малярии.
                   1085: Автор книги о Грасси назвал ученого ИКСОМ ИГРЕКОВ. Одна из рецензий на
                   1086: "ИКСА АЛЬФ" называется "Детский апокалипсис". Какое слово мы заменили
                   1087: АЛЬФОЙ?
                   1088: 
                   1089: Ответ:
                   1090: Муха.
                   1091: 
                   1092: Комментарий:
                   1093: Переносчиками малярии являются комары. Грасси изучил их настолько
                   1094: хорошо, что его прозвали "повелителем комаров". Книга "Повелитель мух"
                   1095: повествует о локальном "детском апокалипсисе".
                   1096: 
                   1097: Источник:
                   1098:    1. П. де Крюи. Охотники за микробами.
                   1099: http://flibusta.is/b/511064/read
                   1100:    2. https://www.kinopoisk.ru/user/737002/comment/1561680/
                   1101: 
                   1102: Автор:
                   1103: Андрей Танана (Минск)
                   1104: 
                   1105: Вопрос 14:
                   1106: Какао-бобы содержат вещества, способные увеличить склонность человека к
                   1107: насилию. В какой стране было изобретено коншИрование - методика,
                   1108: благодаря которой из какао-бобов удаляется до 80% таких веществ?
                   1109: 
                   1110: Ответ:
                   1111: Швейцария.
                   1112: 
                   1113: Комментарий:
                   1114: Чистые какао-бобы провоцируют нездоровую агрессию. Однако благодаря
                   1115: коншированию они становятся абсолютно безобидными. Видимо, неспроста эта
                   1116: методика была изобретена в стране, известной миролюбием и нейтралитетом!
                   1117: 
                   1118: Источник:
                   1119: Т. Нилон. Битвы за еду и войны культур: тайные двигатели истории.
                   1120: http://flibusta.is/b/482946/read
                   1121: 
                   1122: Автор:
                   1123: Евгений Лешкович (Минск)
                   1124: 
                   1125: Вопрос 15:
                   1126: В 1668 году во Франции началась эпидемия чумы. Население некоторых
                   1127: городов сократилось наполовину, однако Париж при этом не пострадал.
                   1128: Объясняя этот факт, Том Нилон пишет, что двумя годами ранее в Париже
                   1129: началась мода на ад. Какие пять букв мы пропустили в вопросе?
                   1130: 
                   1131: Ответ:
                   1132: лимон.
                   1133: 
                   1134: Комментарий:
                   1135: По мнению Нилона, парижан от чумы спасла любовь к лимонаду. Цедра лимона
                   1136: содержит вещества, убивающие переносчиков чумы. Цедру выбрасывали на
                   1137: свалки, ее ели крысы, и это помогало естественным образом
                   1138: продезинфицировать город.
                   1139: 
                   1140: Источник:
                   1141: Т. Нилон. Битвы за еду и войны культур: тайные двигатели истории.
                   1142: http://flibusta.is/b/482946.
                   1143: 
                   1144: Автор:
                   1145: Евгений Лешкович (Минск)
                   1146: 
                   1147: Вопрос 16:
                   1148: Корабль "Дредноут" отличался необычайной скоростью и нес на борту десять
                   1149: 12-дюймовых орудий. Рассказывая о "Дредноуте", современный историк
                   1150: использует фразу, которую приписывают уроженцу Луисвилля. Назовите этого
                   1151: уроженца.
                   1152: 
                   1153: Ответ:
                   1154: Мохаммед Али.
                   1155: 
                   1156: Зачет:
                   1157: Кассиус Клей.
                   1158: 
                   1159: Комментарий:
                   1160: Корабль был быстрым и мощным, т.е. мог "порхать, как бабочка, и жалить,
                   1161: как пчела". Эту фразу приписывают знаменитому боксеру Мохаммеду Али,
                   1162: который родился в Луисвилле, штат Кентукки.
                   1163: 
                   1164: Источник:
                   1165:    1. М. Макмиллан. Война, которая покончила с миром. Кто и почему
                   1166: развязал Первую мировую. http://flibusta.is/b/452488/read
                   1167:    2. http://mirfactov.com/10-mudrostey-mohammeda-ali/
                   1168: 
                   1169: Автор:
                   1170: Евгений Лешкович (Минск)
                   1171: 
                   1172: Вопрос 17:
                   1173: Емельян Пугачёв обращался к народу от имени императора и обещал отменить
                   1174: крепостное право. Чтобы остановить самозванца, Екатерина II объявила,
                   1175: что силу закона на территории России теперь будут иметь только ТАКИЕ
                   1176: документы. Кто создал первый ТАКОЙ документ в Пфальце?
                   1177: 
                   1178: Ответ:
                   1179: Иоганн Гутенберг.
                   1180: 
                   1181: Комментарий:
                   1182: Печатного станка у пугачёвцев не было, поэтому приказ Екатерины II
                   1183: считать законными только напечатанные манифесты лишил их способа
                   1184: воздействовать на народ. Изобретатель книгопечатания Иоганн Гутенберг
                   1185: проживал в княжестве Пфальц. Редактор посвящает вопрос 500-летию
                   1186: белорусского книгопечатания.
                   1187: 
                   1188: Источник:
                   1189:    1. А.В. Иванов. Вилы.
                   1190: https://books.google.ru/books?id=TTlCDQAAQBAJ&pg=PT79#v=onepage&q&f=false
                   1191:    2. https://ru.wikipedia.org/wiki/Гутенберг,_Иоганн
                   1192:    3. https://ru.wikipedia.org/wiki/Майнц
                   1193: 
                   1194: Автор:
                   1195: Евгений Лешкович (Минск)
                   1196: 
                   1197: Вопрос 18:
                   1198: Полковник Дэвид Хэкворт называет себя счастливчиком. В статье Википедии
                   1199: о Хэкворте упоминаются восемь сердец. Какое слово мы пропустили?
                   1200: 
                   1201: Ответ:
                   1202: Пурпурных.
                   1203: 
                   1204: Комментарий:
                   1205: Медаль "Пурпурное сердце" вручается военнослужащим, получившим ранения.
                   1206: Хэкворт был ранен восемь раз, но всё же остался жив - настоящий
                   1207: счастливчик!
                   1208: 
                   1209: Источник:
                   1210: https://ru.wikipedia.org/wiki/Хэкворт,_Дэвид_Хаскелл
                   1211: 
                   1212: Автор:
                   1213: Евгений Лешкович (Минск)
                   1214: 
                   1215: Вопрос 19:
                   1216: Герой Диккенса называет недорогие лондонские трактиры географическими,
                   1217: так как каждая АЛЬФА напоминает ему карту мира с указанием морских
                   1218: течений. Необычная разновидность АЛЬФЫ описана в фольклоре. Назовите
                   1219: АЛЬФУ одним словом.
                   1220: 
                   1221: Ответ:
                   1222: Скатерть.
                   1223: 
                   1224: Комментарий:
                   1225: На скатерти в недорогом трактире - следы от кружек и пищи.
                   1226: 
                   1227: Источник:
                   1228: Ч. Диккенс. Большие надежды. http://flibusta.is/b/301012/read
                   1229: 
                   1230: Автор:
                   1231: Ольга Ярошенко (Могилев)
                   1232: 
                   1233: Вопрос 20:
                   1234: (pic: 20170881.jpg)
                   1235:    Перед вами автопортрет Доменико ДурАнте, туринского художника первой
                   1236: половины XX века. За свою карьеру он получил множество наград, а в 1905
                   1237: году даже стал победителем... Чего?
                   1238: 
                   1239: Ответ:
                   1240: Чемпионата Италии по футболу.
                   1241: 
                   1242: Комментарий:
                   1243: Дуранте совмещал две профессии и успел провести 30 игр за туринский
                   1244: "Ювентус", в футболке которого он себя и изобразил.
                   1245: 
                   1246: Источник:
                   1247: https://ru.wikipedia.org/wiki/Дуранте,_Доменико
                   1248: 
                   1249: Автор:
                   1250: Максим Корнеевец (Минск)
                   1251: 
                   1252: Вопрос 21:
                   1253: Художник Владимир Маковский создал свою картину в конце XIX века. Более
                   1254: известная картина на тот же сюжет была создана тридцатью с лишним годами
                   1255: ранее, хотя разницу выдает только фасон платья. Назовите автора более
                   1256: известной картины.
                   1257: 
                   1258: Ответ:
                   1259: [Василий] Пукирев.
                   1260: 
                   1261: Комментарий:
                   1262: На картине Пукирева "Неравный брак", написанной в 1862 году, у невесты
                   1263: большой громоздкий кринолин, а на картине "К венцу" 1894 года у невесты
                   1264: юбка значительно сужена. Гораздо более выражена разница в возрасте
                   1265: героев обеих картин.
                   1266: 
                   1267: Источник:
                   1268: http://www.liveinternet.ru/users/4231626/post421512623/
                   1269: 
                   1270: Автор:
                   1271: Юлия Ткачёва (Минск)
                   1272: 
                   1273: Вопрос 22:
                   1274: Героиня фантастического рассказа везет в Африку новый вирус, который
                   1275: позволяет любому человеку осуществлять процесс фотосинтеза. При этом она
                   1276: говорит, что у нее в сумке находится смерть одного из ИКСОВ. Назовите
                   1277: ИКСОВ двумя словами.
                   1278: 
                   1279: Ответ:
                   1280: Всадники Апокалипсиса.
                   1281: 
                   1282: Комментарий:
                   1283: Человек, умеющий осуществлять процесс фотосинтеза, не зависит от
                   1284: источников питания, поэтому Голод ему не страшен.
                   1285: 
                   1286: Источник:
                   1287: У.Й. Уильямс. Зеленая Леопардовая Чума. http://flibusta.is/b/119581/read
                   1288: 
                   1289: Автор:
                   1290: Ольга Ярошенко (Могилев)
                   1291: 
                   1292: Вопрос 23:
                   1293: Чтобы продолжать наступление в битве под Москвой, немцы зачастую
                   1294: разводили костры под НИМИ. Назовите ИХ двумя словами, начинающимися на
                   1295: парные согласные.
                   1296: 
                   1297: Ответ:
                   1298: Двигатели танков.
                   1299: 
                   1300: Зачет:
                   1301: Дно танков; днища танков.
                   1302: 
                   1303: Комментарий:
                   1304: Таким образом приходилось отогревать замерзшее топливо.
                   1305: 
                   1306: Источник:
                   1307: М. Хейстингс. Вторая мировая война: Ад на земле.
                   1308: http://flibusta.is/b/389254/read
                   1309: 
                   1310: Автор:
                   1311: Валерий Семёнов (Минск)
                   1312: 
                   1313: Вопрос 24:
                   1314: В 1926 году в московском Клубе радиолюбителей состоялся концерт,
                   1315: вызвавший большой интерес не только у радиотехников, но и у
                   1316: профессиональных музыкантов. Кто давал этот концерт?
                   1317: 
                   1318: Ответ:
                   1319: [Лев] Термен.
                   1320: 
                   1321: Комментарий:
                   1322: Инженер играл на придуманном им терменвоксе. Радиотехников интересовал
                   1323: принцип работы, а музыкантов - новый инструмент.
                   1324: 
                   1325: Источник:
                   1326: Б.Е. Черток. Ракеты и люди. Фили-Подлипки-Тюратам.
                   1327: http://flibusta.is/b/82011/read
                   1328: 
                   1329: Автор:
                   1330: Валерий Семёнов (Минск)
                   1331: 
                   1332: Вопрос 25:
                   1333: Корней Чуковский рассказывает, что в 1924 году из помещения одной
                   1334: биостанции стали систематически пропадать банки с биоматериалами.
                   1335: Назовите основную причину пропаж двумя словами, начинающимися на парные
                   1336: согласные.
                   1337: 
                   1338: Ответ:
                   1339: Сухой закон.
                   1340: 
                   1341: Комментарий:
                   1342: Земноводные были заспиртованными, и пропадали даже несмотря на то, что
                   1343: спирт был с формалином.
                   1344: 
                   1345: Источник:
                   1346: Н.Б. Лебина. Советская повседневность: нормы и аномалии. От военного
                   1347: коммунизма к большому стилю. http://flibusta.is/b/415924/read
                   1348: 
                   1349: Автор:
                   1350: Валерий Семёнов (Минск)
                   1351: 
                   1352: Вопрос 26:
                   1353: В 20-х годах XIX века мастер Рьессек создал первый ИКС, добавив к
                   1354: привычному устройству чернильную иглу. ИКС - это жанр средневековой
                   1355: литературы, содержащий попытки систематизации истории мира. Какое слово
                   1356: греческого происхождения мы заменили ИКСОМ?
                   1357: 
                   1358: Ответ:
                   1359: Хронограф.
                   1360: 
                   1361: Комментарий:
                   1362: Механизм Рьессека фиксировал измеренный промежуток времени с помощью
                   1363: чернильной иглы, ставившей точку на циферблате. В хронографах отмечаются
                   1364: одни из первых попыток ведения исторических летописей в хронологическом
                   1365: порядке.
                   1366: 
                   1367: Источник:
                   1368:    1. http://mywatch.ru/watch-art/art_1644.html
                   1369:    2. https://ru.wikipedia.org/wiki/Хронограф
                   1370:    3. https://ru.wikipedia.org/wiki/Хронограф_(книга)
                   1371: 
                   1372: Автор:
                   1373: Андрей Цмыг (Минск)
                   1374: 
                   1375: Вопрос 27:
                   1376: Герои современного рассказа спасаются от грозы во флорентийской базилике
                   1377: ди Санта-Кроче, однако им кажется, что молнии оказались и там. Назовите
                   1378: причину этого сложным словом.
                   1379: 
                   1380: Ответ:
                   1381: Фотовспышки.
                   1382: 
                   1383: Зачет:
                   1384: Фотоаппарат; фотограф.
                   1385: 
                   1386: Комментарий:
                   1387: Базилика является популярным туристическим местом уже хотя бы в силу
                   1388: того, что там находится усыпальница Галилея.
                   1389: 
                   1390: Источник:
                   1391: У.Й. Уильямс. Зеленая Леопардовая Чума. http://flibusta.is/b/119581/read
                   1392: 
                   1393: Автор:
                   1394: Ольга Ярошенко (Могилев)
                   1395: 
                   1396: Вопрос 28:
                   1397: В этом вопросе ИКС заменяет одно слово.
                   1398:    Цитата из Жюля Верна: "Взгляните-ка на ИКС, <...> разве это не живое
                   1399: существо? Порой гневное, порой нежное! Ночью он спал, как и мы, и вот
                   1400: просыпается в добром расположении духа после покойного сна!". Другой
                   1401: фантаст говорил, что ИКС из его романа должен выглядеть как лавовая
                   1402: лампа. Назовите этого другого фантаста.
                   1403: 
                   1404: Ответ:
                   1405: [Станислав] Лем.
                   1406: 
                   1407: Комментарий:
                   1408: ИКС - океан. Жюль Верн много писал на морскую тематику. Приведенная
                   1409: цитата - из романа "Двадцать тысяч лье под водой". Океан из романа Лема
                   1410: "Солярис" и впрямь был живым.
                   1411: 
                   1412: Источник:
                   1413:    1. Ж. Верн. Двадцать тысяч лье под водой.
                   1414: http://flibusta.is/b/399777/read
                   1415:    2. https://pl.wikipedia.org/wiki/Solaris_(powie%C5%9B%C4%87)
                   1416: 
                   1417: Автор:
                   1418: Павел Дернович (Минск)
                   1419: 
                   1420: Вопрос 29:
                   1421: Внимание, в вопросе есть замена.
                   1422:    В боевике под названием "ГАММА" главный герой наносит ракетный удар
                   1423: по вулкану. Неграмотный школьник описал ГАММУ как разновидность японских
                   1424: мультиков. Что мы заменили ГАММОЙ?
                   1425: 
                   1426: Ответ:
                   1427: Магма.
                   1428: 
                   1429: Комментарий:
                   1430: По сюжету фильма "Магма" главный герой спасает город от судьбы Помпеев,
                   1431: запечатав жерло вулкана ударом ракеты. Школьник перепутал слова "магма"
                   1432: и "манга". "Гамма" и "магма" - анаграммы, чем и обусловлена замена.
                   1433: 
                   1434: Источник:
                   1435:    1. https://www.kinopoisk.ru/film/magma-2005-353592/
                   1436:    2. "Наука и жизнь", 2016, N 9. - С. 89.
                   1437: 
                   1438: Автор:
                   1439: Евгений Гацак (Минск)
                   1440: 
                   1441: Вопрос 30:
                   1442: Легкодоступный набор рукописных изображений цифр MNIST часто
                   1443: используется для проведения экспериментов в сфере машинного обучения.
                   1444: Джеффри Хинтон назвал MNIST ЕЮ машинного обучения. Назовите ЕЕ словом
                   1445: греческого происхождения.
                   1446: 
                   1447: Ответ:
                   1448: Дрозофила.
                   1449: 
                   1450: Комментарий:
                   1451: Благодаря простоте, удобству использования и доступности дрозофила уже
                   1452: много лет является одним из самых популярных объектов экспериментов в
                   1453: биологии.
                   1454: 
                   1455: Источник:
                   1456: http://www.deeplearningbook.org/contents/intro.html
                   1457: 
                   1458: Автор:
                   1459: Даниил Мальцев (Минск)
                   1460: 
                   1461: Вопрос 31:
                   1462: (pic: 20170882.jpg)
                   1463:    Перед вами бокалы для использования ТАМ. Кто упоминается ТАМ в
                   1464: названии романа 2002 года?
                   1465: 
                   1466: Ответ:
                   1467: Кафка.
                   1468: 
                   1469: Комментарий:
                   1470: Заостренные ножки удобно втыкать в песок. "Кафка на пляже" - роман
                   1471: Харуки Мураками.
                   1472: 
                   1473: Источник:
                   1474:    1. http://www.etoday.ru/2016/07/bokaly-dlya-vina-dlya-romatycheskih-svidaniy-na-plyazhe.php
                   1475:    2. https://ru.wikipedia.org/wiki/Кафка_на_пляже
                   1476: 
                   1477: Автор:
                   1478: Максим Корнеевец (Минск)
                   1479: 
                   1480: Вопрос 32:
                   1481: В первых вариантах известной игры игрокам предлагалось пользоваться
                   1482: подручными предметами, поэтому в современном классическом наборе есть
                   1483: ОН. Назовите ЕГО.
                   1484: 
                   1485: Ответ:
                   1486: Наперсток.
                   1487: 
                   1488: Комментарий:
                   1489: В первых вариантах монополии не было фишек, и игроки находили подручные
                   1490: предметы. Наперсток очень удобен в качестве фишки, поэтому он есть в
                   1491: списке традиционных фишек для игры в монополию. В классическом наборе
                   1492: "Монополии" используются фишки в виде ретроавтомобиля, шляпы, тачки,
                   1493: пса, корабля, ботинка, кошки и наперстка.
                   1494: 
                   1495: Источник:
                   1496:    1. https://republic.ru/posts/83164
                   1497:    2. https://lenta.ru/news/2017/02/17/monoliya/
                   1498: 
                   1499: Автор:
                   1500: Валерий Семёнов (Минск)
                   1501: 
                   1502: Вопрос 33:
                   1503: В 1765 году российскому историку Герарду Миллеру, скопившему широчайшую
                   1504: коллекцию рукописей, выделили из казны огромную сумму в шесть тысяч
                   1505: рублей на покупку ИКСА в Москве. Обладателем ИКСА был предусмотрительный
                   1506: герой известного произведения. Назовите ИКС двумя словами.
                   1507: 
                   1508: Ответ:
                   1509: Каменный дом.
                   1510: 
                   1511: Комментарий:
                   1512: Пожары в тогда еще деревянной Москве были частым делом. Было
                   1513: недопустимо, чтобы сгорели драгоценные источники русской истории.
                   1514: 
                   1515: Источник:
                   1516: Артем Ефимов. С чего мы взяли. Три века попыток понять Россию умом. -
                   1517: М.: Индивидуум паблишинг, 2017.
                   1518: 
                   1519: Автор:
                   1520: Валерий Семёнов (Минск)
                   1521: 
                   1522: Вопрос 34:
                   1523: В пещере Атапуэрке были найдены несколько скелетов гейдельбергского
                   1524: человека. Правда, кости были раздроблены и сильно перемешаны. Тем не
                   1525: менее, Станислав Дробышевский с оптимизмом говорит, что испанские
                   1526: антропологи - это не ПЕРВАЯ и не ВТОРАЯ. Назовите ПЕРВУЮ и ВТОРУЮ двумя
                   1527: словами, первое из которых одинаковое.
                   1528: 
                   1529: Ответ:
                   1530: Королевская конница и королевская рать.
                   1531: 
                   1532: Комментарий:
                   1533: Ученые всё же смогли эти скелеты собрать.
                   1534: 
                   1535: Источник:
                   1536: С.В. Дробышевский. Достающее звено. Книга вторая: Люди.
                   1537: http://flibusta.is/b/491954/read
                   1538: 
                   1539: Автор:
                   1540: Валерий Семёнов (Минск)
                   1541: 
                   1542: Вопрос 35:
                   1543: (pic: 20170883.jpg)
                   1544:    Перед вами фотография, сделанная жившим на рубеже XIX и XX веков
                   1545: изобретателем Этьеном-Жюлем Маре. Одна из его книг была в доме епископа.
                   1546: Назовите фамилию этого епископа.
                   1547: 
                   1548: Ответ:
                   1549: Райт.
                   1550: 
                   1551: Комментарий:
                   1552: В книге Маре "Animal Mechanism" описаны механизмы птичьих полетов. После
                   1553: прочтения этой книги его сыновья заинтересовались полетами. Известно,
                   1554: что отец братьев Райт был священником.
                   1555: 
                   1556: Источник:
                   1557:    1. Д. Маккалоу. Братья Райт. Люди, которые научили мир летать.
                   1558: http://flibusta.is/b/480043/read
                   1559:    2. https://ru.wikipedia.org/wiki/Маре,_Этьен-Жюль
                   1560: 
                   1561: Автор:
                   1562: Валерий Семёнов (Минск)
                   1563: 
                   1564: Вопрос 36:
                   1565: В эпизоде рассказа Уильяма Шанна разочарованный в жизни герой
                   1566: присваивает ИМ имена Шелли, Джейн, Раджани, Надин и Эллен и раздумывает,
                   1567: которая из бывших жен его доконает. В этом эпизоде описан самый жесткий
                   1568: вариант... Чего?
                   1569: 
                   1570: Ответ:
                   1571: Русской рулетки.
                   1572: 
                   1573: Комментарий:
                   1574: Герой дает пулям имена своих бывших жен. Самый жесткий вариант русской
                   1575: рулетки - это когда в барабане пять пуль.
                   1576: 
                   1577: Источник:
                   1578: У. Шанн. Чудодейственное средство. http://flibusta.is/b/119615/read
                   1579: 
                   1580: Автор:
                   1581: Ольга Ярошенко (Могилев)
                   1582: 
                   1583: Тур:
                   1584: 3 этап
                   1585: 
                   1586: Дата:
                   1587: 08-Dec-2017
                   1588: 
                   1589: Редактор:
                   1590: 1-18 - Аркадий Рух; 19-36 - Павел Свердлов
                   1591: 
                   1592: Инфо:
                   1593: Аркадий Рух благодарит за тестирование Яну Азриэль, Елену Антонову,
                   1594: Антона Волосатова, Садига Гамидова, Владимира Городецкого, Екатерину
                   1595: Дубровскую, Александра Зинченко, Ирину Зубкову, Лидию Иоффе, Николая
                   1596: Коврижных, Дмитрия Когана, Андрея Кокуленко, Наталью Комар, Евгения
                   1597: Кононенко, Галину Кузьмину, Александра Либера, Данила Маргулиса, Ирину
                   1598: Марзан, Елену Пивень, Дмитрия Сахарова, Дмитрия Слоуща. Павел Свердлов
                   1599: благодарит Бога, родителей и киноакадемию.
                   1600: 
                   1601: Вопрос 1:
                   1602:    <раздатка>
                   1603:    Мой юный друг, спросивший о странице!
                   1604:    Неумолимо время, и былых
                   1605:    Страниц уж нет на тех местах.
                   1606:    </раздатка>
                   1607:    Какую односложную фамилию, по мнению Леонида Каганова, должен носить
                   1608: автор приведенных строк?
                   1609: 
                   1610: Ответ:
                   1611: Чоч.
                   1612: 
                   1613: Комментарий:
                   1614: По имени Еггог. Каганов шутит об ошибке 404.
                   1615: 
                   1616: Источник:
                   1617: https://lleo.me/arhive/no_humor/err2000.shtml
                   1618: 
                   1619: Автор:
                   1620: Аркадий Рух
                   1621: 
                   1622: Вопрос 2:
                   1623: В одном из романов Джорджа Мартина присутствует эпизодический персонаж,
                   1624: который выжил после того, как ему сильным ударом спереди разбили шлем.
                   1625: Назовите фамилию этого персонажа.
                   1626: 
                   1627: Ответ:
                   1628: Поттер.
                   1629: 
                   1630: Комментарий:
                   1631: Такая вот аллюзия на Гарри Поттера, на лбу которого имелся шрам от удара
                   1632: Волан-де-Морта. Мартин любит вставлять в свои книги такие "пасхалки".
                   1633: 
                   1634: Источник:
                   1635: https://7kingdoms.ru/wiki/Робин_Поттер
                   1636: 
                   1637: Автор:
                   1638: Аркадий Рух
                   1639: 
                   1640: Вопрос 3:
                   1641: Герой песни Семена Слепакова отдыхает, глядя российские криминальные
                   1642: сериалы, и признаётся, что иногда он "за ментов", иногда "за воров", а
                   1643: иногда... Закончите одним словом.
                   1644: 
                   1645: Ответ:
                   1646: Засыпает.
                   1647: 
                   1648: Источник:
                   1649: https://www.youtube.com/watch?v=tJzx4PTvXis
                   1650: 
                   1651: Автор:
                   1652: Аркадий Рух
                   1653: 
                   1654: Вопрос 4:
                   1655: По одной из версий, ОН был изобретен еще в конце 80-х годов XX века
                   1656: Кэтрин Хеттингер, которая увидела, как палестинские дети бросают камни в
                   1657: израильских солдат, и решила направить их энергию в мирное русло.
                   1658: Назовите ЕГО.
                   1659: 
                   1660: Ответ:
                   1661: Спиннер.
                   1662: 
                   1663: Источник:
                   1664: https://ru.wikipedia.org/wiki/Спиннер
                   1665: 
                   1666: Автор:
                   1667: Аркадий Рух
                   1668: 
                   1669: Вопрос 5:
                   1670: Внимание, в вопросе есть замена.
                   1671:    Японский персонаж романа Орсона Скотта Карда говорит, что Япония
                   1672: искупила свои преступления во Второй мировой войне после атомных
                   1673: бомбардировок Хиросимы и Нагасаки. Его учитель в ответ возмущенно
                   1674: спрашивает, не стали ли, по его мнению, японцы ИКСАМИ. В современной
                   1675: Японии количество ИКСОВ составляет около трех процентов. Какое слово мы
                   1676: заменили ИКСАМИ?
                   1677: 
                   1678: Ответ:
                   1679: Христиане.
                   1680: 
                   1681: Комментарий:
                   1682: Воздаяние за грехи - понятие христианской этики, чуждое японской
                   1683: традиции.
                   1684: 
                   1685: Источник:
                   1686: О.С. Кард. Дети разума. http://flibusta.is/b/68275/read
                   1687: 
                   1688: Автор:
                   1689: Аркадий Рух
                   1690: 
                   1691: Вопрос 6:
                   1692: 17 мая 1536 года король Англии Генрих VIII помиловал свою жену, после
                   1693: чего пришлось вызывать из-за границы представителя ЭТОЙ ПРОФЕССИИ.
                   1694: Удивительно, но английского специалиста должного уровня найти не
                   1695: удалось. Назовите ЭТУ ПРОФЕССИЮ.
                   1696: 
                   1697: Ответ:
                   1698: Палач.
                   1699: 
                   1700: Комментарий:
                   1701: Королевское помилование состояло в том, что казнь через сожжение на
                   1702: костре было заменено для Анны Болейн обезглавливанием. При Генрихе казни
                   1703: в Англии были обычным делом.
                   1704: 
                   1705: Источник:
                   1706:    1. https://ru.wikipedia.org/wiki/Болейн,_Анна
                   1707:    2. https://ru.wikipedia.org/wiki/Помилование
                   1708: 
                   1709: Автор:
                   1710: Аркадий Рух
                   1711: 
                   1712: Вопрос 7:
                   1713: Статья на сайте antropogenez.ru [антропогенЕз ру], активно занимающемся
                   1714: борьбой со лженаукой, называется "БаальбЕк: без ГМО". В одном из слов
                   1715: предыдущего предложения мы заменили две буквы. Восстановите измененное
                   1716: слово.
                   1717: 
                   1718: Ответ:
                   1719: НЛО.
                   1720: 
                   1721: Зачет:
                   1722: UFO; УФО.
                   1723: 
                   1724: Комментарий:
                   1725: В статье доказывается, что строительство древнего храмового комплекса на
                   1726: вершине холма Телль Баальбек в Ливане обошлось без вмешательства
                   1727: пришельцев, как бы "уфологи" ни пытались доказать обратное.
                   1728: 
                   1729: Источник:
                   1730: http://antropogenez.ru/review/881/
                   1731: 
                   1732: Автор:
                   1733: Аркадий Рух
                   1734: 
                   1735: Вопрос 8:
                   1736: В рассказе Василия МидЯнина редактор литературного журнала настойчиво
                   1737: рекомендует сотрудникам вернуться к работе, так как сам за них трудиться
                   1738: не собирается. Назовите фамилию этого редактора.
                   1739: 
                   1740: Ответ:
                   1741: Пушкин.
                   1742: 
                   1743: Комментарий:
                   1744: "А работать кто будет? Пушкин?".
                   1745: 
                   1746: Источник:
                   1747: В. Мидянин. Что делать, Фауст? http://flibusta.is/b/276766/read
                   1748: 
                   1749: Автор:
                   1750: Аркадий Рух
                   1751: 
                   1752: Вопрос 9:
                   1753: [Ведущему: четко прочитать "все возможные" как два слова.]
                   1754:    В фантастической повести "Фабрика" описывается таинственный завод,
                   1755: производящий все возможные цвета. В предыдущем предложении мы пропустили
                   1756: одну букву. В каком слове?
                   1757: 
                   1758: Ответ:
                   1759: "Фарбрика".
                   1760: 
                   1761: Комментарий:
                   1762: Во многих восточноевропейских языках, включая белорусский, слово "фарбы"
                   1763: переводится как "краски". Логично, что цвета производятся не на Фабрике,
                   1764: а на Фарбрике.
                   1765: 
                   1766: Источник:
                   1767: К.А. Терина. Фарбрика. http://flibusta.is/b/515291/read
                   1768: 
                   1769: Автор:
                   1770: Аркадий Рух
                   1771: 
                   1772: Вопрос 10:
                   1773: В 2011 году один американский журнал опубликовал статью с описанием
                   1774: стратегического плана, разработанного военным ведомством США. Статья
                   1775: называлась "У Пентагона есть план, как остановить наступление ПРОПУСКА".
                   1776: Впрочем, сами военные заявили, что речь шла лишь о "творческом
                   1777: упражнении". Заполните пропуск словом, пишущимся через дефис.
                   1778: 
                   1779: Ответ:
                   1780: Зомби-апокалипсис.
                   1781: 
                   1782: Комментарий:
                   1783: Вместо привычного "потенциального противника" было предложено
                   1784: разработать стратегию действий на случай атаки живых мертвецов.
                   1785: 
                   1786: Источник:
                   1787: https://ru.wikipedia.org/wiki/CONOP_8888
                   1788: 
                   1789: Автор:
                   1790: Аркадий Рух
                   1791: 
                   1792: Вопрос 11:
                   1793: Британский сверхтяжелый танк ТОГ-2 разрабатывался с 1941 по 1944 год. По
                   1794: окончании работ над проектом танк был направлен на военную базу
                   1795: БОвингтон, где до сих пор находится... Ответьте словом греческого
                   1796: происхождения: что именно?
                   1797: 
                   1798: Ответ:
                   1799: Музей.
                   1800: 
                   1801: Комментарий:
                   1802: К концу войны нужда в этом восьмидесятитонном чудовище отпала, так что
                   1803: единственный построенный экземпляр был сразу же передан Бовингтонскому
                   1804: танковому музею.
                   1805: 
                   1806: Источник:
                   1807: https://warhead.su/2017/08/23/zheleznyy-kaput
                   1808: 
                   1809: Автор:
                   1810: Аркадий Рух
                   1811: 
                   1812: Вопрос 12:
                   1813: В пародии на одну популярную телепередачу ведущий предлагает зрителям
                   1814: слушать танец, нюхать рост, щупать громкость и ДЕЛАТЬ ЭТО. Какие два
                   1815: слова мы заменили словами "ДЕЛАТЬ ЭТО"?
                   1816: 
                   1817: Ответ:
                   1818: Смотреть "Голос".
                   1819: 
                   1820: Источник:
                   1821: https://www.1tv.ru/shows/bolshaya-raznica-tv/parodii/parodiya-na-peredachu-golos-bolshaya-raznitsa-fragment-vypuska-ot-27-01-2013
                   1822: 
                   1823: Автор:
                   1824: Аркадий Рух
                   1825: 
                   1826: Вопрос 13:
                   1827: Перед началом показа фильма Роберта Родригеса "Дети шпионов - 4"
                   1828: посетителям некоторых американских кинотеатров раздавались специальные
                   1829: карточки, на которых в определенные моменты предлагалось стирать одну из
                   1830: меток. Какую букву в предыдущем предложении мы пропустили?
                   1831: 
                   1832: Ответ:
                   1833: D.
                   1834: 
                   1835: Зачет:
                   1836: Д.
                   1837: 
                   1838: Комментарий:
                   1839: Фильм имел маркетинговое название "Дети шпионов - 4D". В нужный момент
                   1840: зрителям предлагалось стирать защитный слой с ароматических ячеек, чтобы
                   1841: ощутить тот же запах, что и герои фильма.
                   1842: 
                   1843: Источник:
                   1844: https://ru.wikipedia.org/wiki/Дети_шпионов_4D
                   1845: 
                   1846: Автор:
                   1847: Аркадий Рух
                   1848: 
                   1849: Вопрос 14:
                   1850: Внимание, в вопросе есть замена.
                   1851:    В 1972 году советский инженер-строитель Эммануил Гендель был
                   1852: направлен в Самарканд, чтобы снести минарет одной из местных мечетей.
                   1853: Интересно, что знаменитый европейский город воспользоваться услугами
                   1854: Генделя не пожелал. Какое слово в вопросе мы заменили?
                   1855: 
                   1856: Ответ:
                   1857: Выпрямить.
                   1858: 
                   1859: Зачет:
                   1860: Выровнять и т.п. по смыслу.
                   1861: 
                   1862: Комментарий:
                   1863: Эммануил Гендель - крупнейший советский специалист в области выпрямления
                   1864: и передвижения зданий. Минарет мечети Биби-Ханым в Самарканде был
                   1865: "падающей башней", однако Генделю удалось выпрямить его. А вот Пизанская
                   1866: башня, к радости туристов, так и остается наклонной.
                   1867: 
                   1868: Источник:
                   1869: https://ru.wikipedia.org/wiki/Гендель,_Эммануил_Матвеевич
                   1870: 
                   1871: Автор:
                   1872: Аркадий Рух
                   1873: 
                   1874: Вопрос 15:
                   1875: Николай Рерих назвал свою картину, изображающую летящую по ночному небу
                   1876: комету, "АЛЬФА" - ведь, по мнению художника, АЛЬФА сияет ярко, но
                   1877: недолго. В СССР АЛЬФЫ появились 4 ноября 1939 года. Назовите АЛЬФУ двумя
                   1878: словами точно.
                   1879: 
                   1880: Ответ:
                   1881: Звезда героя.
                   1882: 
                   1883: Комментарий:
                   1884: Кометы, как и герои, сияют ярко, но недолго. Золотая звезда Героя
                   1885: Советского Союза вручалась с 1939 по 1991 год.
                   1886: 
                   1887: Источник:
                   1888:    1. http://nsk.sibro.ru/photo/poster/detail/2814
                   1889:    2. https://ru.wikipedia.org/wiki/Медаль_%C2%ABЗолотая_Звезда%C2%BB_(СССР)
                   1890: 
                   1891: Автор:
                   1892: Аркадий Рух
                   1893: 
                   1894: Вопрос 16:
                   1895: По словам литературоведа Владимира ЛУкова, "АЛЬФА" Проспера Мериме
                   1896: получила свое название потому, что построена по принципу АЛЬФЫ: каждая
                   1897: из составляющих ее новелл "передает какую-то одну характерную черту,
                   1898: жест, душевное движение", которые складываются в общую картину. Назовите
                   1899: АЛЬФУ.
                   1900: 
                   1901: Ответ:
                   1902: Мозаика.
                   1903: 
                   1904: Комментарий:
                   1905: Сборник новелл "Мозаика", который Мериме выпустил в 1833 году,
                   1906: действительно состоит из очень разных новелл, никак между собой не
                   1907: связанных, однако совокупно показывающих всё многообразие жизни.
                   1908: 
                   1909: Источник:
                   1910: http://19v-euro-lit.niv.ru/19v-euro-lit/lukov-vl-merime/2-2-novelly-1829-1830.htm
                   1911: 
                   1912: Автор:
                   1913: Аркадий Рух
                   1914: 
                   1915: Вопрос 17:
                   1916: После того как в Мюнстере открылась выставка, посвященная современному
                   1917: искусству в общественном пространстве, журналистка Анна ТолстОва назвала
                   1918: его высококультурным городом. Какие две буквы в предыдущем предложении
                   1919: мы пропустили?
                   1920: 
                   1921: Ответ:
                   1922: с, п.
                   1923: 
                   1924: Комментарий:
                   1925: На выставке были представлены монументальные скульптуры, вписывающиеся в
                   1926: городской ландшафт, поэтому Мюнстер был назван ВЫСОКОСКУЛЬПТУРНЫМ.
                   1927: 
                   1928: Источник:
                   1929: https://www.kommersant.ru/doc/3329309
                   1930: 
                   1931: Автор:
                   1932: Аркадий Рух
                   1933: 
                   1934: Вопрос 18:
                   1935: Математик Лука ПачОли посвятил свой трактат "О божественной пропорции"
                   1936: герцогу ЛодовИко СфОрца. Кто выполнил иллюстрации к этому трактату?
                   1937: 
                   1938: Ответ:
                   1939: Леонардо да Винчи.
                   1940: 
                   1941: Источник:
                   1942: https://ru.wikipedia.org/wiki/Пачоли,_Лука
                   1943: 
                   1944: Автор:
                   1945: Аркадий Рух
                   1946: 
                   1947: Вопрос 19:
                   1948: (pic: 20170884.jpg)
                   1949:    Изначально рекламным слоганом пародийной комедии "Scary Movie 2"
                   1950: [скЭри мУви два] планировалось сделать фразу "We lied" [уИ лайд] - "Мы
                   1951: лгали". Какие два слова мы скрыли от вас на раздатке?
                   1952: 
                   1953: Ответ:
                   1954: No sequel [чтецу: нОу сИкуэл].
                   1955: 
                   1956: Зачет:
                   1957: Без продолжения.
                   1958: 
                   1959: Комментарий:
                   1960: Посмотрите, вам раздали изображение постера первой части фильма. На нем
                   1961: было написано "No mercy. No shame. No sequel", но потом вышла вторая
                   1962: часть.
                   1963:    Мы продолжаем наш синхрон. Второй тур - так сказать, сиквел!
                   1964: 
                   1965: Источник:
                   1966: http://www.tramvision.ru/lapsus/2005/scary2.shtml
                   1967: 
                   1968: Автор:
                   1969: Егор Сидорович
                   1970: 
                   1971: Вопрос 20:
                   1972: В мультфильме "Маша и медведь" на хоккейной клюшке можно увидеть надпись
                   1973: "Чемпион 1942". Какой символ мы заменили в предыдущем предложении?
                   1974: 
                   1975: Ответ:
                   1976: 2.
                   1977: 
                   1978: Комментарий:
                   1979: На клюшке написано "Чемпион 1242" - год ледового побоища.
                   1980: 
                   1981: Источник:
                   1982: Мультсериал "Маша и медведь", 17-я серия "Маша + каша".
                   1983: https://www.youtube.com/watch?v=AqYsbttfgaQ
                   1984: 
                   1985: Автор:
                   1986: Алексей Евдоченко
                   1987: 
                   1988: Вопрос 21:
                   1989: В окончательный вариант фильма "Белое солнце пустыни" не вошла сцена сна
                   1990: одного из персонажей. В ней он делает то же, что и герой русской песни,
                   1991: но не один раз, а несколько раз подряд. Назовите этого героя песни.
                   1992: 
                   1993: Ответ:
                   1994: Степан Разин.
                   1995: 
                   1996: Комментарий:
                   1997: Красноармейцу Сухову снится, что он - Степан Разин. Бросать в набежавшую
                   1998: волну приходится не одну восточную женщину, а нескольких.
                   1999: 
                   2000: Источник:
                   2001: https://sadalskij.livejournal.com/3262610.html
                   2002: 
                   2003: Автор:
                   2004: Егор Сидорович
                   2005: 
                   2006: Вопрос 22:
                   2007:    <раздатка>
                   2008:    [пропуск1] are from Clash of Clans, [пропуск2] are from Candy Crush
                   2009:    </раздатка>
                   2010:    Перед вами заголовок статьи об аудиториях мобильных игр, в котором мы
                   2011: пропустили два слова. Заполните каждый из пропусков одним словом.
                   2012: 
                   2013: Ответ:
                   2014: Men, Women.
                   2015: 
                   2016: Зачет:
                   2017: Мужчины, женщины.
                   2018: 
                   2019: Комментарий:
                   2020: "Men are from Clash of Clans, women are from Candy Crush". Обыгрывается
                   2021: заголовок книги Джона Грэя "Men Are from Mars, Women Are from Venus" -
                   2022: "Мужчины с Марса, женщины с Венеры".
                   2023: 
                   2024: Источник:
                   2025:    1. https://news.usc.edu/115999/men-are-from-clash-of-clans-women-are-from-candy-crush
                   2026:    2. https://en.wikipedia.org/wiki/Men_Are_from_Mars,_Women_Are_from_Venus
                   2027: 
                   2028: Автор:
                   2029: Александра Бурчалова
                   2030: 
                   2031: Вопрос 23:
                   2032: Автор зомби-хоррора "Некрополитен" парадоксальным образом замечает, что
                   2033: "зомби появились в метрополитене внезапно". Какие три слова мы заменили
                   2034: в предыдущем предложении одним?
                   2035: 
                   2036: Ответ:
                   2037: Как из-под земли.
                   2038: 
                   2039: Зачет:
                   2040: Словно из-под земли.
                   2041: 
                   2042: Источник:
                   2043: https://rudy-de.livejournal.com/255380.html
                   2044: 
                   2045: Автор:
                   2046: Павел Свердлов
                   2047: 
                   2048: Вопрос 24:
                   2049: Статья на сайте "Российской газеты" о новой услуге в московском
                   2050: метрополитене называется "Из-под земли достанут". Какие две буквы мы
                   2051: заменили в предыдущем предложении?
                   2052: 
                   2053: Ответ:
                   2054: вя.
                   2055: 
                   2056: Комментарий:
                   2057: "Доставят". В метро будут размещаться сервисы по доставке почтовых
                   2058: отправлений.
                   2059: 
                   2060: Источник:
                   2061: https://rg.ru/2016/08/04/reg-cfo/v-stolichnom-metro-nachnut-otpravliat-pisma-i-mini-posylki.html
                   2062: 
                   2063: Автор:
                   2064: Ольга Потапова
                   2065: 
                   2066: Вопрос 25:
                   2067: Газета "The Telegraph" в статье о Марин Ле Пен писала, что в случае
                   2068: победы французский политик обещала УЙТИ ПО-АНГЛИЙСКИ. Какой неологизм мы
                   2069: заменили в предыдущем предложении?
                   2070: 
                   2071: Ответ:
                   2072: Фрекзит.
                   2073: 
                   2074: Зачет:
                   2075: По созвучию.
                   2076: 
                   2077: Комментарий:
                   2078: Весной 2017 года во Франции проходили выборы президента, и кандидат
                   2079: Марин Ле Пен в случае своей победы обещала французам выход из ЕС,
                   2080: который уже потенциально окрестили Фрекзитом.
                   2081: 
                   2082: Источник:
                   2083: https://www.telegraph.co.uk/news/worldnews/europe/france/11696466/Call-me-Madame-Frexit-Front-National-leader-Marine-le-Pen-says.html
                   2084: 
                   2085: Автор:
                   2086: Алексей Евдоченко
                   2087: 
                   2088: Вопрос 26:
                   2089: В книге "Датская модель" говорится, что высокие налоги и преобладание
                   2090: бюджетного сектора обычно подавляют экономический рост, инновации и
                   2091: конкурентоспособность. Экономическая наука считает такую модель
                   2092: нежизнеспособной. Далее датская экономика называется экономикой ЕГО.
                   2093: Кого?
                   2094: 
                   2095: Ответ:
                   2096: Шмеля.
                   2097: 
                   2098: Комментарий:
                   2099: По аналогии со шмелем, который по законам физики летать не может,
                   2100: датская экономическая модель в теории обречена на провал.
                   2101: 
                   2102: Источник:
                   2103: М. Бут. Почти идеальные люди. Вся правда о жизни в "Скандинавском раю".
                   2104: http://flibusta.is/b/494052/read
                   2105: 
                   2106: Автор:
                   2107: Алексей Евдоченко
                   2108: 
                   2109: Вопрос 27:
                   2110:    <раздатка>
                   2111:    "Hm! Hm! Hm! Hm!"
                   2112:    </раздатка>
                   2113:    Перед вами название фрагмента известной оперы, который исполняет
                   2114: только что наказанный персонаж. Суть наказания состоит в воплощении в
                   2115: жизнь известной идиомы в буквальном смысле. Назовите идиому.
                   2116: 
                   2117: Ответ:
                   2118: [Закрыть] рот на замок.
                   2119: 
                   2120: Зачет:
                   2121: [Держать] рот на замке; прочие ответы, которые являются идиомами и
                   2122: означают закрытый насильственно рот.
                   2123: 
                   2124: Комментарий:
                   2125: Папагено, персонаж "Волшебной флейты", наказан за вранье и хвастовство
                   2126: надеванием на рот замка. В таком виде он пытается пожаловаться принцу
                   2127: Тамино на свое бедственное положение.
                   2128: 
                   2129: Источник:
                   2130: Опера Моцарта "Волшебная флейта", диалог Папагено и принца Тамино.
                   2131: 
                   2132: Автор:
                   2133: Ольга Потапова
                   2134: 
                   2135: Вопрос 28:
                   2136: Во времена президентства Урхо Кекконена отношения Финляндии и СССР
                   2137: значительно улучшились. За близость к социалистическому лагерю на Западе
                   2138: придумали для Финляндии прозвище, созвучное с названием другой, ныне не
                   2139: существующей страны. Напишите это прозвище.
                   2140: 
                   2141: Ответ:
                   2142: Кеккословакия.
                   2143: 
                   2144: Источник:
                   2145: Ben Livson. The Life of Dr. Mikael Livson.
                   2146: https://books.google.ru/books?id=jM2jg-AU-P8C&pg=PA31#v=onepage&q&f=false
                   2147: 
                   2148: Автор:
                   2149: Алексей Евдоченко
                   2150: 
                   2151: Вопрос 29:
                   2152: Герои 238-й серии мультсериала "Лунтик" испытывают отчаяние и страх
                   2153: нападения, пока на помощь им не приходят светлячки. Название серии
                   2154: совпадает с названием фильма 1980 года, снятого по одноименной книге.
                   2155: Как называется этот фильм?
                   2156: 
                   2157: Ответ:
                   2158: "Сияние".
                   2159: 
                   2160: Источник:
                   2161:    1. Мультсериал "Лунтик", 238-я серия "Сияние".
                   2162: https://www.youtube.com/watch?v=zMvkwbqoRc0
                   2163:    2. https://www.kinopoisk.ru/film/siyanie-1980-409/
                   2164: 
                   2165: Автор:
                   2166: Яна Самойлович
                   2167: 
                   2168: Вопрос 30:
                   2169: (pic: 20170885.jpg)
                   2170:    Какой псевдоним из одного слова взял серийный убийца, в письмах
                   2171: которого содержались такие знаки?
                   2172: 
                   2173: Ответ:
                   2174: Зодиак.
                   2175: 
                   2176: Комментарий:
                   2177: Мы раздали вам 12 знаков. Псевдоним напрашивается сам собой.
                   2178: 
                   2179: Источник:
                   2180:    1. http://retrobazar.com/journal/izvestnye-ljudi/1004_zodiak-istorija-serijnogo-ubijcy.html
                   2181:    2. https://thoughtcatalog.com/julia-metraux/2018/09/7-insane-but-weirdly-believable-theories-about-the-zodiac-killer/
                   2182: 
                   2183: Автор:
                   2184: Павел Свердлов
                   2185: 
                   2186: Вопрос 31:
                   2187: Считается, что пасмурная погода в Сиэтле является одной из причин
                   2188: повышенного уровня самоубийств. Поэтому неудивительно, что штат
                   2189: Вашингтон стал одним из первых, где СДЕЛАЛИ ЭТО. Какие два слова,
                   2190: начинающиеся на соседние буквы алфавита, мы заменили словами "СДЕЛАЛИ
                   2191: ЭТО"?
                   2192: 
                   2193: Ответ:
                   2194: Легализовали марихуану.
                   2195: 
                   2196: Комментарий:
                   2197: Вот такой способ борьбы с самоубийствами.
                   2198: 
                   2199: Источник:
                   2200:    1. https://lookatusa.com/geo/goroda/seattle/al-stl.html
                   2201:    2. https://www.kommersant.ru/doc/3150093
                   2202: 
                   2203: Автор:
                   2204: Алексей Евдоченко
                   2205: 
                   2206: Вопрос 32:
                   2207: Герой сериала "Безумцы", действие которого происходит в шестидесятых,
                   2208: починил одно устройство и продемонстрировал, что оно работает. За это
                   2209: его назвали быстрым коричневым лисом. Какое устройство он починил?
                   2210: 
                   2211: Ответ:
                   2212: Пишущая машинка.
                   2213: 
                   2214: Зачет:
                   2215: Печатная машинка.
                   2216: 
                   2217: Комментарий:
                   2218: "The quick brown fox jumps over the lazy dog" - англоязычная панграмма.
                   2219: Панграммами обычно тестируют работу печатной машинки. Действие сериала
                   2220: "Безумцы" происходит в 1960-х годах, этим можно отсечь клавиатуру.
                   2221: 
                   2222: Источник:
                   2223:    1. Телесериал "Безумцы", s07e13.
                   2224:    2. https://en.wikipedia.org/wiki/The_quick_brown_fox_jumps_over_the_lazy_dog
                   2225: 
                   2226: Автор:
                   2227: Александра Бурчалова
                   2228: 
                   2229: Вопрос 33:
                   2230: Один блогер сетует, что лопапейсу, продающуюся в сувенирных магазинах
                   2231: Исландии, ВАЛЯТ на ОТВАЛИ. Напишите любое из слов, которые мы заменили в
                   2232: предыдущем предложении.
                   2233: 
                   2234: Ответ:
                   2235: Вяжут.
                   2236: 
                   2237: Зачет:
                   2238: Отвяжись.
                   2239: 
                   2240: Комментарий:
                   2241: Кстати, лопапейса - это такой свитер. Это ведь нормально, что в Исландии
                   2242: в сувенирных магазинах продаются свитера?
                   2243: 
                   2244: Источник:
                   2245: https://tchainka.livejournal.com/507075.html
                   2246: 
                   2247: Автор:
                   2248: Ольга Потапова
                   2249: 
                   2250: Вопрос 34:
                   2251: Герой рассказа Дональда Уэстлейка называет полицейские рации слуховым
                   2252: эквивалентом ЕГО: из небольшой коробочки доносятся только треск и
                   2253: повизгивание, но полицейские всё понимают. Назовите ЕГО двумя словами.
                   2254: 
                   2255: Ответ:
                   2256: Врачебный почерк.
                   2257: 
                   2258: Зачет:
                   2259: Почерк врачей, медицинский рецепт и т.п. по смыслу.
                   2260: 
                   2261: Комментарий:
                   2262: Врачи тоже как-то понимают всё, написанное другими врачами.
                   2263: 
                   2264: Источник:
                   2265: Д. Уэстлейк. Воровская дюжина. http://flibusta.is/b/514231/read
                   2266: 
                   2267: Автор:
                   2268: Егор Сидорович
                   2269: 
                   2270: Вопрос 35:
                   2271: Мэт Ширли описывал ИКС в виде диаграммы с восходящими кривыми чувства
                   2272: паники и времени, потраченного на просмотр сериалов, а также нисходящей
                   2273: кривой желания жить. В сообществе wise_advice на портале
                   2274: www.liveinternet.ru ИКС образно сравнивают с концом августа. Назовите
                   2275: ИКС двумя словами, начинающимися на одну и ту же букву.
                   2276: 
                   2277: Ответ:
                   2278: Вечер воскресенья.
                   2279: 
                   2280: Источник:
                   2281:    1. http://medialeaks.ru/2310qaz-diagrammyi-na-kazhdyiy-den
                   2282:    2. http://www.liveinternet.ru/community/wise_advice/post397220273/
                   2283:    3. http://www.cluber.com.ua/lifestyle/poleznyie-sovetyi/2017/03/vot-chto-uspeshnyie-lyudi-delayut-v-voskresene-vecherom/
                   2284: 
                   2285: Автор:
                   2286: Станислав Габрусевич
                   2287: 
                   2288: Вопрос 36:
                   2289: На выставке в Музее современного искусства в Нью-Йорке представлено 111
                   2290: моделей одежды и аксессуаров, оказавших наибольшее влияние на моду
                   2291: XX-XXI веков. Назовите фирму-производителя предмета, находящегося первым
                   2292: в списке экспонатов.
                   2293: 
                   2294: Ответ:
                   2295: "Levi's".
                   2296: 
                   2297: Комментарий:
                   2298: Список был отсортирован по порядку 0-9, A-Z. Первым предметом,
                   2299: представленным на выставке, была модель джинсов 501 Jeans by Levi's.
                   2300: 
                   2301: Источник:
                   2302:    1. https://static.dezeen.com/uploads/2017/09/items-is-fashion-modern-moma_dezeen_2364_col_1-852x569.jpg
                   2303:    2. https://medium.com/items/items-is-fashion-modern-checklist-e353b83e7652
                   2304:    3. http://www.levi.com/US/en_US/features/501-series (название модели
                   2305: на сайте производителя)
                   2306: 
                   2307: Автор:
                   2308: Ольга Потапова
                   2309: 
                   2310: Тур:
                   2311: 4 этап
                   2312: 
                   2313: Дата:
                   2314: 16-Feb-2018
                   2315: 
                   2316: Редактор:
                   2317: Иван Топчий и Павел Малецкий (Минск)
                   2318: 
                   2319: Инфо:
                   2320: Редакторы благодарят за тестирование и ценные замечания: команду "Карты,
                   2321: простынь, два носка", Валерия Семёнова, Александра Курзинера, Валентина
                   2322: Копочеля, Ивана Мозолюка, Алексея Волчка, Алексея Гончарова, Гражину
                   2323: Бобилевич, Германа Чепикова, Арину Арзамазову, Екатерину Мудрагель.
                   2324: 
                   2325: Вопрос 1:
                   2326: [Нулевой вопрос]
                   2327:    Внимание, в вопросе есть замена.
                   2328:    Статья на тематическом сайте рассказывает, что в некоторых монастырях
                   2329: можно попробовать и миньон - но его монахи готовят обычно для себя,
                   2330: чтобы исполнять свои обязанности после трапезы надлежащим образом. Какое
                   2331: слово мы заменили в этом вопросе?
                   2332: 
                   2333: Ответ:
                   2334: Сингл.
                   2335: 
                   2336: Комментарий:
                   2337: Речь в статье идет не о стейках, а о пиве. В частности, о популярных
                   2338: бельгийских аббатских сортах, таких как дуббель, трипель и квадрупель.
                   2339: Для собственных нужд монахи варят светлое пиво с низким содержанием
                   2340: алкоголя, чтобы находиться в трезвом уме при исполнении своих служебных
                   2341: обязанностей. Сингл и миньон - это еще и типы музыкальных пластинок.
                   2342: 
                   2343: Источник:
                   2344:    1. https://www.pivo.by/articles/reviews/belgian-beer
                   2345:    2. https://ru.wikipedia.org/wiki/Сингл
                   2346:    3. https://ru.wikipedia.org/wiki/Мини-альбом
                   2347: 
                   2348: Автор:
                   2349: Алексей Гончаров (Минск)
                   2350: 
                   2351: Вопрос 2:
                   2352: В начале XIX века в США появилась настольная игра "Особняк счастья".
                   2353: Игра учила детей христианским ценностям, поэтому привычного предмета в
                   2354: ней не было, и его роль исполнял ИКС. Такая роль ИКСА нам с вами не
                   2355: должна показаться необычной. Назовите слово, которое мы заменили ИКСОМ.
                   2356: 
                   2357: Ответ:
                   2358: Волчок.
                   2359: 
                   2360: Комментарий:
                   2361: [Ведущему: желательно объявить автора вопроса.]
                   2362:    Обычно в играх для определения случайного числа используется
                   2363: игральный кубик, но он четко ассоциируется с азартными играми. В
                   2364: "Особняке счастья" использовался особый волчок с числами на гранях. В
                   2365: такой же роли - выбор случайного вопроса - используется волчок в
                   2366: элитарном клубе "Что? Где? Когда?".
                   2367: 
                   2368: Источник:
                   2369: https://en.wikipedia.org/wiki/The_Mansion_of_Happiness
                   2370: 
                   2371: Автор:
                   2372: Алексей Волчок (Минск)
                   2373: 
                   2374: Вопрос 3:
                   2375: Последний оргАн фирмы "ВУрлитцер" был произведен в начале 1940-х. Он мог
                   2376: воспроизводить звуки ветра, грома, клаксонов и многие другие. Мы не
                   2377: спрашиваем, какие четыре буквы пропущены в тексте вопроса. Назовите
                   2378: учреждения, в которые поставлялись эти инструменты.
                   2379: 
                   2380: Ответ:
                   2381: Кинотеатры.
                   2382: 
                   2383: Зачет:
                   2384: Кинозалы.
                   2385: 
                   2386: Комментарий:
                   2387: Специальные киноорганы могли не только имитировать пианино, но и
                   2388: воспроизводить другие звуки для немого кино. Из-за распространения
                   2389: звукового кино их перестали производить в начале 1940-х.
                   2390: 
                   2391: Источник:
                   2392: https://ru.wikipedia.org/wiki/Театральный_орган
                   2393: 
                   2394: Автор:
                   2395: Алексей Волчок (Минск)
                   2396: 
                   2397: Вопрос 4:
                   2398: Блиц.
                   2399:    Каждый стикер из набора "Философия" в Telegram [телегрАм] состоит из
                   2400: портрета знаменитого человека и подписи, которую тот якобы произносит.
                   2401:    1. Назовите человека, изображенного на стикере с надписью "Не думаю".
                   2402:    2. Назовите человека, изображенного на стикере с надписью "Ничего не
                   2403: знаю".
                   2404:    3. Назовите человека, изображенного на стикере с надписью "Сказал как
                   2405: отрезал".
                   2406: 
                   2407: Ответ:
                   2408:    1. [Рене] Декарт.
                   2409:    2. Сократ.
                   2410:    3. [Уильям] Оккам.
                   2411: 
                   2412: Комментарий:
                   2413: В первом вопросе отсылка к словам Декарта "Мыслю, следовательно
                   2414: существую", во втором - к словам Сократа "Я знаю, что ничего не знаю", в
                   2415: третьем речь идет о принципе "Бритва Оккама".
                   2416: 
                   2417: Источник:
                   2418: Набор стикеров "Философия" в Telegram.
                   2419: https://tlgrm.ru/stickers/Philosophy_Guys
                   2420: 
                   2421: Автор:
                   2422: Антон Шевченя (Минск)
                   2423: 
                   2424: Вопрос 5:
                   2425: Рекламный слоган Беларусбанка предлагает сделать незабываемым ЕГО.
                   2426: Назовите ЕГО словом английского происхождения.
                   2427: 
                   2428: Ответ:
                   2429: Пин-код.
                   2430: 
                   2431: Комментарий:
                   2432: Беларусбанк предлагает клиентам самим придумать себе пин-код, чтобы уже
                   2433: точно не забыть его. Пин-код обычно состоит из четырех цифр, вопрос в
                   2434: туре стоит четвертым, что тоже могло помочь некоторым командам. А могло
                   2435: и не помочь.
                   2436: 
                   2437: Источник:
                   2438: https://www.belarusbank.by/ru/33139/press/bank_news/31842
                   2439: 
                   2440: Автор:
                   2441: Павел Малецкий (Минск)
                   2442: 
                   2443: Вопрос 6:
                   2444: Андреас Кацулас исполнял роль инопланетного посла Г'Кара в сериале
                   2445: "Вавилон-5". Когда его спросили, почему он называет своего персонажа
                   2446: ЖэкАром, тот ответил: "Я решил, что он - ПРОПУСК". В итоге это хорошо
                   2447: сочеталось с вычурным непрактичным шарфом, который носил персонаж.
                   2448: Назовите слово, которое мы пропустили.
                   2449: 
                   2450: Ответ:
                   2451: Француз.
                   2452: 
                   2453: Источник:
                   2454: "Вавилон-5. Начало", комментарии создателей.
                   2455: 
                   2456: Автор:
                   2457: Алексей Волчок (Минск)
                   2458: 
                   2459: Вопрос 7:
                   2460: [Ведущему: очень четко прочитать окончание в слове "спасениЯ" и слегка
                   2461: выделить интонационно слово "миллиардов".]
                   2462:    По подсчетам пользователя портала "Quora" [квуОра], на спасения
                   2463: персонажей этого актера ушло не менее 900 миллиардов долларов. Назовите
                   2464: этого актера.
                   2465: 
                   2466: Ответ:
                   2467: [Мэтт] Деймон.
                   2468: 
                   2469: Комментарий:
                   2470: Мэтт Деймон - известный американский актер, персонажей которого в
                   2471: множестве фильмов пытаются спасти. Например, в фильмах "Марсианин",
                   2472: "Спасти рядового Райана" и "Интерстеллар".
                   2473: 
                   2474: Источник:
                   2475: https://www.quora.com/How-much-money-has-been-spent-attempting-to-bring-Matt-Damon-back-from-distant-places
                   2476: 
                   2477: Автор:
                   2478: Иван Мозолюк (Минск)
                   2479: 
                   2480: Вопрос 8:
                   2481: Однажды в гости к ЕГО родителям пришел Иван Дмитриев и решил подшутить
                   2482: над ребенком, сказав: "Какой арабчик!". Но десятилетний мальчик не
                   2483: растерялся и ответил: "Зато не рябчик!", намекая на рябое от оспы лицо
                   2484: Дмитриева. Кто был этим находчивым мальчиком?
                   2485: 
                   2486: Ответ:
                   2487: [Александр Сергеевич] Пушкин.
                   2488: 
                   2489: Комментарий:
                   2490: Ну а кто ж еще? Редакторы намеренно поставили этот вопрос седьмым. Ведь,
                   2491: если учитывать нулевой вопрос, он как раз получается восьмым.
                   2492: 
                   2493: Источник:
                   2494: https://kulturologia.ru/blogs/060618/39225/
                   2495: 
                   2496: Автор:
                   2497: Ксения Драгунова (Минск)
                   2498: 
                   2499: Вопрос 9:
                   2500:    <раздатка>
                   2501:    Blankadas velo unusola
                   2502:    En la nebula mara blu',
                   2503:    &#284;i kion lasis, kion volas
                   2504:    en fremdaj landoj ser&#265;i plu?
                   2505:    </раздатка>
                   2506:    Историк Линкольн Пейн пишет, что ИКС в полную силу развернулся в
                   2507: XVIII веке. Назовите слово, которое мы заменили ИКСОМ.
                   2508: 
                   2509: Ответ:
                   2510: Парус.
                   2511: 
                   2512: Комментарий:
                   2513: В раздатке - начало стихотворения Лермонтова "Парус" в переводе на
                   2514: эсперанто. Слово "развернулся" - небольшая подсказка.
                   2515: 
                   2516: Источник:
                   2517:    1. Л. Пейн. Море и цивилизация. Мировая история в свете развития
                   2518: мореходства.
                   2519: https://books.google.ru/books?id=7tcyDwAAQBAJ&pg=PT828#v=onepage&q&f=false
                   2520:    2. https://eo.wikipedia.org/wiki/Velo_(versa%C4%B5o)
                   2521: 
                   2522: Автор:
                   2523: Константин Егиазаров, Валерий Семёнов (Минск)
                   2524: 
                   2525: Вопрос 10:
                   2526: В юмористическом фэнтезийном романе героям объясняют происхождение
                   2527: местной достопримечательности. По легенде, один великан оскорбил богов,
                   2528: и те погрузили его в землю так, что над поверхностью осталась только
                   2529: одна часть тела. Ответьте точно: какая именно?
                   2530: 
                   2531: Ответ:
                   2532: Средний палец [руки].
                   2533: 
                   2534: Комментарий:
                   2535: Без комментариев.
                   2536: 
                   2537: Источник:
                   2538: Ю.А. Фирсанова. Наколдованная любовь.
                   2539: https://books.google.ru/books?id=YlvcDAAAQBAJ&pg=PT91#v=onepage&q&f=false
                   2540: 
                   2541: Автор:
                   2542: Ольга Берёзко (Минск)
                   2543: 
                   2544: Вопрос 11:
                   2545: Один из разделов статьи в журнале "Популярная механика" называется
                   2546: "Хлопок одной ладонью". В нем упоминаются Майкл Фарадей, Джеймс Максвелл
                   2547: и, конечно, Дирак. Назовите объект этой статьи двумя словами, которые
                   2548: начинаются на одну и ту же букву.
                   2549: 
                   2550: Ответ:
                   2551: Магнитный монополь.
                   2552: 
                   2553: Комментарий:
                   2554: Майкл Фарадей и Джеймс Максвелл были основоположниками
                   2555: электромагнетизма. Поль Дирак (имя ученого - небольшая подсказка)
                   2556: предположил существование частицы с магнитным зарядом, которая, правда,
                   2557: до сих пор не обнаружена. Магнитный монополь в статье образно
                   2558: сравнивается с хлопком одной ладонью - одним из центральных понятий
                   2559: дзен.
                   2560: 
                   2561: Источник:
                   2562:    1. https://www.popmech.ru/science/6791-dzhinn-iz-lukovitsy-magnit-ob-odnom-polyuse/
                   2563:    2. https://ru.wikipedia.org/wiki/Магнитный_монополь
                   2564: 
                   2565: Автор:
                   2566: Алексей Гончаров (Минск)
                   2567: 
                   2568: Вопрос 12:
                   2569: В мае 2011 года Трисс Меригольд - привлекательная чародейка, героиня
                   2570: саги о Ведьмаке - появилась в НЕМ и даже "дала" интервью. Назовите ЕГО
                   2571: двумя словами, которые начинаются на одну и ту же букву.
                   2572: 
                   2573: Ответ:
                   2574: Польский "Плэйбой".
                   2575: 
                   2576: Источник:
                   2577: http://www.kaermorhen.ru/modules.php?name=News&file=view&sid=1234
                   2578: 
                   2579: Автор:
                   2580: Ксения Драгунова (Минск)
                   2581: 
                   2582: Вопрос 13:
                   2583: Ричард Уильямс едва не потерял обе ноги из-за обморожения, проведя
                   2584: несколько часов в ледяной воде. Однако через 12 лет в Париже он смог
                   2585: стать олимпийским чемпионом по теннису. К сожалению, его отец погиб и не
                   2586: разделил с ним радость победы. А где отец Уильямса провел свой последний
                   2587: день?
                   2588: 
                   2589: Ответ:
                   2590: На "Титанике".
                   2591: 
                   2592: Зачет:
                   2593: По слову "Титаник".
                   2594: 
                   2595: Комментарий:
                   2596: Уильямс и его отец были пассажирами "Титаника". При крушении Ричард
                   2597: доплыл до перевернутой шлюпки и просидел на ней по колено в воде, из-за
                   2598: чего и отморозил ноги. А вот отец Уильямса в крушении не выжил. Кстати,
                   2599: уже в 1912 году Уильямс выиграл US Open.
                   2600: 
                   2601: Источник:
                   2602:    1. http://sport.tut.by/news/aboutsport/444590.html
                   2603:    2. https://en.wikipedia.org/wiki/R._Norris_Williams
                   2604:    3. https://www.tennisfame.com/hall-of-famers/richard-williams
                   2605: 
                   2606: Автор:
                   2607: Александр Курзинер (Минск)
                   2608: 
                   2609: Вопрос 14:
                   2610: Изначально британский писатель планировал назвать роман "Мертвый, не
                   2611: мертвый". Однако издатель сначала сократил заглавие до "Не мертвый", а в
                   2612: последний момент и вовсе заменил его именем собственным. Каким?
                   2613: 
                   2614: Ответ:
                   2615: Дракула.
                   2616: 
                   2617: Комментарий:
                   2618: Заглавный герой действительно не то чтобы мертв, но и не то чтобы жив.
                   2619:    Для тех, кто написал в ответной карточке слово "Франкенштейн",
                   2620: сообщаем, что название романа Мэри Шелли целиком звучит как
                   2621: "Франкенштейн, или Современный Прометей". Да и написан он не писателем,
                   2622: а писательницей.
                   2623: 
                   2624: Источник:
                   2625: https://www.adme.ru/tvorchestvo-pisateli/14-legendarnyh-knig-kotorye-avtory-sobiralis-nazvat-sovsem-po-drugomu-1412415/
                   2626: 
                   2627: Автор:
                   2628: Мария Орановская-Малецкая (Минск)
                   2629: 
                   2630: Вопрос 15:
                   2631: В вопросе есть замена.
                   2632:    Политическую ситуацию в России перед развалом империи Петр Столыпин
                   2633: описал словом "АЛЬФА". Продолжение игры "АЛЬФА" получило название "Осада
                   2634: Белого дома". Назовите АЛЬФУ.
                   2635: 
                   2636: Ответ:
                   2637: Перестройка.
                   2638: 
                   2639: Зачет:
                   2640: Perestroika.
                   2641: 
                   2642: Комментарий:
                   2643: "Не то, конечно, в стране, находящейся в периоде перестройки, а
                   2644: следовательно, и брожения", - говорил Столыпин. Восьмьюдесятью годами
                   2645: спустя новая Перестройка тоже запустила процессы, приведшие к развалу
                   2646: империи. Осада Белого дома в Москве происходила во время
                   2647: конституционного кризиса в октябре 1993 года.
                   2648: 
                   2649: Источник:
                   2650:    1. М.В. Зыгарь. Империя должна умереть.
                   2651: http://flibusta.is/b/521912/read
                   2652:    2. https://ru.wikipedia.org/wiki/Perestroika
                   2653: 
                   2654: Автор:
                   2655: Валентин Копочель (Минск)
                   2656: 
                   2657: Вопрос 16:
                   2658: 9 мая 2017 года ветеран баскетбола Ману Джинобили поставил решающий
                   2659: блок-шот на последних секундах матча плей-офф НБА. Итог этой встречи
                   2660: один из русскоязычных фанатов прокомментировал известной фразой из
                   2661: четырех слов. Воспроизведите ее.
                   2662: 
                   2663: Ответ:
                   2664: Спасибо деду за победу.
                   2665: 
                   2666: Комментарий:
                   2667: Блок-шот Джинобили позволил команде "Сан-Антонио" выиграть важную
                   2668: встречу. Иронизируя над возрастом 39-летнего на момент матча
                   2669: баскетболиста, один из болельщиков написал "Спасибо деду за победу".
                   2670: Дата матча - 9 мая и слово "ветеран" были небольшими подсказками.
                   2671: 
                   2672: Источник:
                   2673: https://www.sports.ru/basketball/1051104804.html
                   2674: 
                   2675: Автор:
                   2676: Валентин Копочель (Минск)
                   2677: 
                   2678: Вопрос 17:
                   2679: [Ведущему: сделать небольшую паузу между "Ай" и "Тен".]
                   2680:    Среди американских болельщиков встречи баскетбольных команд "Хьюстон
                   2681: Рокетс" и "Сан-Антонио Спёрз" получили прозвище Ай-Тен. Дело в том, что
                   2682: Ай-Тен - это... Что?
                   2683: 
                   2684: Ответ:
                   2685: Трасса, соединяющая Хьюстон и Сан-Антонио.
                   2686: 
                   2687: Зачет:
                   2688: По словам "трасса", "дорога" или "шоссе" с упоминанием городов или без
                   2689: него.
                   2690: 
                   2691: Источник:
                   2692: https://ru.wikipedia.org/wiki/I-10
                   2693: 
                   2694: Автор:
                   2695: Павел Малецкий (Минск)
                   2696: 
                   2697: Вопрос 18:
                   2698: Автор вышедшей в самом конце 2017 года статьи "Легенда ПРОПУСК"
                   2699: рассуждает о том, что удачный фильм просто обречен на сиквелы, ремейки и
                   2700: подражания. Какие три символа мы пропустили?
                   2701: 
                   2702: Ответ:
                   2703: N 18.
                   2704: 
                   2705: Комментарий:
                   2706: Статья посвящена только что вышедшему в прокат фильму "Движение вверх".
                   2707: Ее автор рассуждает о сюжетной и стилистической схожести новинки и
                   2708: фильма "Легенда N 17". Поэтому и называет "Движение вверх" легендой N
                   2709: 18. Данный вопрос - семнадцатый в туре. Но с учетом нулевого он и 17-й,
                   2710: и 18-й одновременно.
                   2711: 
                   2712: Источник:
                   2713: https://www.kp.by/daily/26776/3809458/
                   2714: 
                   2715: Автор:
                   2716: Павел Малецкий (Минск)
                   2717: 
                   2718: Вопрос 19:
                   2719: В период НЭПа в Советской России наблюдались послабления по многим
                   2720: аспектам жизни общества. Так, 1 мая 1921 года многие петроградцы
                   2721: предпочли ИКС большевистской демонстрации. Где проходит ИКС на картине
                   2722: второй половины XIX века?
                   2723: 
                   2724: Ответ:
                   2725: В Курской губернии.
                   2726: 
                   2727: Комментарий:
                   2728: В 1921 году 1 мая совпало с днем Пасхи. В период НЭПа религия не была
                   2729: под официальным запретом, поэтому многие петроградцы предпочли
                   2730: отправиться на крестный ход, а не на демонстрацию. "Крестный ход в
                   2731: Курской губернии" - известная картина Ильи Репина.
                   2732: 
                   2733: Источник:
                   2734:    1. Н.Б. Лебина. Советская повседневность: нормы и аномалии. От
                   2735: военного коммунизма к большому стилю. http://flibusta.is/b/415924/read
                   2736:    2. https://ru.wikipedia.org/wiki/Крестный_ход_в_Курской_губернии
                   2737: 
                   2738: Автор:
                   2739: Валерий Семёнов, Павел Малецкий (Минск)
                   2740: 
                   2741: Вопрос 20:
                   2742: Дуплет.
                   2743:    1. В начале XX века в Англии купить ИКС можно было за три сотни
                   2744: фунтов - заработок фермера за шесть лет. Назовите ИКС одним словом.
                   2745:    2. Русскому поэту-славянофилу Степану Шевырёву принадлежит авторство
                   2746: фразы "загнивающий Запад". В 1860 году Шевырёв навсегда СДЕЛАЛ ЭТО.
                   2747: Сделал что?
                   2748: 
                   2749: Ответ:
                   2750:    1. Трактор.
                   2751:    2. Уехал из России.
                   2752: 
                   2753: Зачет:
                   2754:    2. По смыслу.
                   2755: 
                   2756: Источник:
                   2757:    1. Документальный сериал "Эдвардианская ферма", 3-я серия.
                   2758:    2. https://ru.wikipedia.org/wiki/Гнилой_Запад
                   2759:    3. https://ru.wikipedia.org/wiki/Шевырёв,_Степан_Петрович
                   2760: 
                   2761: Автор:
                   2762: Павел Митар (Минск)
                   2763: 
                   2764: Вопрос 21:
                   2765:    <раздатка>
                   2766:    Буржуазные ценности
                   2767:    </раздатка>
                   2768:    В названии магазина антиквариата мы пропустили один символ.
                   2769: Восстановите это название в исходном виде.
                   2770: 
                   2771: Ответ:
                   2772: Б/уржуазные ценности.
                   2773: 
                   2774: Комментарий:
                   2775: Антиквариат скорее всего кому-то раньше принадлежал, а значит, формально
                   2776: б/у.
                   2777: 
                   2778: Источник:
                   2779: https://www.instagram.com/by_tsennosti/
                   2780: 
                   2781: Автор:
                   2782: Надежда Лейчинская (Минск)
                   2783: 
                   2784: Вопрос 22:
                   2785: Сайт для создания реверсов музыкальных файлов называется "15:00"
                   2786: [пятнадцать ноль ноль]. Что мы заменили в предыдущем предложении?
                   2787: 
                   2788: Ответ:
                   2789: 3pm.
                   2790: 
                   2791: Зачет:
                   2792: 3 PM в любом написании.
                   2793: 
                   2794: Комментарий:
                   2795: Реверсом называется проигрывание произведения в обратном направлении.
                   2796: Сайт позволяет пользователю загрузить файлы форматов .ogg, .wav и,
                   2797: конечно, mp3. 3pm - это mp3 наоборот.
                   2798: 
                   2799: Источник:
                   2800: https://www.mp3-reverser.com/en/
                   2801: 
                   2802: Автор:
                   2803: Алексей Гончаров (Минск)
                   2804: 
                   2805: Вопрос 23:
                   2806: Сторонники БРИТАНСКОГО стиля жизни предпочитают шерстяным свитерам
                   2807: пальто и не боятся выходить из дома в плохую погоду и спорить со своими
                   2808: гостями. Недавно появившееся название этого БРИТАНСКОГО стиля в русском
                   2809: языке совпадает с названием европейского города. Какого?
                   2810: 
                   2811: Ответ:
                   2812: Брюгге.
                   2813: 
                   2814: Комментарий:
                   2815: Датское слово "хюгге" означает состояние блаженства и счастья, которое
                   2816: мы испытываем в компании близких, почувствовав аромат "бабушкиного"
                   2817: варенья, укутавшись в теплое одеяло и попивая кофе, когда за окном
                   2818: слякоть и холод. Хюгге стало одним из слов 2017 года по версии словаря
                   2819: "Collins". Многим британцам пришелся не по вкусу этот тренд, и они
                   2820: начали противопоставлять свой образ жизни новомодному хюгге. Такой стиль
                   2821: получил название "брюгге".
                   2822: 
                   2823: Источник:
                   2824: https://lifehacker.ru/tyomnaya-storona-xygge/
                   2825: 
                   2826: Автор:
                   2827: Валентин Копочель (Минск)
                   2828: 
                   2829: Вопрос 24:
                   2830: Потоковый сервис Netflix [нЕтфликс] решил расширить свою аудиторию,
                   2831: добавив специальную озвучку с подробным описанием происходящего на
                   2832: экране, включая движения героев, выражения их лиц, интерьер, смену сцен
                   2833: и многое другое. Назовите персонажа комиксов Марвел, на сериале о
                   2834: котором было решено опробовать эту технологию.
                   2835: 
                   2836: Ответ:
                   2837: Сорвиголова.
                   2838: 
                   2839: Зачет:
                   2840: Daredevil.
                   2841: 
                   2842: Комментарий:
                   2843: Данная озвучка позволит "смотреть" сериал людям с проблемами зрения, в
                   2844: т.ч. слепым. "Сорвиголова" ("Daredevil") - сериал об одноименном
                   2845: супергерое, альтер-эго слепого адвоката Мэта Мёрдока.
                   2846: 
                   2847: Источник:
                   2848: http://geekcity.ru/sorvigolova-vyjdet-v-formate-dlya-slepyx/
                   2849: 
                   2850: Автор:
                   2851: Николай Будник (Минск)
                   2852: 
                   2853: Вопрос 25:
                   2854: [Ведущему: кавычки не озвучивать.]
                   2855:    В этом вопросе ИКС заменяет слово, которое пишется через дефис.
                   2856:    В 2007 году в Великобритании был проведен опрос, по итогам которого
                   2857: "самыми знаменитыми ИКСАМИ в истории человечества" были названы эпизоды,
                   2858: связанные с Шэрон Стоун и с уроженцем Баку. Что мы заменили ИКСОМ?
                   2859: 
                   2860: Ответ:
                   2861: Стоп-кадр.
                   2862: 
                   2863: Комментарий:
                   2864: Самыми знаменитыми стоп-кадрами британцы признали момент, когда Шэрон
                   2865: Стоун меняет местами скрещённые ноги в сцене допроса из фильма "Основной
                   2866: инстинкт", а также знаменитый гол Джеффри Хёрста в ворота сборной ФРГ,
                   2867: который был засчитан по указанию советского линейного арбитра Тофика
                   2868: Бахрамова и принес англичанам их единственное в истории чемпионство.
                   2869: 
                   2870: Источник:
                   2871:    1. https://www.eurosport.ru/football/world-cup/2010/story_sto2304132.shtml
                   2872:    2. https://ru.wikipedia.org/wiki/Бахрамов,_Тофик_Бахрам_оглы
                   2873: 
                   2874: Автор:
                   2875: Алексей Гончаров (Минск)
                   2876: 
                   2877: Вопрос 26:
                   2878: [Ведущему: кавычки не озвучивать; четко прочитать слово "отстаивать".]
                   2879:    Еще несколько десятилетий назад киномеханику на "Востоке" приходилось
                   2880: отстаивать фильмы, которые планировались к показу. В одно из слов
                   2881: предыдущего предложения мы добавили одну букву. Восстановите его в
                   2882: исходном виде.
                   2883: 
                   2884: Ответ:
                   2885: Оттаивать.
                   2886: 
                   2887: Комментарий:
                   2888: "Восток" - полярная станция. Из-за низких температур фильмы на пленке
                   2889: подмерзали, и перед просмотром их нужно было "согреть".
                   2890: 
                   2891: Источник:
                   2892: В.М. Песков. Зимовка. http://flibusta.is/b/77020/read
                   2893: 
                   2894: Автор:
                   2895: Надежда Лейчинская (Минск)
                   2896: 
                   2897: Вопрос 27:
                   2898:    <раздатка>
                   2899:    Чосон (название Кореи во время правления династии Чосон) в период с
                   2900: 1592 по 1598 годы страдал от набегов японцев (смотрите статью __________
                   2901: _____ (Корея)).
                   2902:    </раздатка>
                   2903:    Какие два слова мы пропустили в розданной вам цитате из Википедии?
                   2904: 
                   2905: Ответ:
                   2906: Семилетняя война.
                   2907: 
                   2908: Комментарий:
                   2909: "Семилетняя война (Корея)" - именно так статья называется в Википедии.
                   2910: Более известная война между европейскими державами обозначена просто как
                   2911: Семилетняя война.
                   2912: 
                   2913: Источник:
                   2914: https://ru.wikipedia.org/wiki/История_Кореи#Чосон
                   2915: 
                   2916: Автор:
                   2917: Алексей Волчок (Минск)
                   2918: 
                   2919: Вопрос 28:
                   2920: В сражении под Кульмом 10-тысячный русский отряд сдержал 30-тысячный
                   2921: корпус французов и спас от плена короля Пруссии. После битвы ИХ не
                   2922: хватило, поэтому солдаты делали ИХ сами, используя металл французских
                   2923: кирас. Назовите ИХ двумя словами точно.
                   2924: 
                   2925: Ответ:
                   2926: Железные кресты.
                   2927: 
                   2928: Зачет:
                   2929: Кульмские кресты.
                   2930: 
                   2931: Комментарий:
                   2932: Прусский король решил наградить всех участников битвы орденами. Первые
                   2933: кресты изготавливались из металла трофейных кирас самими награжденными,
                   2934: после того как они об этом узнали.
                   2935: 
                   2936: Источник:
                   2937:    1. https://ru.wikipedia.org/wiki/Сражение_под_Кульмом
                   2938:    2. https://ru.wikipedia.org/wiki/Кульмский_крест
                   2939: 
                   2940: Автор:
                   2941: Павел Митар (Минск)
                   2942: 
                   2943: Вопрос 29:
                   2944: В вопросе словом "ИКС" заменены два слова.
                   2945:    После поражения под Кульмом пленный французский генерал показал
                   2946: Александру I ИКС, и тот обещал облегчить его участь. В итоге генерал
                   2947: получил свободу меньше чем через год. Изобразите на бланке любой ИКС.
                   2948: 
                   2949: Ответ:
                   2950: Глаз в треугольнике.
                   2951: 
                   2952: Зачет:
                   2953: Циркуль; угольник; циркуль и угольник с буквой G.
                   2954: 
                   2955: Комментарий:
                   2956: По словам очевидца, генерал показал царю масонский знак, что значительно
                   2957: улучшило его положение.
                   2958: 
                   2959: Источник:
                   2960:    1. https://ru.wikipedia.org/wiki/Сражение_под_Кульмом
                   2961:    2. https://ru.wikipedia.org/wiki/Масонство
                   2962: 
                   2963: Автор:
                   2964: Павел Митар (Минск)
                   2965: 
                   2966: Вопрос 30:
                   2967: В 802 году некий Исаак прибыл из Генуи в Ахен ко двору Карла Великого.
                   2968: При нем были богатые дары от Харун ар-Рашида, в том числе один необычный
                   2969: подарок. Это позволяет сравнить Исаака с более известным человеком,
                   2970: жившим примерно на тысячу лет раньше. С кем?
                   2971: 
                   2972: Ответ:
                   2973: Ганнибал.
                   2974: 
                   2975: Комментарий:
                   2976: Этим необычным подарком был слон.
                   2977: 
                   2978: Источник:
                   2979:    1. https://republic.ru/posts/80139
                   2980:    2. https://ru.wikipedia.org/wiki/Ганнибал
                   2981: 
                   2982: Автор:
                   2983: Надежда Лейчинская (Минск)
                   2984: 
                   2985: Вопрос 31:
                   2986: В одном из эпизодов российского комедийного сериала двое друзей
                   2987: разрешают спор традиционным способом. После этого проигравший называет
                   2988: победителя ИКСОМ. Некоторое время ИКСОМ был Тирион Ланнистер. Напишите
                   2989: три слова, которые мы заменили ИКСОМ.
                   2990: 
                   2991: Ответ:
                   2992: Мастер над монетой.
                   2993: 
                   2994: Комментарий:
                   2995: Друзья улаживают спор подбрасыванием монетки. Проигравший называет
                   2996: победителя мастером над монетой. Мастер над монетой - своеобразный
                   2997: аналог министра финансов Семи королевств из цикла произведений Джорджа
                   2998: Мартина.
                   2999: 
                   3000: Источник:
                   3001: ???
                   3002: 
                   3003: Автор:
                   3004: Павел Малецкий (Минск)
                   3005: 
                   3006: Вопрос 32:
                   3007: Рожденный в Калуге в начале XVII века Иван Дмитриевич в русской
                   3008: историографии известен как Воронёнок. Напишите в исходном виде слово,
                   3009: которое мы слегка удлинили.
                   3010: 
                   3011: Ответ:
                   3012: Ворёнок.
                   3013: 
                   3014: Комментарий:
                   3015: Сын Марины Мнишек и Лжедмитрия II, прозванного Тушинским Вором.
                   3016: 
                   3017: Источник:
                   3018: https://ru.wikipedia.org/wiki/Иван_Ворёнок
                   3019: 
                   3020: Автор:
                   3021: Павел Митар (Минск)
                   3022: 
                   3023: Вопрос 33:
                   3024: Группа сортов лилейников, первый из которых зарегистрирован в 1933 году,
                   3025: отличается экзотической и необычной формой цветка. Какой трехбуквенной
                   3026: аббревиатурой эта группа обозначается в международной классификации?
                   3027: 
                   3028: Ответ:
                   3029: UFO.
                   3030: 
                   3031: Комментарий:
                   3032: Сокращение от "Unusual Form" [анъЮжуал форм]. Примерно к тем же годам
                   3033: относятся первые сообщения о летающих тарелках.
                   3034: 
                   3035: Источник:
                   3036:    1. http://www.lileynic.com.ua/index.php?id=3
                   3037:    2. https://en.wikipedia.org/wiki/Unidentified_flying_object
                   3038: 
                   3039: Автор:
                   3040: Ольга Берёзко (Минск)
                   3041: 
                   3042: Вопрос 34:
                   3043: Персонаж сериала о зомби-апокалипсисе с удивлением узнаёт, что
                   3044: преследовавший его охотник за головами сменил имя и присоединился к
                   3045: группе героев, спасающих человечество. Говоря об этом, персонаж
                   3046: упоминает дорогу в Дамаск и сравнивает бывшего противника... С кем?
                   3047: 
                   3048: Ответ:
                   3049: [Апостол] Павел.
                   3050: 
                   3051: Зачет:
                   3052: Савл.
                   3053: 
                   3054: Комментарий:
                   3055: Обращение Савла на пути в Дамаск - один из эпизодов жития апостола
                   3056: Павла, бывшего ранее среди гонителей христиан, который сыграл
                   3057: колоссальную роль в его обращении в христианство.
                   3058: 
                   3059: Источник:
                   3060: Телесериал "Nation Z", s03e01.
                   3061: 
                   3062: Автор:
                   3063: Павел Митар (Минск)
                   3064: 
                   3065: Вопрос 35:
                   3066: Сейчас правозащитница и адвокат Амаль Аламуддин является женой Джоржда
                   3067: Клуни, но о ее личной жизни в прошлом не известно ничего. Журналисты
                   3068: шутили, что, вероятно, всё это время она была занята тем, что прошибала
                   3069: ИХ. Назовите ИХ двумя словами.
                   3070: 
                   3071: Ответ:
                   3072: Стеклянные потолки.
                   3073: 
                   3074: Комментарий:
                   3075: Термин, означающий невидимый и формально никак не обозначенный барьер
                   3076: ("потолок" в карьере), ограничивающий продвижение женщин по служебной
                   3077: лестнице по причинам, не связанным с их профессиональными качествами.
                   3078: 
                   3079: Источник:
                   3080: https://meduza.io/feature/2017/06/08/u-dzhordzha-kluni-i-amal-alamuddin-rodilis-bliznetsy-za-chto-my-lyubim-etu-semyu
                   3081: 
                   3082: Автор:
                   3083: Надежда Лейчинская (Минск)
                   3084: 
                   3085: Вопрос 36:
                   3086: В передаче о периоде правления Александра II описан такой случай:
                   3087: "Оброненную рукопись подобрал мелкий чиновник. <...> Здесь, на этой
                   3088: мостовой, решалось ПРОПУСК с романом ПРОПУСК". Конец цитаты. Какие два
                   3089: слова мы дважды пропустили в этом вопросе?
                   3090: 
                   3091: Ответ:
                   3092: Что делать.
                   3093: 
                   3094: Комментарий:
                   3095: Некрасов обронил рукопись Чернышевского.
                   3096: 
                   3097: Источник:
                   3098: Проект Леонида Парфёнова "Российская империя". Выпуск 12 "Александр II,
                   3099: часть 2" (после 33-й минуты).
                   3100: 
                   3101: Автор:
                   3102: Алексей Волчок (Минск)
                   3103: 
                   3104: Вопрос 37:
                   3105: При рождении ОН получил имя Джеймс Кливленд. Чтобы прокормить семью, ОН
                   3106: неоднократно соглашался выступать против лошадей и охотничьих собак.
                   3107: Назовите ЕГО фамилию.
                   3108: 
                   3109: Ответ:
                   3110: Оуэнс.
                   3111: 
                   3112: Комментарий:
                   3113: Более известное имя "Джесси" знаменитый легкоатлет получил,
                   3114: представившись новой учительнице домашним прозвищем "J.C." (от James
                   3115: Cleveland). Несмотря на победу на Олимпиаде 1936 года, Оуэнс имел
                   3116: большие финансовые проблемы в течение всей жизни.
                   3117: 
                   3118: Источник:
                   3119:    1. http://www.gazettco.com/dzhessi-ouens-legendy-i-fakty/
                   3120:    2. https://ru.wikipedia.org/wiki/Оуэнс,_Джесси
                   3121: 
                   3122: Автор:
                   3123: Иван Мозолюк, Иван Топчий (Минск)
                   3124: 
                   3125: Тур:
                   3126: 5 этап
                   3127: 
                   3128: Дата:
                   3129: 16-Mar-2018
                   3130: 
                   3131: Редактор:
                   3132: Виталий Захарик, Сергей Дубелевич, Сергей Апанович
                   3133: 
                   3134: Вопрос 1:
                   3135: Итак, мы начинаем наш путь по вопросной дистанции!
                   3136:    В интерпретации литератора АльгЕрда БахарЕвича жители столицы
                   3137: принимают ЕГО за дезертира - из-за мощных ног и отсутствия ран на теле.
                   3138: Назовите населенный пункт, из которого ОН прибыл в столицу.
                   3139: 
                   3140: Ответ:
                   3141: Марафон.
                   3142: 
                   3143: Комментарий:
                   3144: ОН - это воин Фидиппид, принесший в Афины весть о победе над персами при
                   3145: Марафоне. В эссе Альгерда Бахаревича афиняне считают Фидиппида не
                   3146: героем, а трусом, бежавшим с поля битвы.
                   3147: 
                   3148: Источник:
                   3149: А. Бахаревич. Каляндар Бахарэвiча. - Мн.: Радыё Свабода, 2014.
                   3150: 
                   3151: Автор:
                   3152: Сергей Дубелевич
                   3153: 
                   3154: Вопрос 2:
                   3155: В конце одного детективного рассказа один из персонажей произносит
                   3156: фразу: "ИКС правосудия на этот раз не помог, но его ИГРЕК по-прежнему
                   3157: обязан карать". Назовите ИКС и ИГРЕК.
                   3158: 
                   3159: Ответ:
                   3160: Щит, меч.
                   3161: 
                   3162: Зачет:
                   3163: В любом порядке.
                   3164: 
                   3165: Источник:
                   3166: А. Конан Дойл. Постоянный пациент. http://flibusta.is/b/214607/read
                   3167: 
                   3168: Автор:
                   3169: Сергей Апанович
                   3170: 
                   3171: Вопрос 3:
                   3172: Герой романа Джонатана Литтелла получает пулевое ранение в голову. После
                   3173: долгой реабилитации он отмечает, что его восприятие мира значительно
                   3174: улучшилось, и сравнивает дырку в своей голове с ЭТИМ. Одно из названий
                   3175: ЭТОГО - Аджна. Назовите ЭТО двумя словами.
                   3176: 
                   3177: Ответ:
                   3178: Третий глаз.
                   3179: 
                   3180: Комментарий:
                   3181: "Мне казалось, что дырка у меня в голове стала третьим глазом, и этот
                   3182: глаз не выносит ослепительного света". Аджна - термин из индуизма.
                   3183: 
                   3184: Источник:
                   3185:    1. Дж. Литтелл. Благоволительницы. http://flibusta.is/b/266164/read
                   3186:    2. https://ru.wikipedia.org/wiki/Чакра#Аджна_(третий_глаз)
                   3187: 
                   3188: Автор:
                   3189: Сергей Апанович
                   3190: 
                   3191: Вопрос 4:
                   3192: Проводя параллели между известными сюжетами, Дмитрий Быков сравнивает
                   3193: Бывалого не со ВТОРЫМ, а с ПЕРВЫМ. В роли же ЧЕТВЕРТОГО писатель видит
                   3194: Шурика. Назовите ЧЕТВЕРТОГО.
                   3195: 
                   3196: Ответ:
                   3197: Д'Артаньян.
                   3198: 
                   3199: Комментарий:
                   3200: Быков сравнивает Бывалого не с Портосом (хотя он обладает внушительной
                   3201: комплекцией), а с флегматичным Атосом. Балбес - Портос, Трус - Арамис,
                   3202: Шурик - гасконец.
                   3203: 
                   3204: Источник:
                   3205: https://www.kommersant.ru/doc/2290603
                   3206: 
                   3207: Автор:
                   3208: Сергей Дубелевич
                   3209: 
                   3210: Вопрос 5:
                   3211:    <раздатка>
                   3212:    Molekularkinetischen
                   3213:    </раздатка>
                   3214:    Блогер Дмитрий Чернышёв рассказывает, как один эскимос, не знавший
                   3215: английский язык, долгое время перемещался по территории Соединенных
                   3216: Штатов, просто повторяя на вокзале то, что говорил стоявший перед ним в
                   3217: очереди человек. Чернышёв озаглавил пост об этом фразой "Ветераны ЕГО".
                   3218: На раздаточном материале вы можете увидеть слово, которое встречается в
                   3219: названии статьи Альберта Эйнштейна о НЕМ. Назовите ЕГО двумя словами.
                   3220: 
                   3221: Ответ:
                   3222: Броуновское движение.
                   3223: 
                   3224: Комментарий:
                   3225: Движение эскимоса было хаотичным, подобно движению броуновской частицы.
                   3226: Полностью название статьи в русском переводе звучит так: "О движении
                   3227: взвешенных в покоящейся жидкости частиц, требуемом
                   3228: молекулярно-кинетической теорией теплоты".
                   3229: 
                   3230: Источник:
                   3231:    1. https://mi3ch.livejournal.com/3909438.html
                   3232:    2. https://onlinelibrary.wiley.com/doi/abs/10.1002/andp.19053220806
                   3233:    3. https://fermlab.hse.ru/data/2013/06/09/1283651839/Einstein%28rus%29.pdf
                   3234: 
                   3235: Автор:
                   3236: Илья Колодник
                   3237: 
                   3238: Вопрос 6:
                   3239: В одном из своих стихотворений Борис Пастернак упоминает "птиц из породы
                   3240: люблю вас". Согласно литературоведу Константину Поливанову, в этой
                   3241: строке есть отсылка к немецкому языку, а также явно прослеживается связь
                   3242: с музыкальным произведением XIX века. Назовите его автора.
                   3243: 
                   3244: Ответ:
                   3245: [Петр Ильич] Чайковский.
                   3246: 
                   3247: Зачет:
                   3248: [Камиль] Сен-Санс.
                   3249: 
                   3250: Комментарий:
                   3251: "Люблю вас" по-немецки - "liebe dich", созвучно со словом "лебеди".
                   3252: Музыкальное произведение, в котором есть лебеди, - это "Лебединое озеро"
                   3253: Чайковского, однако вы могли подумать, что речь об "Умирающем лебеде"
                   3254: Сен-Санса, что тоже не противоречит действительности.
                   3255: 
                   3256: Источник:
                   3257: https://arzamas.academy/materials/393
                   3258: 
                   3259: Автор:
                   3260: Команда "Легионеры Боливарии"
                   3261: 
                   3262: Вопрос 7:
                   3263:    <раздатка>
                   3264:    "Недостающее звено" - внушительный двухтомник, отвечающий на все
                   3265: вопросы антропогенеза: <...> автор подробно и в меру иронично излагает
                   3266: содержание предыдущих серий происхождения человека, рассказывает обо
                   3267: всех возможных предках Homo Sapiens, объясняет, почему и как человек
                   3268: родился именно в семействе гоминидов, и даже предсказывает наше будущее
                   3269: как вида с точки зрения антропогенеза".
                   3270:    </раздатка>
                   3271:    В текст этой рецензии мы добавили две буквы к одному из слов.
                   3272: Напишите эти две буквы.
                   3273: 
                   3274: Ответ:
                   3275: Не.
                   3276: 
                   3277: Комментарий:
                   3278: На самом деле книга антрополога Станислава ДробышЕвского называется
                   3279: "Достающее звено" - но она действительно достаточно подробна, чтобы
                   3280: оправдывать такое название.
                   3281: 
                   3282: Источник:
                   3283: https://www.corpus.ru/products/drobyshevski-dostajushee-zveno-obezjany.htm
                   3284: 
                   3285: Автор:
                   3286: Елена Шибут
                   3287: 
                   3288: Вопрос 8:
                   3289: Живя в Париже в большой бедности, художник ХаИм СУтин часто работал в
                   3290: полуобморочном состоянии. Дело в том, что любому приему пищи должен был
                   3291: предшествовать ОН. Назовите ЕГО словом иностранного происхождения.
                   3292: 
                   3293: Ответ:
                   3294: Натюрморт.
                   3295: 
                   3296: Комментарий:
                   3297: Сутину нечасто удавалось добыть еду, но даже когда он это делал, то
                   3298: всегда сначала рисовал с нее натюрморт, а уже потом принимался за еду.
                   3299: 
                   3300: Источник:
                   3301: https://kulturologia.ru/blogs/241117/36766/
                   3302: 
                   3303: Автор:
                   3304: Сергей Дубелевич
                   3305: 
                   3306: Вопрос 9:
                   3307: Персонаж повести "Верный Руслан" называет своего соседа Трезорку
                   3308: ничтожным псом - несмотря на то что ИХ у Трезорки вдвое больше, чем у
                   3309: него самого. Назовите ИХ.
                   3310: 
                   3311: Ответ:
                   3312: [Буквы] Р.
                   3313: 
                   3314: Комментарий:
                   3315: Главный герой повести "Верный Руслан" - собака. Цитата: "Псом он
                   3316: оказался совсем ничтожным, даром что кличку носил с двумя рокочущими
                   3317: "Р", напоминающими рычание". В имени Трезорка букв "Р" вдвое больше, чем
                   3318: в "Руслане".
                   3319: 
                   3320: Источник:
                   3321: Г.Н. Владимов. Верный Руслан. http://flibusta.is/b/244572/read
                   3322: 
                   3323: Автор:
                   3324: Сергей Дубелевич
                   3325: 
                   3326: Вопрос 10:
                   3327: В 1982 году режиссер Питер Селларс высказал идею о создании оперы "ИКС в
                   3328: Китае" по мотивам событий, случившихся за десять лет до того. По его
                   3329: мнению, в этом сюжете был заложен огромный потенциал, базирующийся на
                   3330: встрече Востока и Запада. В одном из слов вопроса мы пропустили
                   3331: несколько букв. Воспроизведите его в исходном виде.
                   3332: 
                   3333: Ответ:
                   3334: Никсон.
                   3335: 
                   3336: Комментарий:
                   3337: Опера получила название "Никсон в Китае" и базировалась на реальных
                   3338: исторических событиях.
                   3339: 
                   3340: Источник:
                   3341: https://ru.wikipedia.org/wiki/Никсон_в_Китае
                   3342: 
                   3343: Автор:
                   3344: Виталий Захарик
                   3345: 
                   3346: Вопрос 11:
                   3347: Действие романа "Пентаграмма" происходит в крупном столичном городе.
                   3348: Ответьте двумя словами: что автор сравнил с силуэтами жирафов,
                   3349: высящимися на фоне закатного неба?
                   3350: 
                   3351: Ответ:
                   3352: Башенные краны.
                   3353: 
                   3354: Зачет:
                   3355: Строительные краны; подъемные краны; портовые краны.
                   3356: 
                   3357: Источник:
                   3358: Ю Несбё. Пентаграмма. http://flibusta.is/b/164904/read
                   3359: 
                   3360: Автор:
                   3361: Сергей Апанович
                   3362: 
                   3363: Вопрос 12:
                   3364:    <раздатка>
                   3365:    "Играют ли доги в домино? ПЕРВЫЙ и ВТОРОЙ в поисках единой теории
                   3366: мироздания".
                   3367:    </раздатка>
                   3368:    Внимание, в вопросе есть замены.
                   3369:    Книга Пола Хэлперна, посвященная двум известным ученым, в
                   3370: русскоязычном варианте получила название "Играют ли доги в домино?
                   3371: ПЕРВЫЙ и ВТОРОЙ в поисках единой теории мироздания". Догадавшись, кто
                   3372: эти ПЕРВЫЙ и ВТОРОЙ, ответьте: какие слова мы заменили словами "доги" и
                   3373: "домино"?
                   3374: 
                   3375: Ответ:
                   3376: Коты, кости.
                   3377: 
                   3378: Комментарий:
                   3379: Книга Пола Хэлперна называется "Играют ли коты в кости? Эйнштейн и
                   3380: Шрёдингер в поисках единой теории мироздания" - название косвенно
                   3381: строится на известных высказываниях этих физиков.
                   3382: 
                   3383: Источник:
                   3384: https://www.piter.com/collection/pop-science/product/igrayut-li-koty-v-kosti-eynshteyn-i-shryodinger-v-poiskah-edinoy-teorii-mirozdaniya
                   3385: 
                   3386: Автор:
                   3387: Елена Шибут
                   3388: 
                   3389: Вопрос 13:
                   3390: Саркастичный Альфонс Алле приписал ИКСУ фразу "В любой жизни есть взлеты
                   3391: и падения". А один футбольный журналист упоминает ИКСА в статье под
                   3392: названием "Из грязи в князи и обратно". Какую профессию мы заменили
                   3393: словом "ИКС"?
                   3394: 
                   3395: Ответ:
                   3396: Лифтер.
                   3397: 
                   3398: Комментарий:
                   3399: Футбольные специалисты термином "лифтер" называют команды, которые
                   3400: регулярно вылетают в низшую лигу, в следующем сезоне возвращаются в
                   3401: высшую и т.д.
                   3402: 
                   3403: Источник:
                   3404:    1. https://ru.wikiquote.org/wiki/Альфонс_Алле
                   3405:    2. https://footballhd.ru/articles/62765-fk-deportivo-iz-gryazi-v-knyazi-i-obratno.html
                   3406: 
                   3407: Автор:
                   3408: Виталий Захарик
                   3409: 
                   3410: Вопрос 14:
                   3411: В одном романе при рассказе о том, что семья героини разбогатела совсем
                   3412: недавно, упоминается запах АЛЬФЫ. Назовите АЛЬФУ.
                   3413: 
                   3414: Ответ:
                   3415: Типографская краска.
                   3416: 
                   3417: Зачет:
                   3418: Краска.
                   3419: 
                   3420: Комментарий:
                   3421: "Ее родители, простые крестьяне, разбогатели недавно, и их деньги
                   3422: по-прежнему пахли типографской краской".
                   3423: 
                   3424: Источник:
                   3425: Ю Несбё. Спаситель. http://flibusta.is/b/187899/read
                   3426: 
                   3427: Автор:
                   3428: Сергей Апанович
                   3429: 
                   3430: Вопрос 15:
                   3431: Участник передачи "Между молотом и наковальней" сравнил процесс закалки
                   3432: оружия с ЭТИМ. По-гречески ЭТО - "баптИсма". Назовите ЭТО одним словом.
                   3433: 
                   3434: Ответ:
                   3435: Крещение.
                   3436: 
                   3437: Комментарий:
                   3438: Опустив в воду раскаленный кусок металла, кузнец словно "крестил"
                   3439: клинок. В основе баптистского вероучения лежит принцип добровольного и
                   3440: сознательного крещения по вере взрослых людей при наличии твердых
                   3441: христианских убеждений и отказа от греховного образа жизни.
                   3442: 
                   3443: Источник:
                   3444: Передача "Между молотом и наковальней" на телеканале "Viasat History",
                   3445: выпуск от 13.12.2017 г.
                   3446: 
                   3447: Автор:
                   3448: Сергей Апанович
                   3449: 
                   3450: Вопрос 16:
                   3451:    <раздатка>
                   3452:    То, что у всех есть роль своя, - не факт.
                   3453:    Мир - не для нас построенная сцена.
                   3454:    А жизнь - как затянувшийся антракт,
                   3455:    Где после (ПРОПУСК 1) ждут (ПРОПУСК 2).
                   3456:    </раздатка>
                   3457:    В стихотворении Михаила БУчека пропущены фамилии современников.
                   3458: Воспроизведите их.
                   3459: 
                   3460: Ответ:
                   3461: Мендельсон, Шопен.
                   3462: 
                   3463: Комментарий:
                   3464: Авторы свадебного и похоронного маршей. Мендельсон и Шопен жили в одно и
                   3465: то же время (годы жизни - 1809-1847 и 1810-1849 соответственно).
                   3466: 
                   3467: Источник:
                   3468:    1. https://www.stihi.ru/2015/05/11/1174/
                   3469:    2. https://ru.wikipedia.org/wiki/Мендельсон,_Феликс
                   3470:    3. https://ru.wikipedia.org/wiki/Шопен,_Фридерик
                   3471: 
                   3472: Автор:
                   3473: Сергей Дубелевич
                   3474: 
                   3475: Вопрос 17:
                   3476: Послушайте цитату известного американского режиссера: "Естественный,
                   3477: свободный человек - это человек злой, деструктивный. Так называемое
                   3478: "добро" - лишь способ, с помощью которого абсолютно отчужденное от
                   3479: человека государство подчиняет людей, превращает их в...". Закончите
                   3480: фразу двумя словами.
                   3481: 
                   3482: Ответ:
                   3483: "... заводных апельсинов".
                   3484: 
                   3485: Комментарий:
                   3486: Это цитата Стэнли Кубрика о фильме "Заводной апельсин".
                   3487: 
                   3488: Источник:
                   3489: https://ru.wikiquote.org/wiki/Стэнли_Кубрик
                   3490: 
                   3491: Автор:
                   3492: Виталий Захарик
                   3493: 
                   3494: Вопрос 18:
                   3495: По одной из версий, ИКС был приобретен 2 апреля 1917 года в Нью-Йорке,
                   3496: на Пятой авеню, в магазине сантехники. Окончательная судьба ИКСА
                   3497: неизвестна - предполагают, что вскоре он был выброшен как мусор. Какое
                   3498: слово с удвоенной согласной мы заменили словом "ИКС"?
                   3499: 
                   3500: Ответ:
                   3501: Писсуар.
                   3502: 
                   3503: Комментарий:
                   3504: Речь идет о знаменитом произведении Марселя Дюшана "Фонтан", основой
                   3505: которого стал обычный фаянсовый писсуар, приобретенный в магазине
                   3506: сантехники. Вскоре после демонстрации на выставке оригинал работы
                   3507: пропал, по предположению биографа Дюшана - был выброшен. Желаем вам
                   3508: провести перерыв с пользой!
                   3509: 
                   3510: Источник:
                   3511:    1. У. Гомперц. Непонятное искусство. От Моне до Бэнкси.
                   3512: http://flibusta.is/b/468862/read
                   3513:    2. https://ru.wikipedia.org/wiki/Фонтан_(Дюшан)
                   3514: 
                   3515: Автор:
                   3516: Елена Шибут
                   3517: 
                   3518: Вопрос 19:
                   3519: Статья Александра Дмитриева, рассказывающая о качестве услуг, которые
                   3520: предлагает пассажирам своих рейсов во время полетов компания "Аэрофлот",
                   3521: называется "Сервис отличного уровня". В этом названии мы заменили два
                   3522: слова, одно из которых - предлог. Напишите их.
                   3523: 
                   3524: Ответ:
                   3525: На высоте.
                   3526: 
                   3527: Комментарий:
                   3528: Название статьи "Сервис на высоте" намекает и на высокое качество
                   3529: обслуживания, и на сферу деятельности компании "Аэрофлот".
                   3530: 
                   3531: Источник:
                   3532: https://www.kommersant.ru/doc/3293956
                   3533: 
                   3534: Автор:
                   3535: Виталий Захарик
                   3536: 
                   3537: Вопрос 20:
                   3538: (pic: 20170886.jpg)
                   3539:    Эта фотография была сделана в 1962 году. В каком городе?
                   3540: 
                   3541: Ответ:
                   3542: Берлин.
                   3543: 
                   3544: Зачет:
                   3545: Западный Берлин.
                   3546: 
                   3547: Комментарий:
                   3548: Дети строят игрушечную стену.
                   3549: 
                   3550: Источник:
                   3551: https://mi3ch.livejournal.com/3109840.html
                   3552: 
                   3553: Автор:
                   3554: Сергей Апанович
                   3555: 
                   3556: Вопрос 21:
                   3557: СинкопАле - потеря сознания, которая провоцируется резкими движениями
                   3558: головы или ее длительным вынужденным необычным положением. Напишите
                   3559: образованное от имени собственного прилагательное, которое присутствует
                   3560: в названии одного из самых известных синкопАльных состояний.
                   3561: 
                   3562: Ответ:
                   3563: Сикстинская.
                   3564: 
                   3565: Комментарий:
                   3566: Так называемый "синдром Сикстинской капеллы" возникает при длительном
                   3567: необычном положении головы - например, при рассматривании потолочной
                   3568: росписи храмовых сооружений.
                   3569: 
                   3570: Источник:
                   3571: А.С. Никифоров, Е.И. Гусев. Общая неврология.
                   3572: 
                   3573: Автор:
                   3574: Климентий Комиссаров
                   3575: 
                   3576: Вопрос 22:
                   3577: Во время гонки в рамках одного из этапов Кубка мира по биатлону
                   3578: температура воздуха оказалась выше, чем ожидалось. В связи с этим
                   3579: комментатор трансляции на телеканале "Матч-ТВ" пошутил, что ИКСА не
                   3580: повысили в звании, видимо, намекая на то, что победы российским
                   3581: спортсменам в этот день добиться не удастся. Назовите ИКСА двумя
                   3582: словами, начинающимися на одну и ту же букву.
                   3583: 
                   3584: Ответ:
                   3585: Майор Мороз.
                   3586: 
                   3587: Комментарий:
                   3588: Россиянам издавна помогает побеждать Генерал Мороз. В день гонки
                   3589: температура была не очень низкой, поэтому Мороз дотянул максимум до чина
                   3590: майора.
                   3591: 
                   3592: Источник:
                   3593: Трансляция гонки этапа Кубка мира по биатлону на телеканале Матч-ТВ,
                   3594: 30.11.2017 г.
                   3595: 
                   3596: Автор:
                   3597: Виталий Захарик
                   3598: 
                   3599: Вопрос 23:
                   3600: Рекламное агентство "FP7" [эф-пи сЕвен] разработало серию рекламных
                   3601: плакатов, которые привлекают внимание к проблеме злоупотребления
                   3602: лекарствами, позволяющими лишь на какое-то время снять боль. На одном из
                   3603: принтов человек расслабляется, лёжа на таблетке болеутоляющего, словно
                   3604: на диване. Со временем таблетка рассасывается, и человек начинает
                   3605: паниковать, ведь ИКС - это АЛЬФА. Температура АЛЬФЫ варьируется от 500
                   3606: до 1200 градусов по Цельсию. Назовите ИКС и АЛЬФУ.
                   3607: 
                   3608: Ответ:
                   3609: Пол, лава.
                   3610: 
                   3611: Источник:
                   3612: https://www.adsoftheworld.com/media/print/kkt_orthopedic_spine_centre_painkillers_lava
                   3613: 
                   3614: Автор:
                   3615: Денис Валянский
                   3616: 
                   3617: Вопрос 24:
                   3618: [Ведущему: отточия в цитате игнорировать.]
                   3619:    Журналист Уилл Гомперц пишет: "Представьте себе картонную коробку.
                   3620: Условно говоря, ее разбирают <...>, чтобы получилась плоскость,
                   3621: позволяющая нам видеть все стороны сразу. Но при этом авторы хотят
                   3622: сохранить ощущение трехмерности, чего плоский план дать не может.
                   3623: Поэтому они мысленно обходят коробку со всех сторон, выбирая такие
                   3624: ракурсы, которые дадут оптимальное представление об объекте, <...> и
                   3625: определенным образом состыковывают плоскости - <...> так, чтобы ПРОПУСК
                   3626: все-таки угадывался...". Назовите фамилию любого из двух авторов, о
                   3627: которых идет речь.
                   3628: 
                   3629: Ответ:
                   3630: Брак.
                   3631: 
                   3632: Зачет:
                   3633: Пикассо.
                   3634: 
                   3635: Комментарий:
                   3636: Пропущенное слово - "куб". Пабло Пикассо и Жорж Брак считаются
                   3637: основателями кубизма. В приведенной цитате искусствовед Гомперц
                   3638: объясняет, почему термин "кубизм" является не совсем верным.
                   3639: 
                   3640: Источник:
                   3641: У. Гомперц. Непонятное искусство. От Моне до Бэнкси.
                   3642: http://flibusta.is/b/468862/read
                   3643: 
                   3644: Автор:
                   3645: Елена Шибут
                   3646: 
                   3647: Вопрос 25:
                   3648: Блиц.
                   3649:    1. Закончите тремя словами афоризм Ашота Наданяна: "Лучше сто раз
                   3650: поехать к прадедушке, чем один раз...".
                   3651:    2. Закончите одним словом афоризм Ашота Наданяна: "Эпидемия - это
                   3652: когда туберкулезная палочка передается как ...".
                   3653:    3. По словам Ашота Наданяна, "чужой ИКС как звонок будильника:
                   3654: вставать не хочется, а надо". Назовите ИКС коротким словом.
                   3655: 
                   3656: Ответ:
                   3657:    1. "... отправиться к праотцам".
                   3658:    2. "... эстафетная".
                   3659:    3. Гимн.
                   3660: 
                   3661: Источник:
                   3662: https://ru.wikiquote.org/wiki/Ашот_Сергеевич_Наданян
                   3663: 
                   3664: Автор:
                   3665: Виталий Захарик
                   3666: 
                   3667: Вопрос 26:
                   3668: Писатель Эрнест Клайн сравнил ИКСА с пиццей без одного куска. ИКС
                   3669: упоминается в экономическом термине, описывающем реакцию на попытку
                   3670: поглощения, которая заключается в запуске компанией - мишенью программы
                   3671: встречной покупки враждебной фирмы. Назовите ИКСА.
                   3672: 
                   3673: Ответ:
                   3674: Pac-Man.
                   3675: 
                   3676: Зачет:
                   3677: PacMan; Пакман; Пэкмэн; Пэкмен.
                   3678: 
                   3679: Комментарий:
                   3680: Упомянутый термин - "защита пакмана".
                   3681: 
                   3682: Источник:
                   3683:    1. Э. Клайн. Первому игроку приготовиться.
                   3684: http://flibusta.is/b/503132/read
                   3685:    2. http://vocable.ru/termin/zaschita-v-stile-pacman.html
                   3686: 
                   3687: Автор:
                   3688: Сергей Апанович
                   3689: 
                   3690: Вопрос 27:
                   3691: В качестве ИКСА на различных сайтах можно увидеть, в том числе,
                   3692: антистрессовую игрушку, животных, пытающихся поймать свой хвост,
                   3693: изображения атома или изображения Солнечной системы. Какую деталь
                   3694: использует в качестве ИКСА сайт оружейного аукционного дома Рок Айленда?
                   3695: 
                   3696: Ответ:
                   3697: Барабан [револьвера].
                   3698: 
                   3699: Комментарий:
                   3700: В качестве индикатора загрузки часто используются различные вращающиеся
                   3701: вещи - например, атом или животные, ловящие себя за хвост. Словом
                   3702: "спиннер" называют не только антистрессовую вращающуюся игрушку, но и
                   3703: сам индикатор загрузки.
                   3704: 
                   3705: Источник:
                   3706:    1. https://www.google.ru/search?q=spinning+loading+indicator&tbm=isch
                   3707:    2. https://www.rockislandauction.com/
                   3708: 
                   3709: Автор:
                   3710: Команда "Легионеры Боливарии"
                   3711: 
                   3712: Вопрос 28:
                   3713: В вопросе есть замена.
                   3714:    Футбольный журналист, описывая игру ПОнтуса ВЕрнблума, который может
                   3715: сыграть на разных позициях на поле, называет его УНИВЕРСАЛЬНЫМ СОЛДАТОМ.
                   3716: Пик запросов в Google по словам "УНИВЕРСАЛЬНЫЙ СОЛДАТ" пришелся на
                   3717: период с 13 по 19 августа 2017 года. Какие два слова мы заменили словами
                   3718: "УНИВЕРСАЛЬНЫЙ СОЛДАТ"?
                   3719: 
                   3720: Ответ:
                   3721: "Тысячеликий герой".
                   3722: 
                   3723: Комментарий:
                   3724: Комментируя многозадачность Вернблума, журналист "Евроспорта" Дмитрий
                   3725: Панфёров употребил название книги Джозефа Кэмпбелла "Тысячеликий герой".
                   3726: Пик популярности запросов в Google объясняется тем, что во время своего
                   3727: баттла эту книгу упомянул рэпер Оксимирон, чем вызвал небывалый рост ее
                   3728: популярности. До 13 августа 2017 года - даты баттла среднее число
                   3729: запросов в Google по фразе "Тысячеликий герой" было в среднем меньше
                   3730: одного в день, после же резко возросло.
                   3731: 
                   3732: Источник:
                   3733:    1. https://www.eurosport.ru/football/europa-league/2017-2018/story_sto6642889.shtml
                   3734:    2. https://trends.google.com/trends/explore?q=тысячеликий%20герой
                   3735: 
                   3736: Автор:
                   3737: Команда "Фунтики"
                   3738: 
                   3739: Вопрос 29:
                   3740: В вопросе есть замены.
                   3741:     Варлам Шаламов пишет, что мясо любых животных теряет свой
                   3742: специфический запах, если его ПОВЕСИТЬ НА ГВОЗДЬ. ВЕШАНЬЕ НА ГВОЗДЬ -
                   3743: прием, используемый при приготовлении таких блюд, как кивиАк и хАукарль.
                   3744: Какое словосочетание мы заменили фразой "ПОВЕСИТЬ НА ГВОЗДЬ"?
                   3745: 
                   3746: Ответ:
                   3747: Закопать в землю.
                   3748: 
                   3749: Зачет:
                   3750: Зарыть в землю.
                   3751: 
                   3752: Комментарий:
                   3753: Таким вот характерным для скандинавской кулинарии способом ссыльные в
                   3754: ГУЛАГе готовили белок, ворон и других животных.
                   3755: 
                   3756: Источник:
                   3757:    1. В.Т. Шаламов. Сухим пайком. http://flibusta.is/b/474339/read
                   3758:    2. https://bigpicture.ru/?p=518846
                   3759: 
                   3760: Автор:
                   3761: Сергей Апанович
                   3762: 
                   3763: Вопрос 30:
                   3764:    <раздатка>
                   3765:    Так было и так есть, так непременно будет,
                   3766:    Пока не сдан проект, рассвет нас не разбудит.
                   3767:    Пока не стихнет спор за окнами общаги,
                   3768:    Пока ложится ПРОПУСК на белый лист бумаги.
                   3769:    </раздатка>
                   3770:    Перед вами отрывок из неофициального гимна Уральского
                   3771: архитектурно-художественного университета. Заполните пропуск,
                   3772: использовав два дефиса.
                   3773: 
                   3774: Ответ:
                   3775: Koh-i-noor.
                   3776: 
                   3777: Зачет:
                   3778: Кох-и-нур; Кох-и-нор; с незначительными грамматическими отклонениями.
                   3779: 
                   3780: Комментарий:
                   3781: "Koh-i-noor" - знаменитая чешская марка канцтоваров, в первую очередь
                   3782: карандашей.
                   3783: 
                   3784: Источник:
                   3785:    1. http://club.mabuka.ru/content/gimn-uralgakha-planshetnaya-strana
                   3786:    2. https://ru.wikipedia.org/wiki/Koh-i-Noor_Hardtmuth
                   3787: 
                   3788: Автор:
                   3789: Виталий Захарик
                   3790: 
                   3791: Вопрос 31:
                   3792: [Ведущему: отточие в цитате игнорировать.]
                   3793:    Прослушайте цитату: "Когда вся тяжесть происходящего ложится на <...>
                   3794: одного человека, он исчезает, как подломившаяся опора". Назовите
                   3795: произведение, увидевшее свет в 1957 году, фрагмент из которого мы вам
                   3796: привели.
                   3797: 
                   3798: Ответ:
                   3799: "Атлант расправил плечи".
                   3800: 
                   3801: Зачет:
                   3802: "Атлант расправил плечи. Часть II. Или - или".
                   3803: 
                   3804: Комментарий:
                   3805: Смысл цитаты намекает на роль атланта.
                   3806: 
                   3807: Источник:
                   3808: А. Рэнд. Атлант расправил плечи.
                   3809: https://books.google.ru/books?id=o37NAhf9mBMC&pg=PA449#v=onepage&q&f=false
                   3810: 
                   3811: Автор:
                   3812: Сергей Апанович
                   3813: 
                   3814: Вопрос 32:
                   3815: Берлинская авангардная группа "Einstuerzende Neubauten" [айнштУрцэндэ
                   3816: нойбАутн] известна тем, что для записи своих песен в числе прочего
                   3817: извлекает звуки из объектов, найденных в городе. В тематическом
                   3818: интернет-сообществе фотографию группы сопроводили цитатой из
                   3819: произведения первой половины XX века. Назовите автора этого
                   3820: произведения.
                   3821: 
                   3822: Ответ:
                   3823: [Владимир] Маяковский.
                   3824: 
                   3825: Комментарий:
                   3826: "А вы ноктюрн сыграть могли бы на флейте водосточных труб?" - вопрошал
                   3827: Маяковский в стихотворении "А вы могли бы?". Среди "инструментов"
                   3828: "Einstuerzende Neubauten" можно найти в том числе и водосточные трубы.
                   3829: 
                   3830: Источник:
                   3831: (pic: 20170887.jpg)
                   3832: 
                   3833: Автор:
                   3834: Команда "Легионеры Боливарии"
                   3835: 
                   3836: Вопрос 33:
                   3837: Современный писатель называет свежеоштукатуренный фасад здания сырой
                   3838: мечтой ИКСА. ИКСЫ являются главными героями фильма "БомбИ систему".
                   3839: Назовите ИКСА.
                   3840: 
                   3841: Ответ:
                   3842: Граффитист.
                   3843: 
                   3844: Зачет:
                   3845: Граффитчик; художник граффити; уличный художник.
                   3846: 
                   3847: Комментарий:
                   3848: Одно из значений слова "бомбить" - рисовать граффити.
                   3849: 
                   3850: Источник:
                   3851:    1. Ю Несбё. Спаситель. http://flibusta.is/b/187899/read
                   3852:    2. https://en.wikipedia.org/wiki/Bomb_the_System
                   3853: 
                   3854: Автор:
                   3855: Сергей Апанович
                   3856: 
                   3857: Вопрос 34:
                   3858: В стихотворении, посвященном своему знаменитому соотечественнику,
                   3859: Евгений Евтушенко упоминает запретные меловые линии и сравнивает
                   3860: адресата с поэтом, не побоявшимся "лезть в политику". Назовите этого
                   3861: соотечественника.
                   3862: 
                   3863: Ответ:
                   3864: [Лев] Яшин.
                   3865: 
                   3866: Комментарий:
                   3867: Яшин стал первым известным вратарем, активно игравшим за пределами
                   3868: штрафной. Евтушенко сравнивает Яшина с поэтами-шестидесятниками.
                   3869: 
                   3870: Источник:
                   3871: Е.А. Евтушенко. Вратарь выходит из ворот.
                   3872: http://retro-sport.sitecity.ru/ltext_0602180003.phtml?p_ident=ltext_0602180003.p_0602183141
                   3873: 
                   3874: Автор:
                   3875: Сергей Дубелевич
                   3876: 
                   3877: Вопрос 35:
                   3878: Вышедший из храма монах, персонаж Умберто Эко, принял ЭТО за огненный
                   3879: пот. А вот Анджею Сапковскому ЭТО напомнила слеза, скатившаяся по щеке.
                   3880: Назовите ЭТО двумя словами.
                   3881: 
                   3882: Ответ:
                   3883: Капля воска.
                   3884: 
                   3885: Зачет:
                   3886: Капли воска; восковая капля; восковые капли.
                   3887: 
                   3888: Комментарий:
                   3889: Герой Умберто Эко выходил из храма, погруженный в богобоязненные мысли.
                   3890: Капнувший с церковной свечи воск он сравнил с огненным потом.
                   3891: 
                   3892: Источник:
                   3893:    1. У. Эко. Имя Розы.
                   3894: https://books.google.ru/books?id=8-fRBQAAQBAJ&pg=PT209#v=onepage&q&f=false
                   3895:    2. А. Сапковский. Меч Предназначения.
                   3896: http://flibusta.is/b/442970/read
                   3897: 
                   3898: Автор:
                   3899: Сергей Апанович
                   3900: 
                   3901: Вопрос 36:
                   3902: По словам художника-абстракциониста Василия Кандинского, "АЛЬФА в нашем
                   3903: представлении является теснейшей и единственной в своем роде связью
                   3904: молчания и речи. Царство АЛЬФЫ беспредельно". Назовите АЛЬФУ одним
                   3905: словом.
                   3906: 
                   3907: Ответ:
                   3908: Точка.
                   3909: 
                   3910: Комментарий:
                   3911: А мы ставим точку в нашем пакете.
                   3912: 
                   3913: Источник:
                   3914: https://ru.wikiquote.org/wiki/Василий_Васильевич_Кандинский
                   3915: 
                   3916: Автор:
                   3917: Виталий Захарик
                   3918: 
                   3919: Тур:
                   3920: 6 этап
                   3921: 
                   3922: Дата:
                   3923: 13-Apr-2018
                   3924: 
                   3925: Редактор:
                   3926: 1-18 - Василий Бобков и Анастасия Балмакова (Минск); 19-36 - Вера
                   3927: Рабкина (Минск)
                   3928: 
                   3929: Инфо:
                   3930: Василий Бобков и Анастасия Балмакова благодарят за тестирование и ценные
                   3931: замечания соведущие команды, а также лично Веру Рабкину, Дмитрия
                   3932: Медведева, Евгению Гуз, Надежду Потрихалину, Максима Мацуту, Елену
                   3933: Фадееву, Анну Лущик, Александра Столярова, Алексея Блатуна, Михаила
                   3934: Мальцева, Антона Ушкалова, Александра Марцинкевича и команду "Клёк"
                   3935: (Минск). Вера Рабкина выражает благодарность командам "Одушевленные
                   3936: аэросани", "Енотики-7", "И пусть никто не уйдет обиженный", "Эталон
                   3937: этанола", "New-реанимация".
                   3938: 
                   3939: Вопрос 1:
                   3940: Согласно научно-популярному фильму, Дарвин, путешествуя на корабле
                   3941: "Бигль", понимал, что ответ на вопрос о происхождении жизни кроется
                   3942: ПРОПУСК. Пропущенные слова входят в известное пожелание. Напишите
                   3943: пропущенные слова.
                   3944: 
                   3945: Ответ:
                   3946: Под килем.
                   3947: 
                   3948: Комментарий:
                   3949: Дарвин пришел к убеждению, что жизнь зародилась в воде, т.е. буквально
                   3950: под килем корабля. "Семь футов под килем" - известное пожелание удачи.
                   3951: 
                   3952: Источник:
                   3953:    1. Документальный фильм Би-Би-Си "Океаны Солнечной системы".
                   3954:    2. https://frazeolog_ru.academic.ru/433/
                   3955: 
                   3956: Автор:
                   3957: Команда "Одушевленные аэросани"
                   3958: 
                   3959: Вопрос 2:
                   3960: Сол Шульман пишет, что в начале XIX века в Австралии единственной ЕЮ
                   3961: был, как ни странно, ром. Назовите ЕЕ двумя словами.
                   3962: 
                   3963: Ответ:
                   3964: Твердая валюта.
                   3965: 
                   3966: Комментарий:
                   3967: Забавно, что жидкий ром был твердой валютой.
                   3968: 
                   3969: Источник:
                   3970: С. Шульман. Австралия - Terra Incognita: Когда звери еще были людьми.
                   3971: http://flibusta.is/b/312229/read
                   3972: 
                   3973: Автор:
                   3974: Команда "Одушевленные аэросани"
                   3975: 
                   3976: Вопрос 3:
                   3977: Увлечение рыбалкой стало поводом ЕГО знакомства с Фиделем Кастро,
                   3978: которому ОН вручал приз за победу в рыболовных соревнованиях. Назовите
                   3979: ЕГО.
                   3980: 
                   3981: Ответ:
                   3982: [Эрнест] Хемингуэй.
                   3983: 
                   3984: Комментарий:
                   3985: Эрнест Хемингуэй увлекался рыбалкой и долгое время прожил на Кубе.
                   3986: 
                   3987: Источник:
                   3988: https://ru.wikipedia.org/wiki/Дом-музей_Эрнеста_Хемингуэя
                   3989: 
                   3990: Автор:
                   3991: Василий Бобков
                   3992: 
                   3993: Вопрос 4:
                   3994: Одной из причин появления легенд о НИХ являются пузырьки метана, которые
                   3995: вибрируют на частоте инфразвука и могут вызывать у человека панику или
                   3996: страх. Назовите ИХ словом, пишущимся через дефис.
                   3997: 
                   3998: Ответ:
                   3999: Корабли-призраки.
                   4000: 
                   4001: Комментарий:
                   4002: Одна из гипотез гласит, что в районе Бермудского треугольника скопилось
                   4003: огромное количество метана из-за когда-то действовавших на дне вулканов.
                   4004: Температура и давление воды выталкивают газ на поверхность. Люди в
                   4005: страхе и панике просто прыгали за борт и погибали.
                   4006: 
                   4007: Источник:
                   4008: https://www.eg.ru/science/479115/
                   4009: 
                   4010: Автор:
                   4011: Василий Бобков
                   4012: 
                   4013: Вопрос 5:
                   4014: (pic: 20170888.jpg)
                   4015:    Перед вами портрет, на котором изображен Эдвард Джеймс. Назовите
                   4016: автора этого портрета.
                   4017: 
                   4018: Ответ:
                   4019: Рене Магритт.
                   4020: 
                   4021: Комментарий:
                   4022: Любил Магритт вместо головы рисовать что-то другое.
                   4023: 
                   4024: Источник:
                   4025: http://www.vokrugsveta.ru/article/283705/
                   4026: 
                   4027: Автор:
                   4028: Василий Бобков
                   4029: 
                   4030: Вопрос 6:
                   4031: В книге Андрея Фатющенко "Семь чудес света" можно прочесть истории,
                   4032: рассказанные жителями разных стран. Какое слово мы пропустили в
                   4033: предыдущем предложении?
                   4034: 
                   4035: Ответ:
                   4036: Миллиардов.
                   4037: 
                   4038: Комментарий:
                   4039: Книга называется "Семь миллиардов чудес света" и рассказывает о том,
                   4040: насколько каждый человек важен и уникален.
                   4041: 
                   4042: Источник:
                   4043: https://www.ozon.ru/context/detail/id/4794596/
                   4044: 
                   4045: Автор:
                   4046: Анастасия Балмакова
                   4047: 
                   4048: Вопрос 7:
                   4049: Стивен Спилберг с сожалением отмечает, что при общении с современной
                   4050: молодежью разговор занимает пять минут, а ОНА гаджету - двадцать.
                   4051: Благодаря "ЕЙ" в 2008 году одно европейское государство смогло
                   4052: заработать на туристах. Назовите ЕЕ.
                   4053: 
                   4054: Ответ:
                   4055: Молитва.
                   4056: 
                   4057: Комментарий:
                   4058: Стивен Спилберг не одобряет всеобщую погруженность в телефоны и
                   4059: сравнивает позу молодых людей с молящимся человеком. Песня "Молитва"
                   4060: певицы Марии Шерифович позволила Сербии принять конкурс "Евровидение".
                   4061: 
                   4062: Источник:
                   4063:    1. http://readrate.com/rus/news/pervomu-igroku-prigotovitsya-fantastika-osnovannaya-na-realnosti
                   4064:    2. https://www.obozrevatel.com/finance/analytics-and-forecasts/72138-evrovidenie-2017-za-chto-boryutsya-5-gorodov-kandidatov-dyira-v-byudzhete-ili-ryivok-dlya-turizma.htm
                   4065: 
                   4066: Автор:
                   4067: Дарья Соловей
                   4068: 
                   4069: Вопрос 8:
                   4070: (pic: 20170889.jpg)
                   4071:    Александр Пиперски отмечает, что коренные обитатели планеты Пандора
                   4072: используют ЕЕ. Назовите ЕЕ.
                   4073: 
                   4074: Ответ:
                   4075: Восьмеричная система [счисления].
                   4076: 
                   4077: Комментарий:
                   4078: У цивилизации, которая пользуется языком на'ви, на руках по четыре
                   4079: пальца, а не по пять, и им удобно двумя руками досчитать до восьми.
                   4080: 
                   4081: Источник:
                   4082: https://postnauka.ru/video/77568
                   4083: 
                   4084: Автор:
                   4085: Команда "Одушевленные аэросани"
                   4086: 
                   4087: Вопрос 9:
                   4088: Проблему "искусственного полета" удалось успешно решить лишь после того,
                   4089: как братья Райт перестали имитировать птиц и приступили к изучению
                   4090: аэродинамики. Возможно, именно этим объясняется непопулярность одной
                   4091: известной задачи среди ученых. Назовите эту задачу двумя словами,
                   4092: начинающимися на одну и ту же букву.
                   4093: 
                   4094: Ответ:
                   4095: Тест Тьюринга.
                   4096: 
                   4097: Комментарий:
                   4098: Исследователи практически не занимаются решением задачи прохождения
                   4099: теста Тьюринга, считая, что гораздо важнее изучить основополагающие
                   4100: принципы интеллекта, чем продублировать одного из носителей
                   4101: естественного интеллекта.
                   4102: 
                   4103: Источник:
                   4104: https://ru.wikipedia.org/wiki/Тест_Тьюринга
                   4105: 
                   4106: Автор:
                   4107: Сергей Матмусаев
                   4108: 
                   4109: Вопрос 10:
                   4110: Название статьи Александра Зотина о новой гипотетической форме
                   4111: капитализма, при которой доля трудовых доходов упадет до нуля за счет
                   4112: полной автоматизации, лишь одной буквой отличается от реально
                   4113: существующего термина из двух слов. Воспроизведите название этой статьи.
                   4114: 
                   4115: Ответ:
                   4116: "Робовладельческий строй".
                   4117: 
                   4118: Комментарий:
                   4119: В статье рассказывается о том, что доля трудовых доходов будет
                   4120: стремиться к нулю, а доля доходов от капитала, наоборот, приблизится к
                   4121: 100%. Всю работу станут делать роботы, а большинству людей придется
                   4122: сидеть на пособии.
                   4123: 
                   4124: Источник:
                   4125: https://www.kommersant.ru/doc/3455179
                   4126: 
                   4127: Автор:
                   4128: Александр Мартинович
                   4129: 
                   4130: Вопрос 11:
                   4131: Антон Заруцкий пишет, что по структуре перламутр напоминает ЕГО:
                   4132: арагонит чередуется с хитином, который, просачиваясь сквозь поры в
                   4133: пластинках, образует узкие мостики. Назовите ЕГО, использовав имя
                   4134: собственное.
                   4135: 
                   4136: Ответ:
                   4137: Торт "Наполеон".
                   4138: 
                   4139: Комментарий:
                   4140: Перламутр устроен слоями.
                   4141: 
                   4142: Источник:
                   4143: http://www.vokrugsveta.ru/article/279998/
                   4144: 
                   4145: Автор:
                   4146: Василий Бобков
                   4147: 
                   4148: Вопрос 12:
                   4149: В статье журнала "National Geographic" [нэшнл джиогрЭфик] приводится
                   4150: сравнение с мухой, пролетающей мимо включенной фары автомобиля: яркость
                   4151: меняется, и тогда можно обнаружить ЕЕ. Назовите ЕЕ словом греческого
                   4152: происхождения.
                   4153: 
                   4154: Ответ:
                   4155: Планета.
                   4156: 
                   4157: Зачет:
                   4158: Экзопланета.
                   4159: 
                   4160: Комментарий:
                   4161: Для распознавания планет телескоп отслеживает изменение яркости звезд.
                   4162: Оно происходит, когда планета проходит по орбите вокруг звезды и
                   4163: преграждает путь свету, который от нее исходит. Если телескоп фиксирует,
                   4164: что затемнение происходит с определенной периодичностью, значит,
                   4165: вероятнее всего, он обнаружил планету.
                   4166: 
                   4167: Источник:
                   4168: http://www.nat-geo.ru/universe/476151-novaya-zemlya-nasa-obnaruzhilo-pervuyu-ekzoplanetu-pokhozhuyu-na-nashu/
                   4169: 
                   4170: Автор:
                   4171: Дарья Соловей
                   4172: 
                   4173: Вопрос 13:
                   4174: В этом вопросе слово "ИКС" является заменой.
                   4175:    В 1891 году в штат Огайо прибыл Фрэнк Мельбурн, которого называли
                   4176: "волшебником" и "заклинателем". Вскоре после этого местные фермеры
                   4177: договорились вскладчину купить у Мельбурна один ИКС. Рекордное
                   4178: количество ИКСОВ наблюдается на горе ВаиАлеале на Гавайях. Назовите ИКС
                   4179: двумя словами, которые начинаются на одну и ту же букву.
                   4180: 
                   4181: Ответ:
                   4182: Дождливый день.
                   4183: 
                   4184: Зачет:
                   4185: День дождя.
                   4186: 
                   4187: Комментарий:
                   4188: Мельбурн был бродячим заклинателем дождя. Фермеры Огайо страдали от
                   4189: засухи и решили прибегнуть к его услугам. Мельбурну повезло - на
                   4190: следующий день после его манипуляций действительно пошел дождь. На
                   4191: склонах горы Ваиалеале на Гавайях наблюдается около 350 дождливых дней в
                   4192: год, что является мировым рекордом.
                   4193: 
                   4194: Источник:
                   4195:    1. С. Барнетт. Занимательное дождеведение: дождь в истории, науке и
                   4196: искусстве. http://flibusta.is/b/460154/read
                   4197:    2. http://odogde.ru/interesnoe-o-dozhde/samye-dozhdlivye-mesta-i-goroda-mira.html
                   4198: 
                   4199: Автор:
                   4200: Команда "Одушевленные аэросани"
                   4201: 
                   4202: Вопрос 14:
                   4203: (pic: 20170890.jpg)
                   4204:    Перед вами кадр из фильма "Назад в будущее - 3". Забавно, что в 1885
                   4205: году ИХ еще не существовало. Назовите ИХ. Принимается описательный
                   4206: ответ.
                   4207: 
                   4208: Ответ:
                   4209: Шрифты на надгробии.
                   4210: 
                   4211: Зачет:
                   4212: Шрифт Helvetica, шрифт EuroStile, а также описательные ответы с
                   4213: упоминанием слова "шрифт" или "гарнитура". Незачет: Шрифты с засечками,
                   4214: шрифты без засечек, курсив.
                   4215: 
                   4216: Комментарий:
                   4217: Шрифты Helvetica и EuroStile появились значительно позже (в 1957 и 1962
                   4218: годах соответственно). Забавно, что и здесь наблюдается в некотором
                   4219: смысле путешествие [шрифта] во времени.
                   4220: 
                   4221: Источник:
                   4222:    1. Нетология: лекция "Основы графического дизайна: композиция, цвет,
                   4223: типографика".
                   4224:    2. Х/ф "Назад в будущее - 3" (1990), реж. Роберт Земекис.
                   4225:    3. https://ru.wikipedia.org/wiki/Гельветика
                   4226:    4. https://en.wikipedia.org/wiki/Eurostile
                   4227: 
                   4228: Автор:
                   4229: Анастасия Балмакова
                   4230: 
                   4231: Вопрос 15:
                   4232: [Ведущему: сделать небольшую паузу после слова "лавы".]
                   4233:    При излиянии темной жидкой лавы более легкий и светлый кварц
                   4234: "выдавливается" из породы и застывает в трещинах. Ричард Форти
                   4235: сравнивает кварцевую массу с НЕЙ. Назовите ЕЕ.
                   4236: 
                   4237: Ответ:
                   4238: СУкровица.
                   4239: 
                   4240: Зачет:
                   4241: Лимфа.
                   4242: 
                   4243: Комментарий:
                   4244: Кварцевая масса застывает в трещинах породы подобно сукровице в ране.
                   4245: Кровь вытекает из раны, лава - из Земли. Такая вот анатомическая
                   4246: аналогия.
                   4247: 
                   4248: Источник:
                   4249: Р. Форти. Трилобиты: свидетели эволюции.
                   4250: http://flibusta.is/b/363864/read
                   4251: 
                   4252: Автор:
                   4253: Анастасия Балмакова
                   4254: 
                   4255: Вопрос 16:
                   4256: Тираннозавры жили в позднем меловом периоде, в длину они достигали 35-40
                   4257: метров, а весили до 110 тонн. Какое слово мы немного изменили в
                   4258: предыдущем предложении?
                   4259: 
                   4260: Ответ:
                   4261: Титанозавры.
                   4262: 
                   4263: Комментарий:
                   4264: Тираннозавры, конечно, были страшными, но их масса была всего около 6
                   4265: тонн. Гигантские же стотонные динозавры за свой размер получили название
                   4266: титанозавры.
                   4267: 
                   4268: Источник:
                   4269: https://ru.wikipedia.org/wiki/Титанозавры
                   4270: 
                   4271: Автор:
                   4272: Анастасия Балмакова
                   4273: 
                   4274: Вопрос 17:
                   4275:    <раздатка>
                   4276:    Alakshak
                   4277:    </раздатка>
                   4278:    Розданное вам слово с алеутского переводится как "великие зЕмли", или
                   4279: ОН. Назовите ЕГО двухкоренным словом.
                   4280: 
                   4281: Ответ:
                   4282: Полуостров.
                   4283: 
                   4284: Комментарий:
                   4285: Розданное вам слово означает Аляску. Аляска - полуостров на
                   4286: северо-западе Северной Америки.
                   4287: 
                   4288: Источник:
                   4289: https://ibigdan.livejournal.com/14828195.html
                   4290: 
                   4291: Автор:
                   4292: Василий Бобков
                   4293: 
                   4294: Вопрос 18:
                   4295: Памятник-фонтан в саду музея в районе Мейлахти представляет собой ЕГО.
                   4296: Ольга Воробьёва называет ЕГО "желтобрюхим божеством". Назовите ЕГО.
                   4297: 
                   4298: Ответ:
                   4299: Самовар.
                   4300: 
                   4301: Комментарий:
                   4302: Памятник-фонтан находится в Хельсинки, и здесь, кроме столика с
                   4303: самоваром и заварочным чайником, есть еще две чашки с блюдцами и
                   4304: подсвечник. Две струи воды, из заварочного чайника на самоваре и из
                   4305: краника самовара, льются прямо на стол.
                   4306: 
                   4307: Источник:
                   4308: https://www.nkj.ru/open/32614/
                   4309: 
                   4310: Автор:
                   4311: Василий Бобков
                   4312: 
                   4313: Вопрос 19:
                   4314: При въезде в какой европейский город вас встречает плакат, текст
                   4315: которого можно перевести как "Приятно познакомиться!"?
                   4316: 
                   4317: Ответ:
                   4318: Ницца.
                   4319: 
                   4320: Источник:
                   4321: ЛНА.
                   4322: 
                   4323: Автор:
                   4324: Дмитрий Медведев
                   4325: 
                   4326: Вопрос 20:
                   4327: Герой Роджера Желязны сравнивает медсестру с АЛЬФОЙ, но замечает, что
                   4328: отличие в том, что все медсестры похожи друг на друга. Какое слово мы
                   4329: заменили АЛЬФОЙ?
                   4330: 
                   4331: Ответ:
                   4332: Снежинка.
                   4333: 
                   4334: Комментарий:
                   4335: Как известно, все снежинки разные.
                   4336: 
                   4337: Источник:
                   4338: Р. Желязны. Знак Единорога. http://flibusta.is/b/139509/read
                   4339: 
                   4340: Автор:
1.2     ! rubashki 4341: Никита Геер
1.1       rubashki 4342: 
                   4343: Вопрос 21:
                   4344: Штат Невада пустынен и беден фауной, поэтому в конце 2016 года в
                   4345: качестве НЕГО был выбран рыцарь. Назовите ЕГО.
                   4346: 
                   4347: Ответ:
                   4348: Название [хоккейной] команды.
                   4349: 
                   4350: Зачет:
                   4351: Маскот/талисман/логотип [хоккейной] команды; маскот/талисман/логотип.
                   4352: 
                   4353: Комментарий:
                   4354: В конце 2016 года в НХЛ добавился 31-й клуб - из Лас-Вегаса. Из-за
                   4355: скудности выбора команду пришлось назвать не в честь животных, а в честь
                   4356: рыцарей.
                   4357: 
                   4358: Источник:
                   4359: https://www.americansport.ru/las-vegas-golden-knights-komanda-nhl/
                   4360: 
                   4361: Автор:
1.2     ! rubashki 4362: Павел Новиков (Витебск)
1.1       rubashki 4363: 
                   4364: Вопрос 22:
                   4365: Профессор Карл Кокелл разработал тартан для ношения во время
                   4366: определенной научно-исследовательской деятельности. Синий цвет на
                   4367: тартане символизирует наличие воды в прошлом, две толстые белые линии -
                   4368: это полюсы. А что символизирует красный фон?
                   4369: 
                   4370: Ответ:
                   4371: Поверхность Марса.
                   4372: 
                   4373: Зачет:
                   4374: Марс.
                   4375: 
                   4376: Источник:
                   4377: https://mi3ch.livejournal.com/3799668.html
                   4378: 
                   4379: Автор:
                   4380: Надежда Потрихалина
                   4381: 
                   4382: Вопрос 23:
                   4383: Клоч - это главное блюдо зороастрийского праздника Сарсал, отмечающегося
                   4384: весной. Ответьте двумя словами: что принято разбрасывать по полям и
                   4385: огородам в этот день для повышения плодородности?
                   4386: 
                   4387: Ответ:
                   4388: Яичная скорлупа.
                   4389: 
                   4390: Зачет:
                   4391: Яичные очистки; скорлупа яиц.
                   4392: 
                   4393: Комментарий:
                   4394: "Клоч" крайне созвучен со словом "кулич", у Сарсала и Пасхи много схожих
                   4395: черт: пироги-куличи, крашеные яйца, празднование весной. Также известно,
                   4396: что яичная скорлупа является хорошим удобрением, так что неудивительно,
                   4397: что обряд эффективен.
                   4398: 
                   4399: Источник:
                   4400: https://kulturologia.ru/blogs/070418/38512/
                   4401: 
                   4402: Автор:
                   4403: Вера Рабкина
                   4404: 
                   4405: Вопрос 24:
                   4406: После смерти Маяковского распространился слух, будто поэт был убит.
                   4407: Появлению слуха способствовал Константин Луцкий, который по неопытности
                   4408: использовал недостаточное количество вазелина при изготовлении ЕЕ.
                   4409: Назовите ЕЕ двумя словами.
                   4410: 
                   4411: Ответ:
                   4412: Посмертная маска.
                   4413: 
                   4414: Комментарий:
                   4415: Скульптор Константин Луцкий снимал с Маяковского посмертную маску, но
                   4416: плохо смазал лицо вазелином, сорвал кожу со щеки и переносицы, отчего
                   4417: возник слух, будто к Маяковскому был подослан убийца, поэт сопротивлялся
                   4418: и в драке получил рассечение щеки.
                   4419: 
                   4420: Источник:
                   4421: Д.Л. Быков. Тринадцатый апостол. Маяковский. Трагедия-буфф в шести
                   4422: действиях. http://flibusta.is/b/450896/read
                   4423: 
                   4424: Автор:
                   4425: Команда "ИПННУО"
                   4426: 
                   4427: Вопрос 25:
                   4428: Роман "Толстая тетрадь" рассказывает о судьбе двух близнецов,
                   4429: разлученных в детстве. Их судьбы хоть и похожи, но перемешались совсем
                   4430: разным образом. Одного из братьев зовут Лукас. Как зовут второго?
                   4431: 
                   4432: Ответ:
                   4433: Клаус.
                   4434: 
                   4435: Комментарий:
                   4436: Их имена являются анаграммами.
                   4437: 
                   4438: Источник:
                   4439: А. Кристоф. Толстая тетрадь. http://flibusta.is/b/144373/read
                   4440: 
                   4441: Автор:
                   4442: Вера Рабкина
                   4443: 
                   4444: Вопрос 26:
                   4445: Из-за близости к буддизму на ИХ форме начала XX века советская звезда
                   4446: соседствовала с желтой свастикой, что впоследствии привело к репрессиям.
                   4447: Рассказывают, что в 1949 году при отмечании юбилея даже решили опустить
                   4448: некоторые строки произведения. Назовите это произведение.
                   4449: 
                   4450: Ответ:
                   4451: "Памятник".
                   4452: 
                   4453: Зачет:
                   4454: "Я памятник себе воздвиг нерукотворный...".
                   4455: 
                   4456: Комментарий:
                   4457: Речь идет о калмыках. Когда отмечался 150-летний юбилей Пушкина, во
                   4458: время публичного чтения по радио стихотворения "Памятник" было опущено
                   4459: часть строк, чтобы избежать произнесения слов "... и друг степей
                   4460: калмык".
                   4461: 
                   4462: Источник:
                   4463: https://kulturologia.ru/blogs/080418/38529/
                   4464: 
                   4465: Автор:
                   4466: Вера Рабкина
                   4467: 
                   4468: Вопрос 27:
                   4469: Фредрик Шёберг рассказывает о местах, где развитие фауны и флоры пошло с
                   4470: нуля. Как пример он приводит остров Суртсей вблизи Исландии, а также
                   4471: упоминает другой остров в южном полушарии. Какой?
                   4472: 
                   4473: Ответ:
                   4474: Кракатау.
                   4475: 
                   4476: Комментарий:
                   4477: "... на острове вулканического происхождения Суртсей, выступившем из
                   4478: моря около Исландии. Или на Кракатау в Зондском проливе, между островами
                   4479: Ява и Суматра, где в 1883 году взорвался вулкан, вследствие чего дальше
                   4480: развитие и фауны и флоры пошло с нуля".
                   4481: 
                   4482: Источник:
                   4483:    1. Ф. Шёберг. Ловушка Малеза, или О счастье жить в плену необычной
                   4484: страсти, мухах и причудах судьбы. http://flibusta.is/b/442189/read
                   4485:    2. https://ru.wikipedia.org/wiki/Кракатау
                   4486: 
                   4487: Автор:
1.2     ! rubashki 4488: Никита Геер
1.1       rubashki 4489: 
                   4490: Вопрос 28:
                   4491: Автор вопроса, надеясь провести время в тишине, всегда при покупке
                   4492: билета на поезд выбирает одни и те же место и вагон. Какие?
                   4493: 
                   4494: Ответ:
                   4495: 4 33.
                   4496: 
                   4497: Зачет:
                   4498: 33 4.
                   4499: 
                   4500: Комментарий:
                   4501: Очевидно, автор вопроса считает, что есть связь между местом в поезде и
                   4502: сочинением американского композитора Джона Кейджа 4'33", на всём
                   4503: протяжении которого участники ансамбля не извлекают звуков из своих
                   4504: инструментов.
                   4505: 
                   4506: Источник:
                   4507:    1. ЛНА.
                   4508:    2. https://ru.wikipedia.org/wiki/4%E2%80%B233%E2%80%B3
                   4509: 
                   4510: Автор:
                   4511: Евгения Гуз
                   4512: 
                   4513: Вопрос 29:
                   4514: Шелковая АЛЬФА является обязательным атрибутом калмыцкого головного
                   4515: убора. Другая АЛЬФА, согласно легенде, помогла определить, кому
                   4516: принадлежит главенство над землями. Назовите АЛЬФУ двумя словами,
                   4517: начинающимися на одну и ту же букву.
                   4518: 
                   4519: Ответ:
                   4520: Красная кисть.
                   4521: 
                   4522: Комментарий:
                   4523: Главенство над Ольстером было решено в результате состязания - нужно
                   4524: было пересечь водный поток и первым коснуться рукой берега; для победы
                   4525: один из состязавшихся отрезал себе кисть и кинул ее на берег,
                   4526: впоследствии красная кисть Ольстера стала геральдическим символом.
                   4527: 
                   4528: Источник:
                   4529:    1. https://kulturologia.ru/blogs/080418/38529/
                   4530:    2. https://ru.wikipedia.org/wiki/Красная_рука_Ольстера
                   4531: 
                   4532: Автор:
                   4533: ???
                   4534: 
                   4535: Вопрос 30:
                   4536: В ходе изучения системы СКЛАДЫВАНИЯ крыльев божьей коровки, ученые
                   4537: ТОКИЙСКОГО университета пришли к выводу, что крылья имеют много общего с
                   4538: ЭТИМ. Назовите ЭТО заимствованным словом.
                   4539: 
                   4540: Ответ:
                   4541: Оригами.
                   4542: 
                   4543: Комментарий:
                   4544: Компактность и способ складывания крыльев напомнило японцам искусство их
                   4545: страны - оригами.
                   4546: 
                   4547: Источник:
                   4548: https://www.sciencedaily.com/releases/2017/05/170515150714.htm
                   4549: 
                   4550: Автор:
                   4551: Вера Рабкина
                   4552: 
                   4553: Вопрос 31:
                   4554: В белорусском языке это заболевание носит название "прАнцы", что связано
                   4555: с вражескими войсками, которые его завезли. Назовите это заболевание
                   4556: по-русски.
                   4557: 
                   4558: Ответ:
                   4559: Сифилис.
                   4560: 
                   4561: Комментарий:
                   4562: Слово "пранцы" происходит от слова "Франция". Сифилис также называют
                   4563: "французской болезнью", что нашло отражение и в белорусском языке.
                   4564: 
                   4565: Источник:
                   4566: ???
                   4567: 
                   4568: Автор:
                   4569: Надежда Потрихалина
                   4570: 
                   4571: Вопрос 32:
                   4572: "Жизнь после людей" - это научно-популярный фильм, который рассказывает,
                   4573: что будет с планетой после исчезновения человечества. Какой город
                   4574: приводится в качестве иллюстрации мира через двадцать лет после
                   4575: исчезновения людей?
                   4576: 
                   4577: Ответ:
                   4578: Припять.
                   4579: 
                   4580: Источник:
                   4581: https://ru.wikipedia.org/wiki/Жизнь_после_людей
                   4582: 
                   4583: Автор:
                   4584: Команда "ИПННУО"
                   4585: 
                   4586: Вопрос 33:
                   4587: В книге "Вот я" Джонатан Фоер пишет, что иноземные садоводы подрЕзали
                   4588: ЕГО Исаака до самой галисийской почвы. Назовите ЕГО двумя словами.
                   4589: 
                   4590: Ответ:
                   4591: Семейное древо.
                   4592: 
                   4593: Зачет:
                   4594: Генеалогическое древо.
                   4595: 
                   4596: Комментарий:
                   4597: Садоводы были немецкие. Так Фоер иносказательно описывает Холокост.
                   4598: 
                   4599: Источник:
                   4600: Дж.С. Фоер. Вот я. http://flibusta.is/b/507480/read
                   4601: 
                   4602: Автор:
                   4603: Вера Рабкина
                   4604: 
                   4605: Вопрос 34:
                   4606:    <раздатка>
                   4607:    WAM
                   4608:    </раздатка>
                   4609:    В сериале о классической музыке над вариантом ИКСА работает робот под
                   4610: кодовым именем WAM [даблъю-эй-эм]. Назовите ИКС одним словом.
                   4611: 
                   4612: Ответ:
                   4613: Реквием.
                   4614: 
                   4615: Комментарий:
                   4616: В сериале робот, проанализировав все работы Моцарта, пытается закончить
                   4617: Реквием так, как его закончил бы сам мастер. Сам же робот получил
                   4618: название по инициалам композитора - Вольфганг Амадей Моцарт.
                   4619: 
                   4620: Источник:
                   4621: Телесериал "Моцарт в джунглях", s04e06.
                   4622: 
                   4623: Автор:
                   4624: Вера Рабкина
                   4625: 
                   4626: Вопрос 35:
                   4627: Чтобы обеспечить поселение Рьюкан достаточным ИКСОМ, на вершине горы
                   4628: возле него установили огромные объекты, изменяющие положение каждые 10
                   4629: секунд. Мы не спрашиваем, что это за объекты. Назовите ИКС двумя
                   4630: словами.
                   4631: 
                   4632: Ответ:
                   4633: Солнечный свет.
                   4634: 
                   4635: Комментарий:
                   4636: Поселение расположено в долине таким образом, что почти полгода до него
                   4637: не достают солнечные лучи. На вершине горы поставили сразу три огромных
                   4638: зеркала, которые меняют свое положение каждые 10 секунд, следуя за
                   4639: лучами солнца. Отраженный свет освещает примерно 600 квадратных метров,
                   4640: что полностью покрывает центральную площадь городка.
                   4641: 
                   4642: Источник:
                   4643: https://kulturologia.ru/blogs/090418/38536/
                   4644: 
                   4645: Автор:
                   4646: Вера Рабкина
                   4647: 
                   4648: Вопрос 36:
                   4649: Комментируя чрезмерную активность советской цензуры, персонаж Шамиля
                   4650: Идиатуллина шутит, что скоро все иностранные фильмы станут ТАКИМИ.
                   4651: Ответьте, используя слово с двумя корнями: какими - ТАКИМИ?
                   4652: 
                   4653: Ответ:
                   4654: Короткометражными.
                   4655: 
                   4656: Комментарий:
                   4657: Советская власть вырезАла из иностранных фильмов неугодные куски, из-за
                   4658: чего фильмы всё больше и больше сокращались.
                   4659: 
                   4660: Источник:
                   4661: Ш.Ш. Идиатуллин. Город Брежнев. http://flibusta.is/b/477611/read
                   4662: 
                   4663: Автор:
                   4664: Вера Рабкина
                   4665: 

FreeBSD-CVSweb <freebsd-cvsweb@FreeBSD.org>